You are on page 1of 114

TSPSC Group 1 Prelims 2023

200 Important MCQs


from Science & Technology, Environment

Ravi Garlapati
Dear Aspirants
In TSPSC Group 1 Exam, Science & Technology, Environment
has 30 marks (/150) weightage in Prelims and 150 marks (/900)
weightage in Mains making it one of the most important
subjects, a game changer, for you to get your dream job.

Given the vast & complex syllabus, you need an uncomplicated


book that is designed to help you
understand the concepts and retain them
easily.

I know the pain of sincere aspirants. I


poured all my expertise & experience as
a teacher and as a successful govt. job
aspirant into creating a book specifically
designed for Group 1 exam preparation.

This book is meticulously researched,


coherently written, and easy to
understand & retain. It is thoughtfully
made aspirant friend in every way
possible for a book.

This PDF is excerpted from the book. It contains both static and
dynamic current affairs from Science & Technology, Environment.
Hope it helps you.

Ravi Garlapati,
Faculty & Author, Science & Tech; Superintendent of Customs,
Mumbai
 Buy the book on Amazon: https://amzn.eu/d/gNZielj
 Buy the readable PDF for only Rs. 60/- on ‘Simplify Success’ app on
Playstore or www.simplify-success.com
 Join https://t.me/simplifysuccess for Quality Material
Classification of Questions from Science & Technology,
Environment in TSPSC Group 1 Preliminary Test 2022
Area No. Of Static No. Of dynamic Total No. Of
questions questions* Questions
General Science & Science and 15 08 23
Technology
Environment 02 02 04
Disaster Management 0 03 03
Total 17 13 30
* Dynamic questions means questions from current affairs

TSPSC Group-I Services Prelims Syllabus

1. Current Affairs – Regional, National & International.


2. International Relations and Events.
3.General Science; India’s Achievements in Science and Technology.
4. Environmental Issues: Disaster Management- Prevention and Mitigation Strategies.
5. Economic and Social Development of India.
6. World Geography, Indian Geography and Geography of Telangana state.
7. History and Cultural Heritage of India.
8. Indian Constitution and Polity.
9. Governance and Public Policy in India.
10. Policies of Telangana State.
11. Society, Culture, Heritage, Arts and Literature of Telangana.
12. Social Exclusion: Rights issues such as Gender, Caste, Tribe, Disability etc. and
inclusive policies.
13. Logical Reasoning: Analytical Ability and Data Interpretation.

Note: MCQs are given from the highlighted parts above i.e., point 3 and point 4 are covered in this
book.

Science & Technology and Environment by Ravi Garlapati Page 1


Science and Technology MCQs
Note: For conceptual clarity – refer to the chapters in Part – I & Part – II of the book
‘Science & Technology, Environment by Ravi Garlapati’.

Exercise - I Select the correct answer using the


Topics: Emerging Technologies, ICT, Nano codes given below:
Technology, Robotics & AI a) 1, 3 and 4 only
1. Regarding Crypto-currency b) 1 and 4 only
transactions, which of the following c) 2 and 3 only
statements is accurate regarding the d) 1, 2, 3 and 4
'Proof-of-Stake' mechanism that has
been in the news recently? 3. Consider the following the initiatives
a) This mechanism selects the miner of Government of India. Which of them
who solves the mathematical puzzle are aimed to promote the
first as the validator. development of Artificial Intelligence?
b) Validators are randomly assigned 1. Visvesvaraya PhD Scheme
from a pool of people using an 2. FutureSkills PRIME initiative
algorithm in this process. 3. National Mission on
c) It is a method of dividing Interdisciplinary Cyber-Physical
transactions along the different chains Systems
to decrease the cost and accelerate the 4. Mission LiFE
transactions. 5. RAISE scheme
d) This mechanism enables users to Select the correct answer using the
become validators without storing codes given below:
large data on their computer. a) 1, 3 and 4 only
b) 2, 3 and 5 only
2. Non-Fungible Tokens (NFTs) are c) 1, 2, 3 and 5 only
unique digital assets that have been in
d) 1, 2, 3, 4 and 5
the news for quite some time. Which
of the following are possible uses of
Non-Fungible Tokens? 4. In relation to Additive Manufacturing,
1. NFTs can be a new medium of which of the following statements is
investment for various investors. correct?
1. It involves creating a digital design
2. NFTs can enable true ownership of
of the object using a 3D modeling
digital art.
program.
3. NFTs can potentially reduce
2. It involves cutting away material
instances of piracy and fraud in the
from a solid piece to create a
music industry.
physical object.
4. NFTs can aid in the storage of
3. It results in less material waste
medical records and clinical data.
compared to traditional
manufacturing processes.

Science & Technology and Environment by Ravi Garlapati Page 2


Which of the statements given above b) Replacing copper wires with
is/are correct? optical fibers for data
a) 1 only transmission
b) 1 and 3 only c) The process of using optical fibers
c) 2 and 3 only to connect radio towers
d) 1, 2 and 3 d) The increased usage of the
internet in remote and rural areas
of a country.
5. Which of the following technological
applications correctly represent
Cyber-Physical systems? 8. Recently seen in news media "Horizon
1. A self-driving car making decisions scanning" is related to:
about braking, acceleration, and a) the process to diagnose the cancer
changing lanes. cells in the human body.
b) the early detection and assessment
2. A pacemaker automatically
of emerging technologies or
adjusting the heart rate to
threats, which aids in making
maintain the desired pace.
policy decisions.
3. A drone utilizing GPS, cameras,
c) a global network of synchronized
and lidar for navigation.
radio observatories that observe
a) 1 and 2 only b) 2 and 3 only
radio sources associated with
c) 1 and 3 only d) 1, 2 and 3 black holes.
d) involves scanning the soil surface
6. Consider the following statements to identify physical, chemical, and
about 3D printing and 4D printing: biological characteristics of soil.
1. In 3D printing, a 2D structure is
repeated layer by layer to create a 9. How does the use of graphene in
three-dimensional object. bionics relate to the construction of
2. 4D printing allows printed objects artificial systems that have some of
to change their form or function the characteristics of living systems?
over time. 1. Graphene is impermeable to
Which of the statements given above harsh ionic solutions.
is/are correct? 2. Graphene has good electrical
a) 1 only b) 2 only conductivity.
c) Both 1 and 2 d) Neither 1 nor 2 3. Graphene is flexible.
Select the correct answer using the
7. What is the best definition of code given below.
'fiberisation' in the context of a) 1 only b) 2 and 3 only
communication technology? c) 3 only d) 1, 2 and 3
a) The use of LAN cables to access
networks in areas where radio 10. Consider the following statements
wave transmission is not possible about spoofing and phishing:

Science & Technology and Environment by Ravi Garlapati Page 3


1. Spoofing refers to the practice of
stealing the identity of a legitimate Select the correct answer using the
user. code given below:
2. Phishing involves stealing a) 1, 2 and 4 only
sensitive information, such as bank b) 1 and 4 only
account details, from users. c) 2 and 3 only
Which of the statements given above d) 1, 3 and 4 only
is/are correct?
a) 1 only b) 2 only 14. What are the potential benefits of net
c) Both 1 and 2 d) Neither 1 nor 2 neutrality? Choose from the options
below:
11. Which of the following cyber attacks 1. Preserves consumers' right to
involves a phone call or voicemail freely choose and access internet
message? content.
a)DoS b) Smishing 2. Allows access to information
c)Vishing d) Cryptojacking without location-based
restrictions.
12. Consider the following statements 3. Significantly lowers the cost of
regarding Standalone 5G and Non- internet services for consumers.
Standalone 5G: 4. Encourages social inclusivity by
1. In Standalone mode, the 5G catering to a diverse range of
network operates independently users.
of the existing 4G network. a) 1 and 2 only
2. Non-standalone networks utilize b) 2 and 3 only
the existing infrastructure for c) 1, 2 and 4 only
deployment. d) 2, 3 and 4 only
Which of the statements given above
is/are correct? 15. Which of the following are the issues
a) 1 only b) 2 only associated with using Virtual Private
c) Both 1 and 2 d) Neither 1 nor 2 Network (VPN)?
1. Slower internet connection speeds
13. The Indian Farmers Fertilizer 2. Insufficient protection against
Cooperative (IFFCO) has developed online threats
the world's first Liquid Nano Urea. 3. Higher cost of doing business
What benefits does it offer over 4. Loss of data or connection
traditional Urea? 5. Inability to bypass geographical
1. The plant leaves absorb it more restrictions.
efficiently.
2. It has a longer shelf life than Select the correct answer using the codes
traditional urea. given below:
3. It can significantly reduce water a) 1 and 2 only
consumption for irrigation. b) 2 and 5 only
4. It can aid in reducing air pollution. c) 1, 3 and 5 only

Science & Technology and Environment by Ravi Garlapati Page 4


d) 1, 2, 3 and 4 only statements is/are correct? (UPSC
PRELIMS 2016)
16. In the current digital age, businesses 1. It uses light as the medium for high-
rely heavily on data for daily activities speed data transmission.
and financial transactions. In this 2. It is a wireless technology and is
context, which of the following several times faster than ‘Wi-Fi’.
statements accurately describes 'Edge Select the correct answer using the
Data Centers'? code given below.
a) They are data centers that provide a) 1 only b) 2 only
cloud-based services to customers c) Both 1 and 2 d) Neither 1 nor 2
through the internet.
b) They are data centers that offer 19. Which of the following statements
space for multiple organizations to regarding the Open Network for
store their own servers and Digital Commerce (ONDC) project
networking equipment. is/are true?
c) They are small, decentralized 1. ONDC is responsible for regulating
facilities located close to the data the e-commerce ecosystem in
source they process. India.
d) They are solely owned and 2. The ONDC project advocates for an
operated by an organization, open network that facilitates the
supporting their internal operations exchange of goods and services
and services. over digital networks.
3. Consumers can make purchases
17. With reference to ‘Near Field using any platform since ONDC is
Communication (NFC) Technology’, not platform-specific.
which of the following statements (a) 1 only (b) 2 and 3 only
is/are correct? (UPSC PRELIMS 2015) (c) 2 only (d) 1 and 2 only
1. It is a contactless communication
technology that uses electromagnetic 20. Match the following Artificial
radio fields. Intelligence tools with their origin
2. NFC is designed for use by devices company
which can be at a distance of even a Tool Company
metre from each other. 1. Watson Studio A. Open AI
3. NFC can use encryption when
sending sensitive information. 2. AlphaFold B. IBM
Select the correct answer using the 3. ChatGPT C. DeepMind
code given below.
a) 1 and 2 only b) 3 only
c) 1 and 3 only d) 1, 2 and 3 Choose the correct answer from the
options below.
18. With reference to ‘Li-Fi’, recently in a) 1-C, 2-B, 3-A b) 1-A, 2-C, 3-B
the news, which of the following c) 1-A, 2-B, 3-C d) 1-B, 2-C, 3-A

Science & Technology and Environment by Ravi Garlapati Page 5


.
Exercise – I Key
1. b 2. d 3. c 4. b 5. d 6. c 7. c 8. b 9. d 10. c
11. c 12. c 13. a 14. c 15. d 16. c 17. c 18. c 19. b 20. d

Explanations

1. The Ethereum blockchain platform has completely shifted from a 'proof of work'
consensus mechanism to a 'proof-of-stake' mechanism. Option a is incorrect as it
refers to the proof-of-work mechanism. Under proof of work, miners compete to
solve complex mathematical puzzles using specialized computer hardware, and the
first to solve the puzzle becomes the validator. Option b is correct as it accurately
describes the Proof-of-Stake consensus mechanism, which randomly assigns a
validator using an algorithm from a pool of people who 'stake' their coins. Option c is
incorrect as sharding in crypto refers to the process of dividing transactions across
several different chains to decrease fees and increase transaction speed. Option d is
incorrect as it pertains to The Verge, a cryptocurrency that allows users to become
validators on the network without storing extensive data on their computers.

2. Non-fungible tokens are digital assets that are unique and stored on a blockchain, the
same network used for cryptocurrencies. They can include anything digital, such as
images, videos, music, and online articles, and can be converted into NFTs and
monetized. NFTs are not digital art themselves but instead serve as certificates of
authenticity, and most use the Ethereum blockchain, the second-largest
cryptocurrency.
Statement 1 is correct: The uniqueness and scarcity of NFTs make them appealing to
investors looking for new and innovative investment opportunities.
Statement 2 is correct: Artists worldwide can convert their digital artwork, graphical
designs, or photographs into NFTs that can be bought and sold.
Statement 3 is correct: NFTs can provide a way to prove ownership and authenticity
of digital music, which can help reduce piracy and fraud in the music industry.
Musicians can sell their music directly to buyers using NFTs, and buyers can prove
that they own a unique piece of music.
Statement 4 is correct: NFTs can be used to store medical records and clinical data,
providing a secure and private way to access data only to authorized individuals.

3. Option 1 is correct: The Government of India has launched the 'Visvesvaraya PhD
Scheme' to boost the number of PhDs in Electronics System Design & Manufacturing
(ESDM) and IT/IT Enabled Services (IT/ITES) sectors, with a focus on research areas
such as Artificial Intelligence and Machine Learning.
Option 2 is correct: The Ministry of Electronics and IT, in collaboration with
NASSCOM, has launched a programme called FutureSkills PRIME to re-skill/ up-skill
IT professionals in ten emerging areas, including Artificial Intelligence.
Science & Technology and Environment by Ravi Garlapati Page 6
Option 3 is correct: The Department of Science & Technology is executing the
National Mission on Interdisciplinary Cyber-Physical Systems (NM-ICPS), which
includes the establishment of 25 Technology Innovation Hubs in prominent
institutions across the country, with a focus on advanced technologies such as
Machine Learning and Artificial Intelligence.
Option 4 is incorrect: Mission LiFE (Lifestyle for Environment) was launched by
Prime Minister Modi at COP26 in November 2021. It is a global mass movement led
by India aimed at encouraging individual and collective behaviors that promote the
protection and preservation of the environment. As part of Mission LiFE, the Indian
Government published a list of 75 lifestyle practices that promote climate-friendly
behaviors.
Option 5 is correct: In 2020, the Government of India held Responsible AI for Social
Empowerment (RAISE), the first-ever global conference on Artificial Intelligence, to
drive India's vision and roadmap for social transformation, inclusion, and
empowerment through responsible AI.

4. The Ministry of Electronics & Information Technology has released a “National


Strategy on Additive Manufacturing”. It aims to position India as a global hub for
Additive Manufacturing development and deployment.
Statement 1 is correct: Under additive manufacturing technology, a virtual design of
the object is created using a 3D modeling program based on Computer Aided Design
(CAD). The CAD software is used to create a digital 3D model of the object, which can
then be converted into a physical object using an additive manufacturing process.
3D printer is controlled by a computer, which receives the instructions from the CAD
software and controls the build process..
Statement 2 is incorrect: Additive manufacturing, also known as 3D printing, is a
process of creating a physical object by laying down successive layers of material.
The object is built up layer by layer, rather than being cut away from a piece of solid
material as in traditional manufacturing methods.
Subtractive manufacturing, also known as traditional machining, is a process of
creating a physical object by cutting away material from a solid piece of material.
Statement 3 is correct: In traditional manufacturing methods such as subtractive
machining, a large amount of material is cut away or removed to create the desired
shape of the object. This results in a significant amount of material waste. In contrast,
additive manufacturing builds up the object layer by layer, using only the material
that is needed to create the final shape. This process results in minimal material
waste, as only the necessary amount of material is used.

5. CPS or Cyber-Physical Systems integrate computation and physical processes to


create a closed-loop system that interacts with the physical world. They use sensors
to gather data from the physical environment, computational elements to process the
data, and actuators to perform actions in the physical world. Examples of CPS include
self-driving cars, medical devices, and industrial control systems. Correct examples of
Science & Technology and Environment by Ravi Garlapati Page 7
CPS are: a self-driving car that uses computational models and sensors to make
decisions about its movement, a pacemaker that adjusts the pace of the heart based
on sensor data, and a drone that navigates using a combination of sensors and
computational models while following a pre-defined flight path.

6. Automated 3D printing is a process of constructing a three-dimensional object by


adding material rather than removing it, using an additive process. The process
involves laying down successive layers of material until the object is complete.
The key difference between 3D printing and 4D printing is that 4D printing is a
subset of 3D printing that enables printed objects to change their form or function
with time, based on external stimuli like heat, water, current, or light, by
incorporating smart design or responsive materials that cause time-dependent
deformations. In contrast, 3D printing is primarily concerned with repeating a 2D
structure layer by layer in a print path from the bottom to the top until a 3D volume
is produced. Therefore, statement 1 about 3D printing is correct, and statement 2
about 4D printing is also correct.

7. Fiberisation refers to the process of connecting radio or cell towers to each other
using optical fiber cables. This is crucial for achieving high-speed data transfer rates
and low-latency connectivity required by 5G technology. Therefore, option (c) is the
correct answer.
Currently, only 33% of the towers in India are connected through fiber optic cables,
which is significantly lower than the 80-90% fiberisation rate in the United States,
Japan, and China. To transition to 5G, India needs to increase its fiberisation by at
least 16 times.
8. Horizon scanning is a methodical technique that aims to identify significant
upcoming developments and their potential implications over the next ten years,
particularly in the context of emerging technology and its impact. The primary
objective is to determine whether individuals or organizations are adequately
equipped to deal with future changes or risks.
When executed consistently and efficiently, horizon scanning can be merged with
other forecasting tools to assist policymakers in recognizing crucial necessities or
inadequacies. It is also a valuable approach for bringing together experts from
diverse domains to cooperate on a shared problem and create practical solutions.
Therefore, option (b) is correct.

9. Graphene, which is composed of a single layer of carbon atoms bound hexagonally,


has the potential to revolutionize bionics by enabling the creation of replacement
ears and eyes that are more organic than mechanical.
Graphene's resistance to corrosive ionic solutions in the human body makes it less
susceptible to damage. Additionally, its electrical conductivity allows it to interact
with cells that communicate via nerve impulses.

Science & Technology and Environment by Ravi Garlapati Page 8


In addition to its favorable electronic properties, graphene is also flexible, allowing it
to be wrapped around delicate tissues. Therefore, option (d) is the correct answer.

10. The Indian Army recently proposed the use of vehicle-based drone jammers, which
can be achieved through various methods such as radio frequency jamming, satellite
link disruption, spoofing, and dazzling.
Spoofing is an attack on a computer system where the attacker tries to steal the
identity of a legitimate user and impersonate another person. Such an attack can
breach the system's security or steal users' information. Therefore, statement 1 is
true.
Phishing is a type of computer attack where the attacker tries to fraudulently obtain
users' sensitive information through electronic communication by impersonating a
trusted organization. Therefore, statement 2 is correct.
Spoofing can be part of phishing, but phishing cannot be part of spoofing. Spoofing
does not necessarily require fraud, but phishing is always operated in a fraudulent
manner.

11. DoS, or Denial-of-Service, is a type of attack that aims to make a website or network
unavailable to legitimate users by flooding it with traffic. A Distributed Denial of
Service (DDoS) attack is a type of DoS attack that uses multiple sources to overwhelm
an online service.
Smishing is similar to vishing, but instead of phone calls, it uses SMS text messages to
deceive the recipient into divulging sensitive information.
Vishing is a type of cyber attack that involves using a phone call or voicemail message
to trick the recipient into disclosing sensitive information, such as passwords, credit
card numbers, or OTPs.
Cryptojacking is a type of attack where a hacker uses a victim's computer or mobile
device to mine cryptocurrency without their knowledge or consent.

12. Standalone 5G and Non-Standalone 5G


Standalone 5G and Non-Standalone 5G are two main deployment modes for 5G
networks. The selection of the deployment mode depends on the operator's
perception of the market for the new technology, as both approaches have their
advantages and disadvantages.
In Standalone mode, the 5G network operates independently with dedicated
equipment and runs parallel to the existing 4G network. On the other hand, in Non-
Standalone mode, the 5G network utilizes the 4G core infrastructure. Since Non-
Standalone networks are built on existing infrastructure, the initial cost and the time
taken to deploy services through this mode are significantly less than Standalone
networks. However, the benefits of Standalone networks exceed those of Non-
Standalone networks.

Science & Technology and Environment by Ravi Garlapati Page 9


13. The Indian Farmers Fertilizer Cooperative (IFFCO) has developed the world’s first
Liquid Nano Urea, which is essentially urea in the form of a nanoparticle. It aims to
reduce the burden of urea subsidy, prevent the unbalanced and indiscriminate use of
conventional urea, increase crop productivity and reduce soil, water, and air
pollution. The IFFCO’s Nano Biotechnology Research Center (NBRC) developed this
technology.
Statement 1 is correct, as liquid nano urea sprayed directly on the leaves is better
absorbed by the plant, with an efficiency of 85-90%, compared to conventional urea
with an efficiency of about 25%.
Statement 2 is also correct, as liquid nano urea has a higher shelf life of up to a year
compared to conventional urea.
Statement 3 is incorrect, as there is no relation between using liquid nano urea and
significant water reduction for irrigation use. Liquid nano urea releases nitrogen,
which is a major component of chlorophyll.
Statement 4 is correct, as the imbalance in the use of conventional urea leads to most
of the nitrogen being vaporized or lost as a gas, resulting in air pollution. The use of
Nano Urea can help solve this problem by reducing the use of urea by up to 50%,
making it an eco-friendly product.

14. Net neutrality is the principle that internet traffic should be treated equally, without
discrimination or preference given to certain types of traffic or websites, and ISPs
should not block or slow down access to specific websites or services, or charge
higher fees for faster access to certain websites or services. This is based on the idea
that the internet is a public utility that should be accessible to everyone without
discrimination.
Statement 1 is correct: Net neutrality protects consumers' freedom to choose what
they want to access and use on the internet.
Statement 2 is correct: Net neutrality ensures access to information regardless of
location, allowing all internet users to have access to the same information,
regardless of their location or ISP resources.
Statement 3 is incorrect: Reduced cost of internet to consumers is not necessarily an
advantage of net neutrality. ISPs may charge higher fees for data connection, and the
cost of data transfer is passed on to the consumer. For instance, streaming services
accounted for 57% of global bandwidth consumption in 2018, and this additional
cost is borne by the consumer in the form of higher subscription fees.
Statement 4 is correct: Net neutrality promotes social inclusion by ensuring that all
users have equal access to the internet, irrespective of their ability to pay.

15. The Virtual Private Network (VPN) technology creates a secure, encrypted
connection over a public network, allowing users to access a private network
remotely. However, there are a few potential issues associated with using VPNs:

Science & Technology and Environment by Ravi Garlapati Page 10


1. Slower internet connection speeds due to the encryption and decryption of data,
especially on lower-end devices or when the VPN server is located far away.
Hence statement 1 is correct.
2. VPNs do not provide protection against all online threats, such as malware and
phishing. Hence statement 2 is correct.
3. Some VPNs can be expensive, which may not be feasible for some users,
especially for businesses. Hence statement 3 is correct.
4. VPN connections can be unstable and may drop unexpectedly, leading to the loss
of data or connection. Hence statement 4 is correct.
5. The statement that VPNs cannot bypass geographical restrictions is incorrect.
VPNs can be used for geo-spoofing to access content that is not available in a
certain location due to licensing agreements or government restrictions. Hence
statement 5 is incorrect.

16. Edge data centers are decentralized facilities that are located closer to end-users or
devices, providing faster processing, storage, and analysis of data, while reducing
latency and bandwidth requirements. They are particularly suited to support
internet of things (IoT) devices, real-time applications, and other applications
requiring low-latency and high-bandwidth connections. The growth potential of edge
data centers in India is significant.
Option a is incorrect because cloud data centers are data centers operated by cloud
service providers to deliver cloud-based services such as storage, computing, and
software to customers over the internet.
Option b is incorrect because colocation data centers provide space, power, and
cooling for multiple organizations to house their own server and networking
equipment.
Option d is incorrect because enterprise data centers are data centers owned and
operated by a single organization and used to support the organization's internal
operations and services.

17. Statement 1 is correct, but Statement 2 is incorrect. NFC, or Near Field


Communication, is a set of communication protocols that allow electronic devices to
establish radio communication with each other by bringing them into close
proximity, usually within 10 cm or less. An antenna can extend the range to 20 cm.
NFC uses electromagnetic radio fields to transmit data between two devices that
have NFC chips, and transactions occur within a short distance.
Statement 3 is correct: NFC often establishes a secure channel and employs
encryption when transmitting sensitive information.

18. The first statement is correct, as Li-Fi (Light Fidelity) utilizes the frequencies
produced by LED bulbs to transmit information wirelessly by flickering on and off. In

Science & Technology and Environment by Ravi Garlapati Page 11


Order on Amazon: https://amzn.eu/d/gNZielj
Purchase soft copy only for Rs. 60/- on Simplify Success App: https://tinyurl.com/2ffksz85
essence, Li-Fi is a technology for wireless communication that transmits data at an
incredibly high speed using visible light.
The second statement is also correct, as Li-Fi is known to offer data transmission
rates that are approximately 100 times faster than those provided by Wi-Fi, which
relies on radio waves to transfer data.

19. The ONDC project is a government-supported initiative that seeks to democratize e-


commerce by fostering an open network for trading goods and services over digital
networks. The platform is not platform-centric, meaning consumers can buy using
any platform. It is important to note that ONDC is not the regulatory body for the e-
commerce ecosystem in India. As a result, statements 2 and 3 are correct, while
statement 1 is incorrect.

20. IBM Watson Studio is a cloud-based platform that provides tools and services for
building, training, and deploying AI models. It offers a collaborative environment for
data scientists, developers, and subject matter experts to work together on projects.
DeepMind Technologies, an AI research company owned by Alphabet, has developed
AlphaFold, an AI tool that has collaborated with the European Bioinformatics
Institute to predict and publish over 200 million protein structures. The resulting
database is widely regarded as one of the most significant resources for biological
research.
ChatGPT is an AI language model trained by OpenAI. It is based on the GPT-3.5
architecture and has been designed to respond to natural language queries and
generate human-like text

Exercise – II a) 1 only b) 2 only


Topics: Space & Applications c) 2 and 3 only d) 1, 2 and 3

1. Which of the following statements 2. Which of the following statements


regarding the International Liquid about Navigation Systems is/are
Mirror Telescope (ILMT) is/are incorrect?
correct? 1. The Indian Regional Navigation
1. The ILMT is a space telescope Satellite System (IRNSS) is a
situated at Lagrangian Point 2, regional navigation system
roughly 1.5 million kilometers developed indigenously in India,
away from Earth. not a global navigation system.
2. Mercury is utilized as the reflective 2. The GPS Aided GEO Augmented
liquid in ILMT, which is Navigation (GAGAN) is a Satellite-
safeguarded against wind by a Based Augmentation System
thin, transparent film of mylar. (SBAS) launched by India to
3. The ILMT cannot be rotated or improve the accuracy and integrity
directed towards any particular of GPS signals for the safety of civil
direction.
Science & Technology and Environment by Ravi Garlapati Page 12
aviation, by augmenting the IRNSS 1. The Gaganyaan program will be
signal quality. India's first human-spaceflight
3. The ionosphere layer of the program by ISRO with a crew of
atmosphere can cause interference three astronauts to Lower Earth
with radio communication. Orbit (LEO) to demonstrate
indigenous capability.
a) 1 and 2 only b) 2 only 2. Rakesh Sharma is the first and only
c) 2 and 3 only d) 3 only Indian citizen to have travelled to
space.
3. Which of the following statements 3. The Vyomnauts of Gaganyaan
accurately describe Satellite-Based program will be launched by LMV
Internet Systems? III.
1. Satellite internet constellations Which of the statements given above
like Starlink offer lower latency is/are correct?
than fiber optic internet. a) 1 and 3 only b) 3 only
2. Internet services provided by c) 1 only d) 1, 2, and 3
satellite constellations can be
impacted by atmospheric factors 6. Consider the following pairs:
such as moisture and precipitation. Space Feature
3. 'Responsible Space' is an initiative Mission
launched by Starlink to promote
sustainability and safe operation of 1. Psyche It is a Chinese space mission
satellite systems in space to study solar flares.
a) 1 only b) 1 and 2 only
c) 2 only d) 1, 2 and 3 2. JUICE It is a European Space
Agency's mission to observe
4. Consider the following statements Jupiter.
about ISRO's Launch Vehicles:
1. LVM-III employs India's most 3. LUNA- It is a Russian space mission
powerful cryogenic engine. 25 to study Lunar surface.
2. SSLV uses a combination of solid
and liquid propellants. 4. LUCY It is NASA’s mission to study
3. PSLV uses India's first Indigenous Jupiter Trojan Asteroid.
Cryogenic engine.
Which of the statements given above
is/are correct? How many pairs given above are correctly
a) 1 only b) 2 and 3 only matched?
c) 1 and 2 only d) 3 only a) Only one pair
b) Only two pairs
5. Consider the following statements c) Only three pairs
regarding India's Space Program: d) All four pairs

Science & Technology and Environment by Ravi Garlapati Page 13


7. Consider the following statements c) It is an air defense system used to
about the Solid Fuel Ducted Ramjet counter hostile aerial attacks.
(SFDR) technology: d) It is a method for extracting iron
1. SFDR technology enables missiles from its ore.
to intercept aerial threats at
subsonic speeds only. 10. Consider the following statements:
2. The SFDR technology is a joint 1. The Defence Research and
venture between India and France. Development Organization
3. The SFDR technology uses (DRDO) has recently established
atmospheric air as an oxidizer. an Integrated Cryogenic Engine
Which of the statements given above Manufacturing Facility.
is/are correct? 2. India is the sole Asian country that
a) 1 and 3 only b) 3 only has conducted a successful test
c) 1 only d) 1, 2, and 3 flight for Cryogenic Engine.
3. Cryogenics is the scientific study
8. Which of the following statements that deals with the production and
about Thermobaric Weapons are consequences of extremely low
correct? temperatures.
1. Russia created Thermobaric Which of the statements given above
Weapons, a type of nuclear is/are correct?
weapons. a) 1 and 2 only b) 2 only
2. Oxygen obtained from the c) 3 only d) 1, 2 and 3
atmosphere is used to produce a
blast in Thermobaric Weapons. 11. Recently, the Defence Research and
3. All forms of Thermobaric Weapons Development Organization (DRDO)
are strictly prohibited by the claimed to have successfully
Hague Convention, including their demonstrated Quantum Key
use, production, and transfer. Distribution (QKD) technology.
a) 2 only b) 3 only Consider the following statements
c) 1 and 2 only d) 1, 2 and 3 regarding QKD:
1. QKD relies on the principles of
9. Which of the following options quantum mechanics to provide a
accurately describes the meaning of secure communication technique.
the term 'Iron Dome' as it is 2. It is believed that QKD has the
commonly used in news? ability to detect any intrusion or
a) It is an architectural design eavesdropping on the
intended to safeguard buildings from transmission network.
excessive heat. 3. One of the challenges with this
b) It is an agricultural structure technology is the current
designed to maintain an ideal limitation in achieving long-range
temperature for plant growth. transmission beyond 100
kilometers.

Science & Technology and Environment by Ravi Garlapati Page 14


Which of the statements given above
is/are correct? 14. Identify the correct statements about
a) 1 only b) 1 and 2 only Mission Prarambh:
c) 2 and 3 only d) 1, 2, and 3 1. It is India's first privately
developed space launch vehicle.
12. What are the activities that can be 2. The mission was authorized by
effectively carried out using Geo- Antrix, the commercial and
Spatial technology, given the current marketing arm of the Indian Space
level of technology? Research Organization.
1. Locating and tracking goods 3. Under this mission, the suborbital
2. Analyzing the distribution of launch vehicle Vikram-S has been
mineral resources launched.
3. Mapping weather forecasts to Choose the correct answer using the
specific regions code given below:
4. Assessing vegetation on specific a) 1 and 2 only b) 2 and 3 only
terrains c) 1 and 3 only d) 1, 2 and 3
5. Managing individual finances.
a) 1, 2 and 5 only 15. Consider the following statements
b) 2 and 4 only regarding the technological
c) 1, 2, 3 and 4 only achievements of the Mars Orbiter
d) 2, 3, 4 and 5 only Mission (MOM):
1. Machine learning models were
used by the mission to classify
13. Which of the following statements
extra-terrestrial landslides.
regarding Chandrayaan-3 mission are
correct? 2. The mission captured images of
1. Chandrayaan-3 is a mission that the far side of Mars' moon Deimos
follows the Chandrayaan-2 for the first time.
mission, and it aims to showcase 3. The MOM is credited with the
India's capability to perform a soft discovery of ‘supra-thermal’
landing on the Moon's surface. Argon-40 atoms in the Martian
2. Chandrayaan-3 will be launched exosphere.
using the LVM 3 launch vehicle. 4. The mission observed and
3. The Chandrayaan-3 mission captured the seasonal changes in
consists of an Orbiter, Lander, and the thickness of Martian polar ice
Rover, which will be used to caps.
explore the uncharted South Pole Which of the statements given above are
region of the Moon. correct?
4. Chandrayaan-3 is solely an Indian a) 1, 2, and 3 b) 1, 3, and 4
mission and not a joint c) 2 and 4 d) 1, 2, 3, and 4
collaboration with Japan.
a) 1 and 2 only b) 2 and 4 only 16. For the measurement/estimation of
c) 1, 2 and 3 only d) 3 and 4 only which of the following are satellite

Science & Technology and Environment by Ravi Garlapati Page 15


images/remote sensing data used? 19. What are Lagrangian points, often
(UPSC PRELIMS 2019) discussed in the context of space
1. Chlorophyll content in the exploration?
vegetation of a specific location a) They are points where a small
2. Greenhouse gas emissions from rice object is relatively stable under
paddies of a specific location the gravitational influence of two
3. Land surface temperatures of a larger objects.
specific location b) They are locations on the outer
Select the correct answer using the edges of the Milky Way galaxy.
code given below. c) They are points in the solar
a) 1 only b) 2 and 3 only system where the orbits of two
c) 3 only d) 1, 2 and 3 planets intersect.
d) They are locations on the surface
17. What are the possible applications of of Jupiter where frozen water has
ISRO’s Inflatable Aerodynamic been detected.
Decelerator (IAD)?
1. Recovery of spent stages of a 20. Which of the following entities is
rocket. responsible for the lunar orbiter
2. Landing payloads on Mars or mission called "Danuri"?
Venus. a) European Union b) Japan
3. Creating space habitats for human c) China d) South Korea
spaceflight missions.
Select the correct answer using the 21. Consider the following statements
codes given below: about Mars.
a) 1 and 2 only b) 2 and 3 only 1. The orbital speed of a planet
c) 1 and 3 only d) 1, 2 and 3 changes as its distance from the
sun changes.
2. The Mars Orbiter Mission (MOM)
18. Consider the following statements
is the first interplanetary mission
regarding the Magellanic Clouds:
by the Indian Space Research
1. These clouds can be seen with the
Organization (ISRO) to Mars. The
naked eye in the southern
orbiter was designed to operate in
hemisphere.
an elliptical orbit around Mars
2. They are a part of the Milky Way
ranging from 372 km to 80,000
galaxy.
km.
3. The Magellanic Clouds provide a
3. The density of the Martian
setting for researching active star
atmosphere is higher compared to
formation and evolution.
the Earth's atmosphere.
Which of the statements given above
Which of the above statements are
is/are correct?
correct?
a) 1 and 2 only b) 1 and 3 only
a) 1 only b) 1 and 2 only
c) 3 only d) 1, 2 and 3
c) 2 and 3 only d) 3 only

Science & Technology and Environment by Ravi Garlapati Page 16


22. Select the correct statement(s) about 2. Antrix Corporation and New Space
Scramjet Engines, Sounding Rockets, India Limited (NSIL) both operate
and BrahMos from the options below. under the administrative control
1. Scramjet engines enable efficient of the Department of Space (DoS).
operation at hypersonic speeds 3. ISRO has launched more foreign
and use supersonic combustion. satellites than Indian-origin
2. Sounding rockets developed by satellites.
ISRO are solid propellant rockets a) 1 only b) 2 only
used for upper atmospheric c) 1 and 3 d) 1, 2, and 3
research.
3. BrahMos is a supersonic cruise 24. Which of the following is not a
missile that uses a ramjet engine purpose of HAGAR, MACE, and
and can be launched from land, Astrosat observatories?
sea, and air. a) Observing very high energy
a) 1 only b) 1 and 2 only gamma-ray emissions from celestial
c) 2 and 3 only d) 3 only objects.
b) Studying dark matter and dark
23. Which of the following statements are energy.
true? c) Exploring high energy gamma
1. The Amazonia-1 mission is the radiation in the Universe.
first dedicated commercial mission d) Observing the universe in various
of New Space India Limited (NSIL). regions of the electromagnetic
spectrum.

Exercise – II Key

1. c 2. b 3. c 4. c 5. d 6. c 7. b 8. a 9. c 10. c
11. d 12. c 13. a 14. c 15. d 16. d 17. d 18. b 19. a 20. d
21. b 22. b 23. d 24. b

Explanations
1. The International Liquid Mirror Telescope (ILMT) is located at the Devasthal
Observatory campus of the Aryabhatta Research Institute of Observational Sciences
(ARIES) in Nainital, Uttarakhand, India, and it is not situated at the Lagrangian Point.
Therefore, Statement 1 is incorrect.
The ILMT consists of three main components: a rotating container with a highly
reflective liquid (mercury) that forms the primary mirror and is protected from wind
by a thin transparent film of mylar, an air compressor that operates an air bearing on
which the liquid mirror rests, and a drive system. Thus, Statement 2 is correct.
As liquid mirror telescopes cannot be tilted and track like conventional telescopes,
Statement 3 is also correct.

Science & Technology and Environment by Ravi Garlapati Page 17


2. NavIC (Navigation with Indian Constellation), also known as IRNSS (Indian Regional
Navigation Satellite System), is a regional satellite navigation system developed
indigenously by the Indian Space Research Organization (ISRO) for national
applications. Therefore, Statement 1 is correct.
GAGAN (GPS Aided GEO Augmented Navigation) is a Satellite-Based Augmentation
System (SBAS) launched by ISRO and the Airport Authority of India (AAI) to ensure
the safety of civilian aviation by augmenting the Global Positioning System (GPS)
signal quality. It does not augment the IRNSS signal quality, as IRNSS is a separate
regional navigation system. Hence, Statement 2 is incorrect.
The ionosphere is a layer of the atmosphere that can interfere with radio
communication, as it contains ionized gases that are affected by solar radiation. This
interference can impact radio signals, including those used in navigation systems like
GPS. Therefore, Statement 3 is correct.

3. Satellite internet is offered by companies such as SpaceX through constellations like


Starlink, which provide internet access to nearly 40 countries. However, the latency
offered by satellite internet is typically higher than that of cable and fiber internet
due to the satellites being positioned at a distance of 35,000 kms above the Earth
(although Starlink's satellites are placed in LEO at a distance of 300 miles). Cable and
fiber internet generally offer latency in the range of 20 to 50 milliseconds (ms), while
satellite internet latency can range as high as 600 ms, making Statement 1 incorrect.
Moisture and precipitation can interfere with internet services provided by satellite
constellations. Heavy snowfall, for example, can disrupt communication and the
accumulation of snow around the satellite dish may need to be removed to restore
service, so Statement 2 is correct.
'Responsible Space' is actually an initiative by OneWeb to promote sustainability and
safe operation of satellite systems in space. The goal is to prevent harm to the
environment of lower Earth orbit while developing connectivity and mobility.
Therefore, Statement 3 is incorrect.

4. The LVM-III (earlier known as GSLV Mk. III), an evolved version of India's
Geosynchronous Satellite Launch Vehicle, is a 3-stage launch vehicle capable of
carrying 4-ton class satellites into Geosynchronous Transfer Orbit (GTO) or about 10
tons to Low Earth Orbit (LEO). The vehicle comprises two solid strap-on motors
(S200), the largest solid boosters in the world, one liquid core stage (L110) with twin
liquid engines, and a high-thrust cryogenic upper stage (C25) with a fully indigenous
cryogenic engine. Therefore, statement 1 is correct.
The SSLV is a 3-stage launch vehicle that is specifically designed to launch satellites
to serve the emerging global market for launching small satellites. It comprises three
solid-fuel-based stages and a liquid fuel-based velocity trimming module (VTM) that
is used to place satellites in orbit. The SSLV uses solid and liquid propellants, but not
cryogenic propellants. Therefore, statement 2 is correct.
Science & Technology and Environment by Ravi Garlapati Page 18
The PSLV is a medium-lift launcher with a range up to various orbits, including the
GTO, LEO, and Polar Sun Synchronous Orbit. The PSLV is a 4-stage system that
comprises a combination of solid and liquid-fuelled rocket stages. GSLV Mk. III is the
first launch vehicle that uses the cryogenic engine, whereas the Polar Satellite Launch
Vehicle doesn't use cryogenic engines. Therefore, statement 3 is not correct.

5. Statement 1 is correct as the Gaganyaan program aims to demonstrate India's ability


to undertake a human space flight mission to LEO with a crew of three astronauts.
Two uncrewed and one human-crewed mission are approved as part of the program.
Statement 2 is also correct as Rakesh Sharma, an Indian citizen, was the first and only
Indian to travel to space in a Soviet rocket in April 1984. While other Indian-origin
persons have also travelled to space, including Kalpana Chawla and Sunitha Williams,
Rakesh Sharma was the first and only Indian citizen to do so.
Statement 3 is also correct as the human-rated LMV – III (earlier known as GSLV Mk-
III) is identified as the launch vehicle for the Gaganyaan mission.

6. Pair 1 is incorrect: Psyche is a NASA mission that will explore the metallic asteroid 16
Psyche, which is located in the asteroid belt between Mars and Jupiter. This asteroid
is thought to be the exposed core of a planet that was destroyed in the early solar
system. No spacecraft has ever visited a metallic asteroid like Psyche. The mission is
set to launch in August 2022.
Pair 2 is correct: The Jupiter Icy Moon Explorer (JUICE) is an interplanetary
spacecraft developed by the European Space Agency. Its primary mission is to study
the giant gas planet Jupiter and its three largest icy moons, Ganymede, Callisto, and
Europa. The mission is a collaboration between European countries, Japan, and the
United States.
Pair 3 is correct: The Luna-25 spacecraft, also known as Luna-Glob, is a Russian lunar
lander set to launch in October 2021. It is the first Russian lunar mission since the
Luna-24 mission in 1976. The primary goal of the mission is to study the Moon's
south pole and its environment, including its soil and atmosphere.
Pair 4 is correct: The Lucy mission is a NASA mission launched in October 2021. Its
goal is to study the Trojan asteroids, which are small bodies that orbit the Sun in two
loose groups ahead of and behind Jupiter. These asteroids are thought to be
remnants of the early solar system, and studying them could provide insights into the
formation and evolution of the solar system.

7. DRDO has recently conducted a successful test of the Solid Fuelled Ducted Ramjet
(SFDR) technology at the Integrated Test Range (ITR) in Chandipur.
Statement 1 is incorrect: SFDR based propulsion enables the missile to intercept
aerial threats at very long ranges at supersonic speeds, unlike subsonic speeds. The
development of SFDR technology will enable India to make its own long-range air-to-
air missile and surface to air missile.

Science & Technology and Environment by Ravi Garlapati Page 19


Statement 2 is incorrect: The Solid Fuel Ducted Ramjet (SFDR) has been developed
under a joint Indo-Russian R&D project, not with France.
Statement 3 is correct: The SFDR system uses a solid fuelled air-breathing propulsion
ramjet engine, which takes up oxygen from the atmosphere during flight. This makes
it lighter than solid-propellant rockets, which carry fuel and oxidiser in the rocket
itself and limits their range.

8. The first statement is incorrect: Thermobaric weapons are also referred to as fuel-air
bombs, aerosol bombs, or vacuum bombs, and they are regarded as the most
destructive non-nuclear weapons created to date.
The second statement is correct: Oxygen from the surrounding air is employed to
create a powerful and high-temperature blast in thermobaric weapons.
The third statement is incorrect: While no international regulations prohibit the
production, use, and transfer of all types of thermobaric bombs, if a nation employs
them to target civilians, it may be found guilty of a war crime under the Hague
Conventions.

9. The correct option is C, which states that the Iron Dome system is an air defense
system designed by Israel, which uses Tamir interceptor missiles and radar to detect
and eliminate incoming missiles or rockets. This system is capable of detecting and
engaging targets up to a range of 70 kilometers, and it is effective in all weather
conditions.

10. Statement 1 is incorrect: The Integrated Cryogenic Engine Manufacturing Facility


(ICMF) has been established by Hindustan Aeronautics Limited (HAL) in Bengaluru,
and it will cater to the entire rocket engine production under one roof for ISRO.
Statement 2 is incorrect: In 2014, India successfully flew GSLV-D5 with a cryogenic
engine and became the 6th country to develop cryogenic engines after the USA,
France, Japan, China, and Russia. Cryogenic engines are widely used in launch
vehicles around the world. India is not the only country in Asia to have successfully
conducted a test flight with a cryogenic engine.
Statement 3 is correct: Cryogenics is a scientific field that studies the production and
effects of very low temperatures. The term cryogenics generally refers to
temperatures below approximately -150 C.

11. Quantum Key Distribution (QKD) is a technique that uses the principles of quantum
mechanics to securely distribute cryptographic keys. The keys are transmitted
between two parties using a shared quantum channel, such as a fiber optic cable,
with the states of individual photons encoding the keys.
Statement 1 is correct: QKD uses the principles of quantum mechanics to provide a
secure communication technique.

Science & Technology and Environment by Ravi Garlapati Page 20


Statement 2 is correct: QKD offers a high level of security and has the potential to
detect any intrusion or eavesdropping on the transmission. Because of the unique
properties of photons, any third party who tries to read or copy the photons in any
way will change the photons' state.
Statement 3 is correct: One of the limitations of QKD is the distance over which the
photons can travel, which is typically around 100km due to signal loss over long
distances.
Heisenberg's Uncertainty Principle: This principle states that it is impossible to know
the exact velocity and position of a particle at the same time, and one can calculate
velocity or position to precision but not both at the same time

12. Geo-spatial technology is the use of technology to collect, analyze, and visualize data
related to a specific location. This can include maps, satellite imagery, and geographic
information systems (GIS) data, and is utilized in various fields, such as urban
planning, environmental management, military operations, and natural resource
management. Examples of geo-spatial technologies are:
 Option 1: Geo-spatial technology can be utilized to track the location and
movement of goods and to analyze data related to the quality of goods, such as
temperature or humidity data.
 Option 2: GIS and remote sensing are examples of geo-spatial technology that can
be used to map and analyze the distribution of mineral resources.
 Option 3: Geo-spatial technology can be used to visualize and analyze weather
data, such as through the use of GIS or by overlaying weather forecasts onto
maps.
 Option 4: Remote sensing or GIS can be utilized by geo-spatial technology to
assess the state of vegetation on a selected terrain.
 Option 5: Managing personal finances is not typically related to geo-spatial
technology, as it is not a direct use-case for this technology.

13. Statement 1 and 2 are correct. Chandrayaan-3 is a follow-on mission to Chandrayaan-


2 and it will be launched by LVM – III (earlier called GSLV Mk-III) from Sriharikota.
Statement 3 is incorrect. Unlike Chandrayaan-2, Chandrayaan-3 will only be carrying
a lunar lander and rover, not an orbiter, lander, and rover. It will communicate with
Earth with the help of the orbiter from Chandrayaan-2.
Statement 4 is also incorrect. Chandrayaan-3 is an indigenous mission by India, while
India and Japan are collaborating for a joint Lunar Polar Exploration Mission set to
launch around 2024-2025.

14. Statement 1 is correct: On November 18, 2022, Vikram-S, a suborbital launch vehicle
developed by Skyroot Aerospace Pvt. Ltd., Hyderabad, was successfully launched
under Mission Prarambh, making it the first privately developed space launch vehicle
in India.

Science & Technology and Environment by Ravi Garlapati Page 21


Statement 2 is incorrect: The mission was authorized by the Indian National Space
Promotion and Authorization Center (IN-SPACe) and not Antrix, which is the
commercial arm of the Indian Space Research Organization.
Statement 3 is correct: Vikram-S is a suborbital launch vehicle that carried three
customer payloads and validated the majority of the technologies in the Vikram
series of space launch vehicles.

15. In 2013, ISRO launched the Mars Orbiter Mission (MOM) or Mangalyaan, which has
been orbiting Mars since 2014, using the Polar Satellite Launch Vehicle (PSLV) rocket
C25. After successfully operating for 8 years, the spacecraft lost communication with
the ground station in October 2022. Despite this setback, the mission is widely
regarded as a major accomplishment in planetary exploration.
Statement 1 is correct: Using machine learning models, the mission provided an
opportunity to classify extra-terrestrial landslides.
Statement 2 is correct: The spacecraft captured, for the first time, the far side of
Deimos, one of the natural satellites of Mars.
Statement 3 is correct: The mission is credited with the discovery of ‘supra-thermal’
Argon-40 atoms in the Martian exosphere, which gave some clues on one of the
potential mechanisms for the escape of atmosphere from Mars.
Statement 4 is correct: The mission also observed the seasonal changes in thickness
of the Martian polar ice caps, i.e., the polar ice caps spread during Martian winter and
shrink during Martian summer.

16. Remote sensing is a technique that enables the collection of information about the
earth's surface and atmosphere from a distance, typically by using sensors on
satellites. The data obtained through remote sensing can be utilized for studying and
monitoring various environmental changes, including alterations in land use and
land cover, natural disasters, weather patterns, and climate change. All of the
applications mentioned in statements 1 to 3 can be accomplished using remote
sensing data.
India has developed several remote sensing satellites, such as Cartosat-1 and 2,
Resourcesat-1 and 2, Oceansat-1 and 2, Risat-1, Megha-Tropiques, SARAL, Scatsat,
and the INSAT series. These satellites are a part of the Indian Remote Sensing
Satellite (IRS) program.

17. Statement 1, 2, and 3 are correct. An Inflatable Aerodynamic Decelerator (IAD) was
developed by the Vikram Sarabhai Space Center and was successfully test flown in a
Rohini sounding rocket from TERLS, Thumba. This marks the first time that an IAD
was designed specifically for spent stage recovery, and all objectives of the mission
were successfully demonstrated. The IAD has potential in a variety of space
applications, including the recovery of spent rocket stages, landing payloads on Mars
or Venus, and creating space habitats for human spaceflight missions.

Science & Technology and Environment by Ravi Garlapati Page 22


18. NASA's James Webb telescope has recently discovered a star formation, called NGC
346, in a dynamic cluster within the Small Magellanic Cloud (SMC). The Magellanic
clouds are visible to the naked eye from the southern hemisphere, but cannot be
observed from most northern latitudes, making statement 1 correct.
The Milky Way galaxy has several satellite galaxies, with the largest being the Large
Magellanic Cloud, and these companion galaxies were named after Portuguese
navigator Ferdinand Magellan, whose crew discovered them during the first voyage
around the world in the 16th century. However, the Magellanic Clouds were
recognized early in the 20th century as companion objects to the Milky Way, but they
are not part of the Milky Way galaxy, making statement 2 incorrect.
The Magellanic Clouds serve as laboratories for the study of active stellar formation
and evolution, making statement 3 correct.

19. Lagrangian points refer to the points in space where the gravitational forces of two
larger bodies balance the centrifugal force of a smaller body. Therefore, statement (a)
is correct.
In 1772, the French mathematician Joseph-Louis Lagrange was the first to discover
the existence of five Lagrangian points in the Earth-Sun system.
Objects located at Lagrangian points remain relatively stable because the
gravitational forces of the two larger bodies balance the centrifugal force of the
smaller body. As a result, objects can maintain their position without expending
energy. Because of their stability, these points are useful in space exploration and
satellite missions.
In the Earth-Sun system, the five Lagrangian points are designated as L1, L2, L3, L4,
and L5.

20. Danuri is the first lunar orbiter mission of


South Korea, which is conducted in
collaboration with NASA. The primary
goal of this mission is to explore the
Moon's surface and to identify potential
landing sites for future missions.
The Korea Aerospace Research Institute
(KARI) is responsible for the design,
manufacturing, and operation of the
orbiter, while NASA assists in the
development of one of the six scientific payloads and provides support for spacecraft
communication and navigation. Therefore, the correct option is (d).

21. Kepler's Law of planetary motion states that a planet sweeps equal areas in equal
times, which means that planets move faster when they are closer to the Sun and
slower when they are farther away. Therefore, statement 1 is correct.

Science & Technology and Environment by Ravi Garlapati Page 23


The Mars Orbiter Mission (MOM) is ISRO's first interplanetary mission to Mars, and it
is designed to orbit Mars in an elliptical orbit with an apogee of 80,000 km and a
perigee of 372 km, which makes statement 2 correct.
Statement 3 is incorrect, as the Martian atmosphere has a much lower density than
Earth's atmosphere, although it exhibits weather phenomena due to various
chemical, thermodynamic, and fluid dynamic effects.

22. Statement 1 is correct, as scramjet engines are designed to operate efficiently at


hypersonic speeds and allow supersonic combustion, while ramjet engines use the
forward motion of the vehicle to compress incoming air for combustion without a
rotating compressor.
Statement 2 is correct, as ISRO-developed sounding rockets are used for upper
atmospheric research and are one or two-stage solid propellant rockets.
Statement 3 is correct, as BrahMos is a ramjet supersonic cruise missile that can be
launched from land, sea, and air.

23. Amazonia-1, an earth observation satellite of Brazil, was launched by India's PSLV
C51 in February 2021, marking the first dedicated commercial launch for NSIL. NSIL
is a wholly owned Government of India undertaking and commercial arm of ISRO,
under the administrative control of the Department of Space (DoS). Antrix
Corporation, also under the DoS, shares similar responsibilities of promoting and
commercializing ISRO's products and services. Hence, statements 1 and 2 are correct.
ISRO launched its first satellite, Rohini Technology Payload (RTP), using an Indian-
made Satellite launch vehicle (SLV-3 E1) in 1979. Since then, ISRO has launched 130
satellites of Indian origin and 422 foreign satellites belonging to 36 countries until
March 2023. Therefore, statement 3 is also correct.

24. High Altitude Gamma Ray experiment (HAGAR) and Major Atmospheric Cherenkov
Experiment Telescope (MACE) are ground-based telescopes that observe high-
energy gamma rays from celestial objects.
Astrosat is a space-based observatory that can observe the universe in various
electromagnetic spectrum regions, and recently detected UV light from a galaxy 9.3
billion light-years away. None of these observatories are designed to study dark
matter or dark energy.

Science & Technology and Environment by Ravi Garlapati Page 24


Exercise – III a) Biogas is a blend of gases,
Topic: Energy Resources predominantly composed of
methane and carbon dioxide.
1. Recently seen in news, Etalin b) Biogas combustion is smokeless
hydroelectric project is proposed to and does not leave any residue.
be set up in which state? c) Biogas has a higher calorific value
a) Arunachal Pradesh b) Assam than diesel.
c) Tripura d) Manipur d) Biogas can be utilized in diesel
engines.
2. Hydrogen has recently gained
attention as a potential alternative 5. Consider the following statements
source of energy. What are the about National Coal Index (NCI)
advantages of using hydrogen as fuel? 1. The National Coal Index (NCI) is
1. It can be produced from renewable used to track the changes in the
energy sources. usage of coal in India's energy mix.
2. It is more efficient than petrol as a 2. The financial year 1991-92 serves
fuel. as the base year for the National
Coal Index (NCI).
3. It can result in zero vehicular
emissions. Which of the statements given above
is/are correct?
Select the correct answer from the
options below: a) 1 only b) 2 only
a) 1 and 2 only b) 1 and 3 only c) Both 1 and 2 d) Neither 1 nor 2
c) 2 and 3 only d) 1, 2, and 3
6. The Prime Minister recently declared
that India has met its goal of blending
3. Gas hydrates are viewed as a potential
10% sugarcane-based ethanol with
source of energy, but their use raises
petrol before the scheduled time.
concerns. Which of the following
Considering this, which of the
statements are true?
following are the benefits of ethanol
1. Decomposition of gas hydrates can
blending?
contribute to global warming.
1. Decrease in petroleum imports
2. Most gas hydrates are composed of
methane, the release of which can 2. Increase in farmers' earnings
trigger submarine landslides. 3. Decrease in carbon monoxide
3. India has not yet discovered any emissions
gas hydrate deposits. Select the appropriate answer using
a) 2 and 3 only b) 1 and 2 only the given code:
c) 1 and 3 only d) 1, 2, and 3 a) Only 1 and 2 b) Only 2
c) 1, 2, and 3 d) Only 1 and
3
4. Which of the statements below is
incorrect about biogas?

Science & Technology and Environment by Ravi Garlapati Page 25


7. Consider the following statements c) All three statements.
about the Renewable Energy d) None of the statements.
Certificate (REC) Mechanism:
1. RECs are market-based 10. Regarding nuclear power plants,
instruments that certify ownership consider the following statements:
of one megawatt-hour (MWh) of 1. Normal water is utilized as a
electricity generated from a moderator in light water reactors.
renewable energy source. 2. Graphite serves as both a coolant
2. Once a REC has been sold by the and moderator in the Pressurized
REC registry in India, it cannot be Heavy water reactor.
purchased again. 3. Fast breeder Reactors do not
3. RECs are valid for about one year require moderators to function.
from the date of issuance. Which statements are correct?
Which of the above statements is/are a) 1 and 3 only b) 2 and 3 only
correct? c) 2 only d) 1, 2 and 3
a) 1 only b) 1 and 2 only
c) 1 and 3 only d) 1, 2 and 3 11. What are the fields in which nuclear
radiation is applied?
8. Which of the following statements 1. Agriculture
about Producer gas is/are correct? 2. Medical diagnostics
1. Burning coal or coke without the 3. Oil drilling
presence of oxygen generates 4. Cancer treatment
Producer Gas
5. Tracking the movement of
2. Producer gas is a colorless gas that pollutants
mainly consists of Methane and
Select the correct answer using the
hydrogen.
code given below.
3. Mostly used as fuel in the iron and
a) 1 and 3 only
steel manufacturing industries.
b) 3 and 4 only
a) 1 and 2 only b) 2 and 3 only
c) 1, 4 and 5 only
c) 3 only d) 1, 2 and 3
d) 1, 2, 3, 4 and 5
9. How many of the following statements
12. How does nuclear energy pose risks
about Liquid Petroleum Gas (LPG) and
and challenges?
Compressed Natural Gas (CNG) are
1. Generation of hazardous
correct?
radioactive waste.
1. LPG is an odorless and colorless
gas mainly consisting of propane 2. Possibility of a major nuclear
and butane. accident leading to the release of
radioactive materials into the air.
2. LPG is lighter than CNG.
3. Limitations in reprocessing used
3. CNG is heavier than air.
nuclear fuel.
a) Only one statement.
4. Challenges in the storage and
b) Only two statements.
disposal of used nuclear fuel.
Science & Technology and Environment by Ravi Garlapati Page 26
Select the correct answer using the a) Only One pair
code given below: b) Only Two pairs
a) 1, 2 and 4 only c) All Three pairs
b) 2 and 4 only d) None of the pairs
c) 1, 2 and 3 only
d) 1, 2, 3 and 4 15. Which of the following statements is
correct regarding Zinc-Air batteries?
13. Consider the following statements 1. Zinc-air batteries are rechargeable
about Tokamak: and are meant to be used many
1. It is a device that employs times before disposal.
magnetic fields to confine plasma 2. Zinc-air batteries require a
in a torus shape. continuous supply of oxygen from
2. Conditions similar to those in external sources to sustain
Tokamak can occur naturally in chemical reactions within them.
stars. 3. Zinc-air batteries are used in small
3. India constructed its first devices such as hearing aids and
indigenous Tokamak reactor in the wristwatches.
1980s. 4. Zinc-air batteries are generally less
Which of the statements given above expensive than lithium-ion
are correct? batteries.
a) 1 and 2 only b) 2 and 3 only Choose the correct answer from
c) 1 and 3 only d) 1, 2 and 3 options below.
a) 1 and 3 only b) 2 and 4 only
14. Consider the following pairs regarding c) 3 and 4 only d) 2, 3 and 4 only
Ethanol production:
Technological Raw material 16. In reference to Flex Fuel Vehicles, how
level used many of the following statements
is/are correct?
1. First Edible food crops 1. They can run on pure ethanol.
Generation 2. Flex Fuel vehicles running on 20%
ethanol ethanol blended petrol will not
eliminate carbon monoxide
2. Second Algae grown in emissions.
Generation wastewater 3. Increased use of Flex Fuel Vehicles
ethanol can create additional revenue for
farmers.
3. Third Genetically
Generation a) Only one statement
engineered
ethanol b) Only two statements
feedstock
c) All three statements
How many of the pairs given above are d) None of the statements
correctly matched?

Science & Technology and Environment by Ravi Garlapati Page 27


17. Which of the following statements rare earth elements, including
about Small Modular Reactors (SMRs) thorium, in India.
is correct with respect to nuclear Which of the statements given above
reactors? is/are correct?
a) SMRs only use heavy water as a a) 1 only b) 2 only
coolant, unlike standard reactors. c) Both 1 and 2 d) Neither 1 nor 2
b) They are typically cheaper and
faster to build than standard reactors. 19. Which of the following statements
c) SMRs are not vulnerable to damage about Nuclear Reactors in India is/are
caused by seismic activity, unlike correct?
standard reactors. 1. The Madras Atomic Power Station
d) SMRs cannot be used to enrich (MAPS) located at Kalpakkam was
uranium for the production of nuclear the first nuclear power station to
weapons. be built in India.
2. Boron is a material used to
18. Consider the following statements: maintain a controlled chain
1. Thorium can be used as nuclear reaction in nuclear reactors.
fuel without requiring any 3. Nuclear poison refers to a
processing or enrichment, unlike substance that can absorb
uranium.
neutrons near the reactor core.
2. Kerala holds the largest deposits (a) 1 only (b) 3 only
of monazite, a mineral that
(c) 2 and 3 only (d) 1, 2 and
contains significant amounts of

Exercise – III Key

1. a 2. d 3. b 4. c 5. d 6. c 7. b 8. c 9. a 10. a
11. d 12. a 13. d 14. a 15. c 16. b 17. b 18. d 19. c

Explanations
1. A proposed hydroelectric project, called Etalin, is planned to be established in
Arunachal Pradesh. The project, which has a capacity of 3,097 MW, would involve
clearing more than 1100 hectares of forested land and cutting down over 280,000
trees. Conservationists and wildlife scientists from the state have written to the Forest
Advisory Committee (FAC) of the Union Ministry of Environment, Forest and Climate
Change (MoEF&CC) expressing their concerns regarding the potential risks to the
local biodiversity posed by the project.

2. Statement 1 is correct. Hydrogen can be produced from renewable sources, and it is


classified into three types: grey, blue, and green. Grey hydrogen is produced from
fossil fuels and is the primary type currently generated. Blue hydrogen is derived
from fossil fuels but with carbon capture and storage. Green hydrogen, which comes
entirely from renewable sources, offers specific benefits. It is a clean-burning fuel that
Science & Technology and Environment by Ravi Garlapati Page 28
can decarbonize various sectors, including transportation, iron and steel, and
chemicals. Additionally, renewable energy that cannot be stored or used by the grid
can be channeled to produce hydrogen.
Statement 2 is correct. Hydrogen is two to three times more efficient than burning
petrol. One kilogram of hydrogen gas holds about the same amount of energy as 2.8
kilograms of petrol.
Statement 3 is correct. Hydrogen is a clean fuel that produces only water when
burned in air, releasing no carbon-based emissions. As a result, using hydrogen can
lead to zero vehicular emissions.

3. Gas hydrates are a potential energy source, but their use raises concerns. Statement 1
is correct in that their decomposition can release methane, a greenhouse gas that
contributes to global warming. Statement 2 is also correct in that gas hydrates are ice-
like crystalline minerals that mostly form from methane and their sudden release can
cause submarine landslides and trigger tsunamis. However, statement 3 is incorrect.
Recent research found that methane hydrate deposits of biogenic origin are located in
the Krishna-Godavari basin, which is a rich source of natural gas.

4. Biogas is produced by the process of Anaerobic Digestion in Biogas Plants using


organic waste such as biomass from farms, gardens, kitchens, industry, and municipal
waste. It's a mixture of gases, mainly methane, carbon dioxide, and traces of other
gases. Biogas burns cleanly and without leaving any residue. It can be used as a fuel
for cooking, lighting, generating electricity, and in diesel engines. The calorific value of
biogas is lower than diesel fuel, making option (c) incorrect.

5. The NCI is a price index that tracks the change in the price of coal each month relative
to the fixed base year of FY 2017-18. Hence both statements 1 and 2 are incorrect.
Recent Context: The Ministry of Coal and Mines organized an Investors' conclave to
discuss commercial coal mines auctions and opportunities in the mining sector. The
Ministry of Coal has launched commercial coal mines auctions on a revenue share
basis, using the National Coal Index (NCI) to determine the revenue share based on
market prices.
The Indian Statistical Institute, Kolkata, developed the concept, design, and
Representative Prices of the Index, which is released every month.

6. Prime Minister Narendra Modi announced that India has achieved its target of
blending 10% sugarcane-extracted ethanol in petrol ahead of schedule. Originally,
India aimed to increase this ratio to 20% by 2030, but in 2021, the deadline was
advanced to 2025 according to the ethanol roadmap by NITI Aayog.
Benefits of ethanol blending include:
 Reducing India's share of oil imports (which is almost 85%) and saving precious
foreign exchange.

Science & Technology and Environment by Ravi Garlapati Page 29


 Increasing farmers' incomes by boosting ethanol output. The Ministry of
Petroleum and Natural Gas reports that ethanol blending led to the payment of
over Rs 40,600 crore to farmers in the last eight years.
 Lowering the emissions of some regulated pollutants such as carbon monoxide
(CO), hydrocarbons (HC), and CO2. Therefore, the correct answer is option (c).

7. Renewable Energy Certificates (RECs) certify that the bearer owns one MWh of
electricity generated from a renewable energy source, and are used to address the
mismatch between renewable energy availability and the obligation of entities to
meet their Renewable Purchase Obligations (RPO). Therefore, statement 1 is correct.
The REC received can be sold on the open market as an energy commodity or as a
carbon credit to offset emissions. The Central Electricity Regulatory Commission
(CERC) has notified the regulation on RECs to promote renewable energy sources and
develop the electricity market.
A REC that has already been sold cannot be purchased again, and all RECs have a
unique number with location and generation information. Therefore statement 2 is
correct
RECs are valid for approximately three years from the date of issuance and their price
is determined in power exchange within the forbearance and floor price set by CERC.
Therefore statement 3 is not correct.

8. The gas is not produced without the presence of oxygen; it requires a controlled
amount of oxygen to be produced. Therefore, statement 1 is also incorrect.
Producer gas is a type of fuel gas that is produced by burning coal or coke with limited
oxygen and controlled moisture. The gas comprises flammable gases such as carbon
monoxide and hydrogen, as well as non-flammable gases like nitrogen and carbon
dioxide. Methane is not present in the gas. Hence, statement 2 is incorrect.
Producer gas is primarily used as a fuel in the iron and steel manufacturing industries,
such as in firing coke ovens and blast furnaces, and is used as a reducing agent in
metallurgical operations. Thus, statement 3 is correct.

9. LPG, a by-product of crude petroleum extraction, is a liquefied gas consisting of


propane, butane, and traces of propylene and butylene. While it is odorless and
colorless, an odorant is added to it for safety purposes. Statement 1 is correct.
CNG, or Compressed Natural Gas, is primarily methane compressed at a pressure of
200 to 248 bars. It is a cleaner fuel for transportation vehicles and contains methane.
In terms of density, LPG is heavier than CNG, so Statement 2 is not correct.
CNG is lighter than air because its density is lower than that of air, causing it to rapidly
dissipate into the atmosphere when released. Statement 3 is not correct.

Science & Technology and Environment by Ravi Garlapati Page 30


10. Light water reactors employ light water to generate electricity from nuclear energy,
as it can act as both a moderator and a coolant to remove the energy produced by
nuclear fission. Therefore, Statement 1 is true.
In a pressurized heavy water reactor, heavy water (D2O; D is deuterium an isotope of
Hydrogen) is employed as a coolant and moderator. Graphite serves as a moderator
because it reflects fast-moving neutrons, but it is not employed as a coolant in the
Pressurized Heavy water reactor. Therefore, Statement 2 is incorrect.
In nuclear reactors, moderators are used to reduce the velocity of the neutrons. Fast
neutrons produced during fission have high energy and travel quickly. These neutrons
are not as effective at causing fission as slower-moving ones. However, unlike most
conventional nuclear reactors, fast reactors do not need an effective moderator, so
their neutrons remain high-energetic. Although these fast neutrons are not as efficient
at causing fission, they are readily absorbed by a uranium isotope (U238), which then
transforms into plutonium (Pu239). This plutonium isotope can be reprocessed and
utilized as more reactor fuel or in the production of nuclear weapons. As a result, the
Fast Breeder Reactor can function without moderators. Hence, Statement 3 is correct.

11. Nuclear applications in agriculture rely on the use of isotopes and radiation
techniques to combat pests and diseases, increase crop production, protect land and
water resources. Hence option 1 is correct.
Common uses of nuclear medicine for diagnosis include, scans of the heart, lungs,
kidneys, gallbladder, and thyroid Hence option 2 is correct.
The use of radioactive materials in oil drilling can provide valuable information
about the subsurface properties of oil reservoirs and help to improve production
efficiency and safety. Hence option 3 is correct.
Nuclear medicine therapy uses a small amount of radioactive material combined with
a carrier molecule. Nuclear medicine therapies treat cancer and other conditions.
Hence option 4 is correct.
Radioactive isotopes can greatly be utilized for the accurate measurement of
pollutants in the environment including groundwater and river. Hence option 5 is
correct.

12. Statement 1 is correct: The generation of radioactive waste is a significant


environmental concern associated with nuclear power. This waste can include
materials such as uranium mill tailings, spent reactor fuel, and other radioactive
wastes that can remain hazardous to human health for thousands of years.
Statement 2 is correct: Although nuclear accidents are rare, when they occur, they can
have severe consequences for both people and the environment. Examples of major
nuclear accidents include the Chernobyl disaster in 1986 and the Fukushima nuclear
disaster in 2011.
Statement 3 is incorrect: Used nuclear fuel is typically not reprocessed to extract
fissile materials in most countries, although it has been done in some countries like

Science & Technology and Environment by Ravi Garlapati Page 31


France. Reprocessing can reduce the volume of high-level waste, but it also generates
additional waste and has proliferation risks.
Statement 4 is correct: The safe storage and disposal of used nuclear fuel is a
significant challenge for nuclear energy. The high level of radioactivity in used fuel
means that it must be carefully stored and monitored for thousands of years to
prevent harm to people and the environment.

13. A tokamak is a machine that uses a strong magnetic field to contain plasma in the
form of a torus. This is an important technology being developed for producing
controlled thermonuclear fusion power. Statement 1 is correct.
Statement 2 is also true, as plasma confinement is the process of keeping a plasma
under control by different forces, which is crucial for achieving fusion reactions.
These conditions occur naturally in stars, where gravity maintains them.
Statement 3 is correct as well. India's first Tokamak reactor, called Aditya, was
constructed in 1980 and completed 30 years of safe operation in January 2020.

14. The correct matching is as given below.


Technological level Raw material used
1. First Generation ethanol Edible food crops
2. Second Generation ethanol Food waste
3. Third Generation ethanol Algae grown in wastewater
4. Fourth Generation ethanol Genetically engineered feedstock

The 4 generations of ethanol production are:


1. First Generation: The first generation of ethanol production is based on the
fermentation of simple sugars derived from food based feedstocks such as corn,
wheat, and sugarcane.
2. Second Generation: The second generation of ethanol production is based on the
use of non-food feedstocks such as wood, grasses, and agricultural waste.
3. Third Generation: The third generation of ethanol production involves the use of
algae as a feedstock for ethanol production. Algae can be grown in a variety of
conditions and can produce high yields of lipids and carbohydrates, which can be
used to produce ethanol.
4. Fourth Generation: The fourth generation of ethanol production involves the use
of genetically modified organisms (GMOs) to produce ethanol. This generation is
still in development and not yet commercially viable.

15. Statement 1 is incorrect: Zinc-air batteries are not disposable batteries, they can be
rechargeable. They are batteries that use zinc and oxygen to create a reaction that
produces electricity. These batteries are known for their high energy density, which
means they can store a lot of energy in a small space.

Science & Technology and Environment by Ravi Garlapati Page 32


Statement 2 is incorrect: Zinc-air batteries do not require an external source of
oxygen to sustain the chemical reactions within them. They rely on the oxygen in the
air to produce the chemical reaction.
Statement 3 is correct: Zinc-air batteries are available in various sizes, from small
batteries used in hearing aids and watches to larger batteries used in electric vehicles
and grid energy storage. Zinc-air batteries are now used instead of mercury batteries
in small devices due to environmental concerns associated with mercury batteries.
Statement 4 is correct: Zinc-air batteries are generally cheaper than lithium-ion
batteries.

16. Statement 1 is correct: Flex engines have the capability of running on either 100%
petrol or 100% bio ethanol. A flex fuel or flexible fuel can operate on more than one
fuel type or even a blend of fuels, with the most common versions using a mix of
petrol and ethanol or methanol.
Statement 2 is incorrect: Flex fuel vehicles operating on a 20% ethanol blended petrol
may not completely eliminate carbon monoxide emissions. However, they can reduce
carbon monoxide emissions by 50% in two-wheelers and 30% lower in four-wheelers
than petrol. Additionally, hydrocarbon emissions can be reduced by 20% compared to
petrol.
Statement 3 is correct: The increased use of Flex Fuel Vehicles could lead to a wider
adoption of ethanol or methanol as a fuel source. This could result in an additional
source of revenue for farmers.

17. Option a is incorrect as Pressurized Heavy Water Reactors use heavy water
(deuterium oxide D2O) as both a coolant and moderator, and not just as a coolant.
However, some SMRs use different coolants such as sodium, lead, gas, or salt instead
of water.
Option b is correct as Small Modular Reactors (SMRs) are designed to be cheaper and
quicker to build and commission than traditional nuclear plants, with the
International Atomic Energy Agency (IAEA) defining them as nuclear power stations
producing under 300 megawatts electrical (MWe) of nuclear energy.
Option c is incorrect as SMRs, like any other nuclear reactors, can be susceptible to
seismic activity, including earthquakes and tsunamis.
Option d is incorrect as SMRs are not used to enrich uranium to produce nuclear
weapons, but rather to generate electricity. However, some experts have raised
concerns about the potential misuse of SMRs for nuclear weapons or other malicious
purposes if they fall into the wrong hands.

18. The use of uranium and thorium in nuclear reactors varies due to their different
characteristics. Unlike uranium, thorium cannot be directly used as nuclear fuel
because it does not contain enough fissile material to start a nuclear chain reaction.
Therefore, statement 1 is incorrect.

Science & Technology and Environment by Ravi Garlapati Page 33


India's major resource of rare earth minerals is monazite, and the Atomic Minerals
Directorate for Exploration and Research (AMD) has found 12.73 million tonnes of
monazite reserves in India. However, contrary to statement 2, Kerala does not have
the highest monazite resources in India. The largest monazite resource in India, with
3.78 million tonnes of monazite, is found in Andhra Pradesh.

19. India's first nuclear reactor was Apsara, built at Tarapur, Maharashtra in 1956.
Madras Atomic Power Station (MAPS) was designed and built by India on its own.
Thus, Statement 1 is incorrect.
Control rods, made of neutron-absorbing materials such as boron, are used to control
the chain reaction of nuclear fission in nuclear reactors. Therefore, Statement 2 is
correct.
A neutron-absorbing material, also known as a nuclear or neutron poison, is
intentionally inserted into certain types of reactors to decrease the reactivity of their
initial fresh fuel load. Thus, Statement 3 is correct.

Exercise - IV 1. It is a form of macronutrient


Topics: Biotechnology & Crop Science required by plants.
2. It is essential for growth
1. HD-3385, developed by ICAR, is a new hormone production and
variety of which species? internode elongation in plants.
a) Rice b) Wheat Which of the statements given above
c) Tomato d) Brinjal is/are correct?
(a) 1 only (b) 2 only
2. Consider the following pairs: (c) Both 1 and 2 (d) Neither 1
Plant cell organelles: Functions nor 2
1. Mitochondria: Generates energy
2. Ribosome: Distributes 4. Which of the following are related to
synthesized material the process of photosynthesis?
3. Lysosome: Waste disposal system 1. Absorption of light energy by
4. Golgi apparatus: Synthesizes chlorophyll.
biological proteins 2. Conversion of light energy to
Which of the pairs given above is/are chemical energy and splitting of
correctly matched? water molecules into hydrogen
(a) 1, 2, 3 and 4 and oxygen.
(b) 1 and 3 only 3. Reduction of carbon dioxide to
carbohydrates.
(c) 2, 3 and 4 only
Select the correct answer using the
(d) 4 only
code given below.
(a) 1 and 2 only (b) 2 and 3 only
3. With reference to Zinc, consider the
(c) 1 and 3 only (d) 1, 2 and 3
following statements:

Science & Technology and Environment by Ravi Garlapati Page 34


5. Consider the following pairs:
Mode of reproduction: Example 9. Which of the following statements
1. Vegetative propagation: Ginger about stem cells is/are true?
2. Pollination: Tomato 1. Stem cells are special cells that can
Which of the pairs given above are self-replicate and differentiate into
correctly matched? various types of cells.
(a) 1 only (b) 2 only 2. HIV patients can be completely
(c) 1 and 2 (d) None cured by stem cells obtained from
umbilical cord blood transplants.
6. Consider the following pairs: Choose the correction option from
Mode of reproduction : Example below.
1. Fragmentation : Yeast (a) 1 and 2 (b) 2 only
2. Budding : Jellyfish (c) 1 only (d) None
Which of the pairs given above are
correctly matched? 10. Which of the following statements
(a) 1 only (b) 2 only about Polymerase Chain Reaction
(c) 1 and 2 (d) None (PCR) technique is/are incorrect?
1. PCR helps in making many copies
of a small amount of DNA.
7. With reference to plant transportation
2. DNA molecule is first cooled to
system, consider the following
break it up into two strands
statements:
followed by heating with primers
1. Phloem moves water and minerals
so that they bind to the DNA.
obtained from the soil to the leaves.
3. RT-PCR diagnosis used for COVID-
2. Xylem transports products of
19 employs NAAT method.
photosynthesis from the leaves to
other parts of the plants. (a) 1 and 2 only (b) 2 only
3. Transpiration involves loss of water (c) 2 and 3 only (d) 1, 2 and 3
vapour through the stomata of
plants. 11. Researchers have recently developed
Which of the statements given above Proton therapy, a more efficient
is/are not correct? cancer treatment method.
(a) 1 and 2 only (b) 2 and 3 only 1. Proton therapy uses x-rays for
treating cancer.
(c) 1 and 3 only (d) 3 only
2. Cyclotron generates high-power
energy radiation for proton
8. Which of the following plant
therapy.
hormones acts as a general plant
3. Proton therapy is less likely to
growth inhibitor and an inhibitor of
damage healthy tissues
plant metabolism?
surrounding malignant tumors.
(a) Abscisic acid
Which of the above statements is/are
(b) Cytokinins
correct?
(c) Gibberellins
(a) 1 and 3 only (b) 1 and 2 only
(d) Auxin

Science & Technology and Environment by Ravi Garlapati Page 35


(c) 2 and 3 only (d) 1, 2, and 3
15. Consider the following statements
12. Consider the statements regarding regarding the stem cell transplant:
Ribose Nucleic Acid (RNA): 1. It is also known as bone marrow
1. RNA is present in the transplant.
mitochondria and nucleus of a 2. In allogeneic transplantation the
human cell. patient’s own stem cell is used.
2. Transfer RNA (t-RNA) carries Which of the statements given above
information from DNA to is/are correct?
ribosomes for protein synthesis. (a) 1 only (b) 2 only
Which statement(s) is/are incorrect? (c) Both 1 and 2 (d) Neither 1
(a) 1 only (b) 2 only nor 2
(c) Both 1 and 2 (d) Neither 1
nor 2 16. 'RNA interference (RNAi)' technology
has gained popularity in the last few
13. Which of the following statements years. Why? (UPSC PRELIMS 2019)
about gene banks is/are correct? 1. It is used in developing gene
1. Gene banks are biorepositories silencing therapies.
that preserve genetic material. 2. It can be used in developing
2. Gene bank is a method of in-situ therapies for the treatment of cancer.
conservation of flora and fauna. 3. It can be used to develop hormone
3. The world's second-largest gene replacement therapies.
bank is located in India. 4. It can be used to produce crop
Select the correct answer using the plants that are resistant to viral
code given below: pathogens.
(a) 1 and 3 only (b) 1 and 2 only Select the correct answer using the
(c) 2 and 3 only (d) 1, 2, and 3 code given below.
(a) 1, 2 and 4 (b) 2 and 3
14. With reference to Green Manure, (c) 1 and 3 (d) 1 and 4 only
consider the following statements :
1. These are crops grown specifically 17. Consider the following pairs about
for maintaining soil fertility and various cloning techniques and their
structure. applications:
2. Glyricidia, Pongamia and Subabul Cloning Potential
are used as Green Manure. Techniques Applications
3. Green manure usage will
subsequently reduce the consumption 1. Gene Regenerating a
of Chemical fertilizers in India. Cloning whole new organ
Which of the statements given above from embryonic
is/are correct ? stem cell
(a) 1 and 2 only (b) 2 and 3 only
(c) 1 and 3 only (d) 1, 2 and 3 only 2. Reproducing copy of
Science & Technology and Environment by Ravi Garlapati Page 36
Reproductive a whole animal
Cloning 20. Consider the following statements
about Genome Edited Plants (GEPs):
3. Treating the 1. CRISPR-Cas9 technique is often
Therapeutic neurodegenerative used for genome editing.
Cloning diseases 2. Bt-Cotton is India’s first non-
transgenic Genome Edited Crop.
Which of the statements given above
How many pairs above are correctly is/are correct?
matched? (a) 1 only (b) 2 only
a) Only one pair (c) 1 and 2 (d) None
b) Only two pairs
c) All three pairs 21. Which of the following are types of
d) None of the pairs bio- plastics?
1. Polyhydroxyalkanoate
18. The term ‘Genomic Surveillance’ can 2. Polylactic acid
be best described as a process of 3. Polyethylene terephthalate
constant monitoring of the: Select the correct answer using the
(a) Harmful mutations that occur due code given below.
to exposure to mutagens. (a) 1 and 2 only (b) 2 and 3 only
(b) Pathogens and analysing their (c) 1 and 3 only (d) 1, 2 and 3
genetic similarities and differences.
(c) Gene pool of a human population 22. Consider the following:
and assessing genetic diversity. 1. Bacteria
(d) Genetic markers to link an 2. Fungi
inherited disease with the responsible 3. Virus
gene. 4. Protozoa
Which of the above can be used in
19. Consider the following statements pesticides to prevent crop damage?
about Genome Edited Plants (GEPs): (a) 1 only (b) 2 and 4 only
1. All the gene edited crops are (c) 1, 3 and 4 only (d) 1, 2, 3 and 4
developed using insertion of foreign
genetic material.
23. Which of the following statements
2. It is mandatory for researchers in regarding fortification of food is/are
India to seek approval from the correct?
Genetic Engineering Appraisal 1. The Food Safety and Standards
Committee (GEAC) to modify a Authority of India provides
genome using gene editing. fortification guidelines.
Which of the statements given above 2. According to FSSAI rules, packaged
is/are correct? milk must be fortified with Vitamin
(a) 1 only (b) 2 only A and D.
(c) 1 and 2 (d) None

Science & Technology and Environment by Ravi Garlapati Page 37


3. Golden Rice, produced through 1. It is an autonomous statutory body
genetic fortification, requires which regulates food safety and
significant changes in cultivation standards.
practices. 2. It is under the Ministry of Health &
(a) 1 and 2 only (b) 2 and 3 only Family Welfare.
(c) 1 and 3 only (d) 1, 2, and 3 Which of the statements given above
is/are correct?
24. With reference to the Food Safety and (a) 1 only (b) 2 only
Standards Authority of India (FSSAI), (b) Both 1 and 2 (d) Neither 1 nor 2
consider the following statements:

Exercise – IV Key
1. b 2. b 3. b 4. d 5. c 6. b 7. a 8. a 9. a 10. b
11. c 12. d 13. a 14. c 15. a 16. a 17. b 18. b 19. d 20. a
21. a 22. d 23. a 24. c

Explanations
1. The new wheat variety called HD-3385 has been developed by the Indian Council of
Agricultural Research (ICAR) to address the challenges posed by climate change and
rising heat levels. It can be sown early and harvested before the end of March.

2. Mitochondria are known as the powerhouse of the cell.


The energy required for various chemical activities needed for life is released by
mitochondria in the form of ATP (Adenosine Triphosphate) molecules [If
Mitochondria is the Power Plant. ATP is the Electricity].
Lysosomes are a kind of waste disposal system of the cell.
Lysosomes help to keep the cell clean by digesting any foreign material as well as
worn-out cell organelles.
During the disturbance in cellular metabolism, for example, when the cell gets
damaged, lysosomes may burst and the enzymes in them digest their own cell.
Therefore, lysosomes are also known as the ‘suicide bags’ of a cell.
Golgi apparatus is a special part of our cells that helps process and package
molecules so that they can be sent to the right place to do their jobs.
A ribosome is a complex molecular machine found inside the living cells that produce
proteins from amino acids during a process called protein synthesis or translation.
Hence, option (b) is the correct answer.

3. Plant growth and development are largely determined by nutrient availability. Plants
require two types of nutrients- macronutrients and micronutrients.
Macronutrients are the nutrients required by plants in large amounts. These include
carbon, hydrogen, oxygen, nitrogen, phosphorus, sulfur, calcium, and potassium.
Micronutrients are the ones required in very small amounts by plants. These include

Science & Technology and Environment by Ravi Garlapati Page 38


iron, zinc, boron, iron, manganese, molybdenum, etc. Hence, statement 1 is not
correct.
Zinc is also involved in the activation of certain enzymes and is essential for growth
hormone production and internode elongation in plants. Hence, statement 2 is
correct.

4. Photosynthesis is the process by which autotrophs (those that can produce their own
food) take in substances from the outside and convert them into stored forms of
energy. This material is taken in the form of carbon dioxide and water which is
converted into carbohydrates in the presence of sunlight and chlorophyll.
Carbohydrates are utilised for providing energy to the plant.
The following events occur during this process of photosynthesis:
 Absorption of light energy by chlorophyll.
 Conversion of light energy to chemical energy and splitting of water molecules
into hydrogen and oxygen.
 Reduction of carbon dioxide to carbohydrates. Hence option (d) is the correct
answer.

5. There are several ways by which plants produce their offspring. These are
categorised into two types: (i) asexual, and (ii) sexual reproduction. In asexual
reproduction plants can give rise to new plants without seeds, whereas in sexual
reproduction, new plants are obtained from seeds.
Vegetative propagation
 It is a type of asexual reproduction in which new plants are produced from
roots, stems, leaves and buds.
 Since reproduction is through the vegetative parts of the plant, it is known as
vegetative propagation.
 For example- potato, Ginger, Sweet potato. Hence pair 1 is correctly matched.
Pollination
 Pollen is a fine to coarse powdery substance made up of microspores in seed
plants. It is produced by the male reproductive organs of a flower, called the
anther, and contains the male gametes (sperm cells) of the plant.
 The transfer of pollen from the anther to the stigma of a flower is called
pollination.
 If the pollen lands on the stigma of the same flower or another flower of the
same plant, it is called self-pollination. When the pollen of a flower lands on
the stigma of a flower of a different plant of the same kind, it is called cross-
pollination.
 Tomatoes are self-pollinating, meaning they have flowers that contain both
the male and female parts, so more than one plant is not needed for
reproduction. Hence pair 2 is correctly matched.

Science & Technology and Environment by Ravi Garlapati Page 39


6. Fragmentation
 It is a form of asexual reproduction or cloning, where an organism is split into
fragments.
 Each of these fragments develops into mature, fully grown individuals that are
clones of the original organism.
 When water and nutrients are available algae grow and multiply rapidly by
fragmentation.
 An alga breaks up into two or more fragments. These fragments or pieces
grow into new individuals. This process continues and they cover a large area
in a short period of time.
Yeast reproduces through Budding. Hence pair 1 is not correctly matched.
Budding
 It is an asexual reproduction method in which a new organism develops from
a bud of an existing organism.
 Until the new organism matures, it remains attached to the parent organism.
It is separated from the parent organism when it gets matured by leaving scar
tissues behind.
 Bacteria, yeast, corals, flatworms, Jellyfish and sea anemones are some animal
species which reproduce through budding.

7. Plants do not move, and plant bodies have a large proportion of dead cells in many
tissues. As a result, plants have low energy needs, and can use relatively slow
transport systems. Plant transport systems will move energy stores from leaves and
raw materials from roots. These two pathways are constructed as independently
organised conducting tubes, xylem and phloem.
The xylem and the phloem transport water, sugars, and other important substances
around a plant. What is commonly referred to as ‘sap’ is indeed the substances that
are being transported around a plant by its xylem and phloem.
One, the xylem moves water and minerals obtained from the soil. The other, phloem
transports products of photosynthesis from the leaves where they are synthesised to
other parts of the plant. Hence, statement 1 and statement 2 are not correct.
Transpiration is a process that involves loss of water vapour through the stomata of
plants. The loss of water vapour from the plant cools the plant down when the
weather is very hot, and water from the stem and roots moves upwards or is 'pulled'
into the leaves. Hence, statement 3 is correct.

8. Plant growth regulators (PGRs) are small, simple molecules of diverse chemical
composition. The PGRs can be broadly divided into two groups based on their
functions in a living plant body.
One group of PGRs are involved in growth promoting activities, such as cell division,
cell enlargement, flowering, fruiting and seed formation. These are also called plant
growth promoters, e.g., auxins, gibberellins and cytokinins.

Science & Technology and Environment by Ravi Garlapati Page 40


The PGRs of the other group play an important role in plant responses to wounds and
stresses. They are also involved in various growth inhibiting activities such as
dormancy and abscission (shedding of leaves, fruits, flowers).
The PGR abscisic acid (ABA) belongs to this group. It acts as a general plant growth
inhibitor and an inhibitor of plant metabolism.
Hence option (a) is the correct answer.
Note: The gaseous PGR, ethylene, could fit either of the groups, but it is largely an
inhibitor of growth activities.

9. Statement 1 is correct as stem cells are undifferentiated cells that can divide and
produce more stem cells while also transforming into specialized cell types. Stem
cells can be obtained from various sources in adults, including embryos, bone
marrow, and umbilical cords.
Statement 2 is also correct as a woman with HIV who received a blood stem cell
transplant to treat her acute myeloid leukemia has reportedly been cured of HIV. The
stem cells in the umbilical cord blood had a gene variant that made them resistant to
HIV infection.

10. PCR is a technique used to "amplify" small segments of DNA, making it valuable in
many laboratory and clinical techniques. Statement 1 is correct.
In PCR, the DNA is first heated to separate it into two strands, and then primers are
added to bind to the DNA. However, in Statement 2, it is incorrectly stated that the
DNA is cooled for separation. The correct order is to heat the DNA before adding the
primers and then cool it. Statement 2 is incorrect.
The RT-PCR test used for COVID-19 employs the NAAT method to amplify the virus's
genetic material for detection. Statement 3 is correct. The NAAT procedure amplifies
the nucleic acids and detects the virus using various methods such as RT-PCR, NEAR,
TMA, LAMP, HDA, CRISPR, and SDA. Therefore, the answer is (b) 2 only as it is the
only incorrect option.

11. Statement 1 is incorrect. Proton therapy uses protons, not x-rays, which can cause
damage to healthy tissues surrounding the tumor.
Statement 2 is correct. Cyclotron is used to speed up protons to create high energy
and give the targeted radiation dose to the tumor.
Statement 3 is correct. Proton therapy delivers less radiation to healthy tissues
surrounding the malignant tumor, lowering the risk of radiation damage to these
tissues. Additionally, it is useful for treating children as it reduces the chance of
harming healthy, growing tissue.

12. DNA is the genetic material present in most living organisms and is found in the cell
nucleus and mitochondria. RNA is also present in the majority of living organisms

Science & Technology and Environment by Ravi Garlapati Page 41


and viruses. However RNA is not present in mitochondria. Thus, statement 1 is
incorrect .
The types of RNA involved in protein synthesis are messenger RNA (mRNA), transfer
RNA (tRNA), and ribosomal RNA (rRNA). mRNA carries genetic information from
DNA and tRNA carries amino acids to the ribosome during protein synthesis. Thus,
statement 2 is incorrect because tRNA carries amino acids, not information codes.

13. Gene banks are facilities that store and preserve genetic material, such as seeds, DNA,
and other biological samples. These banks are essential resources for genetic
research and conservation efforts. Therefore, Statement 1 is correct.
In-situ conservation refers to the conservation and protection of species and
ecosystems in their natural habitats. On the other hand, ex-situ conservation is the
conservation of species and ecosystems outside of their natural habitats, such as in
gene banks. Therefore, Statement 2 is not correct.
The National Bureau of Plant Genetic Resources has established the world's second-
largest refurbished gene bank in New Delhi, which can safely store up to one million
germplasms. Hence, Statement is 3 correct.

14. Green manure is un-decomposed material used as manure and is made up of plants
from the leguminous family that are grown specifically to maintain soil fertility and
structure. Hence statement 1 is correct.
Green leaf manure, on the other hand, involves the application of green leaves and
twigs of trees, shrubs, and herbs collected from elsewhere. The source of green leaf
manure is forest trees and plants growing in wastelands, field bunds, and specific
plant species such as neem, mahua, wild indigo, Glyricidia, Karanji (Pongamia
glabra), Calotropis, Avise (Sesbania grandiflora), Subabul, and other shrubs. As given
plant/trees are green leaf manures, not green manure, statement 2 is incorrect.
Dhaincha, Cowpea, Sunhemp, summer moong, marsh pulses, and guar are crops
grown as green manure crops to improve soil health and enhance crop productivity.
High-intensity agricultural practices lead to deficiencies of micronutrients such as
iron and zinc, affecting soil productivity, which is why green manuring helps improve
soil health and crop productivity. Hence statement 3 is correct.

15. Recent context: Researchers reported fourth patient of HIV cured after stem cell
transplant (SCT). SCT, also called bone marrow transplant, is a procedure in which a
patient receives healthy stem cells to replace damaged stem cells. Hence statement 1
is correct.
Two main types:
Autologous transplantation: Uses patient’s own stem cells. These cells are removed,
treated and returned to his body after a conditioning regimen.
Allogeneic transplantation: Uses stem cells from a donor. Hence statement 2 is not
correct.

Science & Technology and Environment by Ravi Garlapati Page 42


16. RNA interference (RNAi) is a biological process that occurs within cells, in which
small RNA molecules are used to silence or "turn off" specific genes. This is
accomplished by the RNA molecules binding to and targeting messenger RNA
(mRNA) molecules, which are responsible for carrying genetic information from the
DNA in the cell's nucleus to the ribosomes, where proteins are synthesized.
When RNAi occurs, the targeted mRNA is destroyed or prevented from being
translated into protein, effectively shutting down the gene's expression.
Statement 1 and 4 are correct: RNA interference (RNAi) or Post-Transcriptional Gene
Silencing (PTGS) may have important practical applications in agriculture (to
produce crop plants that are resistant to viral pathogens) and other areas.
Statement 2 is correct: RNAi is used in developing therapies for the treatment of viral
infection, dominant disorders, neurological disorders, and many types of cancers.
Statement 3 is incorrect: Hormone replacement therapy (HRT) is a treatment to
relieve symptoms of the menopause. It replaces hormones (Oestrogen and
Progesterone) that are at a lower level as you approach the menopause.

17. Pair 1 is incorrect: Gene cloning produces copies of genes or segments of DNA. Hence,
it is also known as DNA cloning, where a fragment of DNA is cloned. It has several
applications:
Medical application including synthesis of vitamins, hormones, and antibiotics;
Agricultural applications including production of nitrogen fixing bacteria;
Pair 2 is correct: Reproductive cloning produces copies of whole animal. It involves
transfer of DNA from the donor animal’s somatic cell to an oocyte. Dolly, the sheep
was the first successful reproductive cloning experiment. Samrupa, the world's first
cloned buffalo calf that was cloned at Karnal’s National Dairy Research Institute.
Pair 3 is correct: In some of the recent researches held on various animals, it is found
that the neurodegenerative diseases like Parkinson’s disease and Alzheimer's can be
treated with the use of therapeutic cloning.

18. Genomic surveillance is the process of constantly monitoring pathogens and


analysing their genetic similarities and differences. Genomic surveillance works on
the basis that everything with a genome (humans, animals, plants, bacteria, viruses
etc.) needs to replicate their genetic material in order to reproduce, and this results
in changes known as mutations. Genomic surveillance is critical for stronger
pandemic and epidemic preparedness and response.

19. The Department of Biotechnology (DBT) had issued Guidelines for Safety Assessment
of Genome Edited Plants in 2022 to ease norms for developing GM crops.
Statement 1 is incorrect: The genome edited plants derived from the use of genome
editing techniques are generally classified under three categories:

Science & Technology and Environment by Ravi Garlapati Page 43


Site-Directed Nuclease (SDN)-1, a site-directed mutagenesis (the process of
intentionally introducing mutations or changes in the genetic material (DNA or RNA)
of an organism) without using a DNA sequence template. It doesn’t involve insertion
of a DNA fragment from foreign genome.
SDN-2, a site-directed mutagenesis using a DNA sequence template. It also doesn’t
involve insertion of foreign genetic material.
SDN-3, site-directed insertion of gene/large DNA sequence using a DNA sequence
template, which involves insertion of DNA fragment of a foreign origin.
Site-Directed Nuclease (SDN)- Site-directed nucleases (SDNs) are a class of
engineered nucleases that can cleave DNA at specific sites within the genome. SDNs
are commonly used in genome editing techniques to introduce precise modifications
to the DNA sequence of an organism.
Statement 2 is Incorrect: The ‘Guidelines for Safety Assessment of Genome Edited
Plants, 2022’ exempt researchers who use gene-editing technology to modify the
genome of the plant from seeking approvals from the Genetic Engineering Appraisal
Committee (SDN 1 and SDN 2 are exempted from rule 7-11 of the Environment
Protection Act).

20. The Department of Biotechnology (DBT) had issued Guidelines for Safety Assessment
of Genome Edited Plants in 2022 to ease norms for developing GM crops.
Statement 1 is correct: CRISPR-Cas9 (‘clustered regularly interspaced short
palindromic repeats’) is the most common and efficient system used for genome
editing. It is often known as genetic/molecular scissor. A targeted segment of DNA is
located, cut, removed and then replaced with a correct sequence. It does not involve
introduction of foreign genetic material.
Statement 2 is incorrect: Bt-cotton is a genetically modified transgenic plant that has
been grown in the country for the past two decades. It contains the pesticide gene
from the bacteria Bacillus thuringiensis (Bt). It has been modified to produce an
insecticide to combat the cotton bollworm. On the other hand, non-transgenic plants
do not involve introduction of genetic material from outside. Rather it involves
redesigning (shortening, lengthening, removal) of a gene that is already present in
the plant.

21. Bioplastic refers to plastic made from plants like maize, wheat or sugarcane or other
biological material instead of petroleum. Bio-plastics are biodegradable and
compostable plastic materials.
Polyhydroxyalkanoates (PHAs) are polyesters produced in nature by numerous
microorganisms. Hence option 1 is correct.
Polylactic acid (PLA) is a thermoplastic polyester that has become a popular material
due to it being economically produced from renewable resources and in 2021, had
the highest consumption volume of any bioplastic of the world. Hence option 2 is
correct.

Science & Technology and Environment by Ravi Garlapati Page 44


Polyethylene terephthalate (PET or PETE) is a strong, stiff synthetic fibre and resin
and a member of the polyester family of polymers. Hence option 3 is not correct.

22. Microbial-based pesticides are biological pesticides (bio-pesticides) that are derived
from microorganisms such as bacteria, fungi, viruses, and protozoa. These pesticides
are used to control pests, weeds, and diseases in agriculture, horticulture, and
forestry.
One example of a microbial-based pesticide is Bacillus thuringiensis (Bt), a naturally
occurring soil bacterium that produces a protein that is toxic to certain insects. Bt is
commonly used to control pests such as caterpillars, beetles, and moths in crops like
corn, cotton, and vegetables.
Another example is Trichoderma, a fungi that can be used as a biocontrol agent
against plant pathogens, and can also promote plant growth.

23. Fortification involves adding essential micronutrients to food to improve its


nutritional quality. The FSSAI provides regulations and guidelines for packaging and
labelling of food products, food standards, and food additives. Therefore, statement 1
is correct.
According to FSSAI standards, packaged milk must be fortified with Vitamin A and D,
and the notification to make this mandatory was issued in December 2020. Hence,
statement 2 is also correct.
Golden Rice is biofortified with provitamin A and does not require any significant
changes in cultivation practices. Thus, statement 3 is incorrect.

24. The Food Safety and Standards Authority of India (FSSAI) has been established under
Food Safety and Standards Act, 2006 which consolidates various acts & orders that
have hitherto handled food related issues in various Ministries and Departments.
It is an autonomous statutory body which regulates and monitor manufacturing,
distribution and processing of food products. Hence, statement 1 is correct.
FSSAI has been created for laying down science based standards for articles of food
and to regulate their manufacture, storage, distribution, sale and import to ensure
availability of safe and wholesome food for human consumption.
Ministry of Health & Family Welfare, Government of India is the Administrative
Ministry for the implementation of FSSAI. Hence, statement 2 is correct.

Exercise – V 1. It is a double-stranded DNA virus


Topic: Infections & Diseases that affects human beings.
2. Gardasil 9 is a gender-neutral HPV
1. Regarding Human Papillomavirus vaccine manufactured by Merck &
(HPV), which of the following Co and is available in India.
statements is/are true? 3. Cervical cancer is primarily caused
by HPV.

Science & Technology and Environment by Ravi Garlapati Page 45


(a) 1 and 2 only (b) 2 and 3 (c) 2, 3 and 4 only (d) 3 and 5 only
only
(c) 2 only (d) 1, 2 and 3 only 4. Consider the following pairs:
Vitamins/Minerals: Deficiency Diseases
2. Which of the following statements 1. Vitamin A : Night blindness
accurately describe antimicrobial 2. Vitamin D : Scurvy
resistance? 3. Iron : Anaemia
1. Antibiotic resistance does not 4. Iodine : Goitre
occur naturally and is primarily How many pairs given above are
caused by the inappropriate use of correctly matched?
antibiotics in humans and animal (a) Only one pair
feed. (b) Only two pairs
2. The use of Colistin in poultry is (c) Only three pairs
prohibited in India.
(d) All four pairs
3. Antibiotic resistance in bacteria
can result from genetic changes
5. With reference to Enzyme-linked
that reduce the effectiveness of
Immunosorbent Assay (ELISA),
drugs used to treat infections.
consider the following statements:
(a) 1 and 2 only (b) 1 and 3 only
1. Performing an ELISA involves at
(c) 2 and 3 only (d) 1, 2 and 3 least one antibody with specificity
for a particular antigen.
3. Lumpy Skin Disease (LSD) is caused in 2. It can be used to detect Covid
cattle and water buffaloes by a antibodies that the body develops in
poxvirus called Lumpy Skin Disease response to infection by the SARS-
Virus (LSDV). Consider the following CoV-2 virus.
statements about this disease: Which of the statements given above
1. LSD virus have a single-stranded is/are correct?
DNA genome.
(a) 1 only (b) 2 only
2. It is a non-zoonotic disease
(c) Both 1 and 2 (d) Neither 1 nor 2
therefore it is safe to drink milk from
the infected cattle.
6. Consider the following statements
3. Incubation period of LSD is around
regarding Viruses:
28 days.
1. All viruses contains both RNA and
4. Due to rapid mutations in LSDV, a
DNA.
vaccine against it cannot be
2. Bacteriophages (viruses that infect
developed.
the bacteria) are usually single-
5. It has a very high mortality of stranded DNA viruses
greater than 50%. Which of the statements given above
Which of the statements given above is/are correct?
are correct? (a) 1 only (b) 2 only
(a) 1 and 2 only (b) 2 and 3 (c) Both 1 and 2 (d) Neither 1 nor 2
only

Science & Technology and Environment by Ravi Garlapati Page 46


7. With reference to hand-foot-and- 10. Which of the following statements
mouth disease (HFMD), consider the given below are correct?
following statements: 1. Swine flu is a zoonotic respiratory
1. It is different from foot-and-mouth disease.
disease which commonly affects 2. WHO classifies swine flu as the
cows, sheep, and pigs. Neglected tropical disease.
2. Tomato flu is a clinical variant of 3. Dizziness, sore throat and watery
the HFMD and results in tomato- eyes are the symptoms of swine flu.
shaped red blisters on several Select the correct answer using the
parts of the body. code given below:
Which of the statements given above (a) 1 and 2 only (b) 2 and 3 only
is/are correct? (c) 1 and 3 only (d) 1, 2 and 3
(a) 1 only (b) 2 only
(c) Both 1 and 2 (d) Neither 1 11. Which of the following diseases are
nor 2 classified as Neglected Tropical
Diseases by WHO?
8. Consider the below claims about the 1. Trachoma
Central Drugs Standard Control 2. Leprosy
Organization: 3. Snake-bite (envenoming)
1. It is in charge of approving new 4. Helminthiases
medications and clinical trials across 5. Sleeping sickness
the nation.
Select the correct answer using the
2. The Directorate General of Health code given below:
Services oversees its operations.
(a) 1, 2, and 3 only
Which of the aforementioned
(b) 1, 2, and 4 only
statements is true?
(c) 2 and 4 only
(a) 1 only (b) 2 only
(d) All the above
(c) Both 1 and 2 (d) Neither 1 nor 2
12. Which of the following statements are
9. Which of the following diseases are
not correct regarding Monoclonal
caused by protozoa?
Antibodies (mAbs) ?
1. Dengue
1. They are artificially created
2. Malaria proteins that mimic human
3. Kala-Azar antibodies in the immune system.
4. Tuberculosis 2. Monoclonal antibodies are highly
Select the correct answer using the reactive and can bind to more than
code given below. one protein simultaneously.
(a) 1, 2 and 4 only 3. CAR-T cell therapy involves
(b) 2 and 3 only genetically engineered White
(c) 1, 2, 3 and 4 Blood cells to target and destroy
(d) 1 and 3 only cancer cells.

Science & Technology and Environment by Ravi Garlapati Page 47


Select the correct answer using the while B- lymphocytes help T- cells to
code given below : produce them.
(a) 1 and 3 only (b) 2 only 3. Hydrochloric Acid (HCL) produced
(c) 2 and 3 only (d) 1, 2 and 3 in stomach is an example for innate
immune response.
13. What are the defense mechanisms Which of the statements given above
found in the human body? Which of is/are correct ?
the options below is/are included? (a) 1 only (b) 1 and 2 only
1. Immune system for protection (c) 3 only (d) 1 and 3 only
against infections.
2. Hormonal release as a means of 16. Which of the following statements
stress resistance. is/are correct?
3. Haemostasis to halt bleeding and 1. BCG vaccine provides protection
prevent blood loss. against meningitis and
Choose the appropriate response disseminated TB in children.
code: 2. BCG vaccine is not a part of India's
(a) 1 only (b) 1 and 2 only universal immunization program.
(c) 1 and 3 only (d) 1, 2 and 3 3. XDR-TB in HIV patients leads to
high mortality.
14. Consider the following statements: (a) 1 only (b) 1 and 2 only
1. DNA vaccines are stable at (c) 3 only (d) 1 and 3 only
higher temperatures and thus
easier to store unlike RNA 17. The term 90-90-90 strategy recently
vaccines. seen in news is related to which one of
2. DNA vaccines, once injected, the following diseases?
cannot alter natural genetic (a) AIDS (b) Monkeypox
material of a cell unlike RNA virus
vaccines. (c) Nipah virus (d) SARS
Which of the statements given above
is/are correct? 18. Which of the following diseases
(a) 1 only (b) 2 only recently in the news is associated with
(c) Both 1 and 2 (d) Neither 1 nor 2 the 'G4' virus?
(a) Yellow fever
15. Consider the following statements, (b) Japanese encephalitis
with reference to Human Immune (c) Swine flu
system : (d) Bird flu
1. When our body encounters a
pathogen for the first time, it produces
anamnestic immune response.
2. T-lymphocytes produce antibodies
into our blood to fight with pathogens,

Science & Technology and Environment by Ravi Garlapati Page 48


Exercise – V Key

1. d 2. c 3. b 4. c 5. c 6. d 7. c 8. c 9. b 10. c
11. d 12. b 13. d 14. a 15. c 16. d 17. a 18. c

Explanations

1. HPV is a group of more than 200 related viruses that can be transmitted through
sexual and non-sexual modes, including direct skin-to-skin contact. HPV is a double-
stranded DNA virus, hence statement 1 is correct.
Prevention of cervical cancer through vaccination is an effective option, and there are
several vaccines available in the market. Gardasil 9 is a gender-neutral HPV vaccine
manufactured by Merck & Co, which is available in India. Cervavac is the first
indigenously developed quadrivalent HPV vaccine for the prevention of cervical
cancer, manufactured by the Serum Institute of India. Therefore, statement 2 is
correct.
Cervical cancer is the most common cancer caused by HPV infection. Persistent
infection with high-risk HPV types, particularly HPV 16 and 18, can lead to the
development of cervical, vulvar, vaginal, anal, penile, and oropharyngeal cancers.
Therefore, statement 3 is correct.

2. Antibiotic or antimicrobial resistance occurs when bacteria and fungi acquire the
ability to survive exposure to drugs that are meant to kill them. As a result, the germs
can continue to grow and cause infections that are difficult, and sometimes
impossible, to treat. Although resistance is a natural phenomenon, the misuse of
antibiotics in humans and animals accelerates the process, making it a major cause of
antibiotic resistance. Therefore, Statement 1 is incorrect.
In 2019, India's Ministry of Health banned the manufacture, sale, and distribution of
colistin, an antibiotic often misused in poultry industry, for food-producing animals,
poultry, aqua farming, and animal feed supplements. Therefore, Statement 2 is
correct.
Antibiotic resistance develops in bacteria when they acquire the ability to survive
exposure to drugs designed to kill them. This ability typically occurs through genetic
mutations. Hence, Statement 3 is correct.

3. Lumpy Skin Disease has recently spread in India, and it has severe implications on
the economy of India.
Statement 1 is incorrect: LSD is caused by lumpy skin disease virus (LSDV) which
belongs to the genus capri poxvirus. All Capri poxviruses have a double-stranded
DNA genome.
Statement 2 is correct: LSD is a non-zoonotic disease i.e.; the disease is non
transmissible from animals to humans. It cannot be transmitted either by direct

Science & Technology and Environment by Ravi Garlapati Page 49


contact or by consumption of milk. Therefore, it is safe to drink milk from the
infected cattle.
Statement 3 is correct: Incubation period of LSD disease is around 28 days, but
experimentally infected cattle may develop clinical signs as early as 6-9 days. It is
characterized by a mild fever, followed by the sudden appearance of skin nodules.
Statement 4 is incorrect: Indian Council of Agricultural Research (ICAR) is aiming to
commercially launch the “Lumpi-ProVacInd” Vaccine. This vaccine provides 100%
protection against LSD in cattle.
Statement 5 is incorrect: According to the Food and Agriculture Organization (FAO),
the mortality rate is less than 10%.

4. Diseases that occur due to lack of nutrients over a long period are called deficiency
diseases.
In its more severe forms, vitamin A deficiency contributes to blindness by making the
cornea very dry, thus damaging the retina and cornea. Hence pair 1 is correctly
matched.
Scurvy is the most prominent disease linked to Vitamin C deficiency. It denotes a
heavy lack of vitamin C in the diet. Hence pair 2 is not correctly matched.
Vitamin D deficiency can lead to a loss of bone density, which can contribute to
osteoporosis and fractures (broken bones). In children, it can cause rickets- a rare
disease that causes the bones to become soft and bend.
Anaemia is a condition in which blood lacks adequate healthy red blood cells. Red
blood cells carry oxygen to the body's tissues. Anaemia typically happens due to
insufficient iron. Hence pair 3 is correctly matched.
Goiter is a condition where the thyroid gland grows larger. Iodine deficiency is the
most common cause of goiter in the world. Hence pair 4 is correctly matched.

5. The enzyme-linked immunosorbent assay (ELISA) is an immunological assay (test)


commonly used to measure antibodies, antigens, proteins and glycoproteins in
biological samples.
Antibodies are blood proteins produced in response to a specific antigen. ELISA helps
to examine the presence of antibodies in the body, in case of certain infectious
diseases.
Principle of ELISA: ELISA works on the principle that specific antibodies bind the
target antigen and detect the presence and quantity of antigens binding. Performing
an ELISA involves at least one antibody with specificity for a particular antigen.
ELISA can provide a useful measurement of antigen-antibody concentration. Hence,
statement 1 is correct.

Science & Technology and Environment by Ravi Garlapati Page 50


Indian Council of Medical Research (ICMR)-National Institute of Virology (NIV) at
Pune has developed and validated the indigenous IgG ELISA test “COVID KAVACH
ELISA” for antibody detection for COVID-19. It is an IgG Elisa-based test. This means
that the test will be done to detect the Immunoglobulin G (IgG) antibody.
If the IgG antibody is detected, it can be concluded that the person was exposed to
SARS-CoV-2. Hence, statement 2 is correct.

6. In addition to proteins, viruses also contain genetic material, that could be either
RNA or DNA. Leuko virus contains both DNA and RNA. Hence statement 1 is not
correct.
In general, viruses that infect plants have single stranded RNA and viruses that infect
animals have either single or double stranded RNA or double stranded DNA.
Bacterial viruses or bacteriophages (viruses that infect the bacteria) are usually
double stranded DNA viruses. Hence statement 2 is not correct.
Viruses cause diseases like mumps, small pox, herpes and influenza. AIDS in humans
is also caused by a virus.

7. An increase in cases of hand-foot-and-mouth disease (HFMD) has been reported in


Delhi-NCR in 2022.
Hand-foot-and-mouth disease is a mild, contagious viral infection common in young
children.
Symptoms include
sores in the mouth
and a rash on the
hands and feet. it is a
mild, self-limiting
disease with no
significant adverse
effects. It lasts seven
to ten days.
Hand-foot-and-mouth
disease is most
commonly caused by
a coxsackievirus. This
virus is part of a
group of viruses
called enteroviruses
(viruses transmitted
through the
intestine).
Hand, foot, and mouth disease is often confused with foot-and-mouth disease (FMD),
which affects cows, sheep, and pigs. Humans do not get the animal disease, and
animals do not get the human disease. Hence, statement 1 is correct.
Science & Technology and Environment by Ravi Garlapati Page 51
Tomato Flu is caused by Coxsackievirus A16. Tomato flu has been reported from at
least four states — Kerala, Tamil Nadu, Haryana, and Odisha.
Researchers believe that it is a different clinical presentation of hand-foot-and mouth
disease (HFMD). Hence, statement 2 is correct.

8. The Ministry of Health & Family Welfare's Directorate General of Health Services is re
sponsible for CDSCO. As a result, statement 2 is true.
In accordance with the Drugs and Cosmetics Act, 1940, CDSCO is in charge of: Drug
approval; Clinical trial administration; Drug standard setting; and Regulation of the
quality of imported drugs in the nation. Thus, statement 1 is true.

9. Dengue is caused by a virus while Tuberculosis is caused by bacteria.


Protozoan-caused Diseases:
 Kala-azar: Leishmania Donovani parasite is the root cause
 Malaria: Plasmodium parasite causes malaria. Female Anopheles mosquitoes
spread it.
 Diarrhea: Although several intestinal protozoa can induce diarrhoea, the most
significant ones are Entamoeba histolytica, and Giardia intestinalis.
 sleeping sickness: Trypanosoma brucei causes it.

10. Swine flu (H1N1) is a respiratory disease caused by type A influenza viruses that are
transmitted from pigs to humans. It can cause serious illnesses like pneumonia and
lung infection, and make other illnesses like asthma or diabetes worse. Swine flu is
recognized as a global health problem. Hence, statement 1 is correct.
Neglected tropical diseases (NTDs) are found in several countries in Africa, Asia, and
Latin America and are especially common in tropical areas where people do not have
access to clean water or safe ways to dispose of human waste. Swine flu is not
classified as an NTD by the WHO. Hence, statement 2 is not correct.
The symptoms of Swine flu include fever, cough, runny nose, sore throat, body aches,
chills, fatigue, dizziness, and red, watery eyes. Hence, statement 3 is correct.

11. The World Health Organisation (WHO) identifies 20 different diseases that are
common in tropical regions as Neglected Tropical Diseases.
These diseases primarily impact poor communities, and women and children in
tropical regions are disproportionately affected by them.
Environmental factors often contribute to these diseases, and many of them are
spread by vectors or have complex life cycles, making their control difficult for public
health officials.
Neglected Tropical Diseases:
Buruli ulcer, Chagas disease, dengue and chikungunya, dracunculiasis (Guineaworm
disease), echinococcosis, foodborne trematodiases, human African trypanosomiasis
(sleeping sickness), leishmaniasis, leprosy (Hansen's disease), lymphatic filariasis,

Science & Technology and Environment by Ravi Garlapati Page 52


mycetoma, chromoblastomycosis, and other deep mycoses, onchocerciasis (river
blindness), rabies, scabies, and other ectoparasitoses, schistosomiasis, soil-
transmitted helminthiases, snakebite envenoming, taeniasis/cysticercosis, trachoma,
and yaws and other endemic treponematoses.

12. The immune system produces antibodies, which are proteins that attach to specific
antigens (foreign substances) in the body. Monoclonal antibodies (mAbs or MoAbs)
are artificially created proteins that mimic human antibodies and can be designed to
target specific antigens. So, statement 1 is correct.
Contrary to statement 2, monoclonal antibodies are highly specific and typically bind
to a single protein or antigen. They do not bind to more than one protein
simultaneously.
CAR-T cell therapy is a type of cancer immunotherapy in which White Blood cells (T
cells) are genetically engineered in a laboratory to express chimeric antigen
receptors (CARs) on their surface. These CARs allow the modified T cells to target
and destroy cancer cells. So, statement 3 is correct.

13. The body's initial defense mechanisms that provide protective responses against
various threats are referred to as "innate," "inborn," or "nonspecific" immunity.
These primary forms of host defense mechanisms in our body include:
 Immunity to protect against infections
 Metabolic defense to detoxify and metabolize foreign chemicals
 Hemostasis to stop bleeding and prevent blood loss
 Release of hormones as a means of stress resistance.

14. (For a detailed differences between DNA & RNA Vaccines refer to the descriptive
question in earlier section)
Comparatively, DNA vaccines are safe, easy, affordable to produce, and, unlike RNA
vaccines, are stable at room temperature. Hence, statement 1 is correct.
RNA vaccines are believed by many scientists to be less risky than DNA vaccines
since they would not affect the host’s DNA or even have to enter the nucleus of a host
cell because the mRNA has already been formed. This also means the process of
making antibodies is faster. Hence, statement 2 is not correct.

15. The primary immune response happens when the immune system encounters a new
pathogen for the first time. The anamnestic immune response happens when the
immune system has already encountered the same pathogen before.
B-cells are responsible for starting the antibody-mediated immune response. They
mature in the bone marrow and then travel in the blood to other parts of the body.
When activated, B-cells turn into plasma cells that release antibodies. T-cells help
regulate the immune response and activate B-cells to produce antibodies. They also

Science & Technology and Environment by Ravi Garlapati Page 53


play a role in the cell-mediated immune response. Therefore, statements 1 and 2 are
both incorrect.
The hydrochloric acid (HCL) by the stomach plays an important role in protecting
the body against pathogens ingested with food or water. It is an example of Innate
immune response (natural, immediate, non-specific immune response). So,
Statement 3 is correct.

16. Statement 1 is correct, as the BCG vaccine provides protection against meningitis and
disseminated TB in children with an efficacy of 75-87%. However, it does not prevent
primary infection and the reactivation of latent pulmonary infection (which causes
TB).
Statement 2 is incorrect, as the BCG vaccine is a part of India's universal
immunization program.
Statement 3 is correct, as Extensively drug-resistant tuberculosis (XDR-TB) is a form
of TB that is resistant to many anti-TB drugs, making it difficult to treat. HIV patients
are particularly susceptible to XDR-TB, which increases the mortality rate.

17. United Nations Programme on HIV/AIDS (UNAIDS) created a plan called the 90-90-
90 Strategy to stop the spread of AIDS. The plan had three goals to reach by 2020:
1. Make sure 90% of people with HIV know they have it.
2. Make sure 90% of people with HIV are getting treatment.
3. Make sure 90% of people receiving treatment have low levels of the virus.
At the end of 2020, the progress towards these goals was:
1. 78% of people with HIV knew they had it.
2. 83% of people with HIV who knew they had it were receiving treatment.
3. 85% of people with HIV who were receiving treatment had low levels of the virus.

18. A new strain of swine flu virus (H1N1) has been recently discovered in pigs in China
by scientists, which has been given the name G4. This new strain, also known as G4
EA H1N1, is believed to have descended genetically from the H1N1 strain that caused
a pandemic in 2009. Researchers identified the new strain through surveillance of
influenza viruses in pigs during a study that was conducted between 2011 and 2018
in ten provinces of China. They found that this strain of influenza virus is infecting
Chinese pigs and has the potential to cause a pandemic. Experiments have shown
that the G4 virus can be transmitted from animals to humans, but there is no
evidence yet that it can be transmitted from human to human.

Science & Technology and Environment by Ravi Garlapati Page 54


Environment & Disaster Management MCQs
Note: For concepts on Environment refer to Part – II of the book ‘Science & Technology,
Environment by Ravi Garlapati’.
Exercise – I 2. Soliga is an indigenous population
Topic: Environmental Issues including of Karnataka’s Biligiri Rangana Hills
Pollution Which of the statements given above
is/are correct?
1. Consider the following statements a) 1 only b) 2 only
regarding the protected areas in c) Both 1 and 2 d) Neither 1 nor 2
Madhya Pradesh.
1. Madhav National Park is a notified 4. Consider the following statements:
as national park recently. 1. ‘In Our LiFEtime’ campaign was
2. Translocation of tigers project is lauched by India at a side event of the
being implemented in Durgavati Tiger COP 27 at Sharm-el-Sheikh, Egypt.
Reserve
Which of the statements given above 2. The campaign aims to shift to 50%
is/are correct? of total energy needs to solar power
a) 1 only b) 2 only within our lifetime in the spirit of the
c) Both 1 and 2 d) Neither 1 nor 2 goals of International Solar Alliance.

2. Match the following Protected Which of the statements given above


areas/wetlands, recently in news, is/are correct?
with their states: (a) 1 only (b) 2 only
Protected Area State (c) Both 1 and 2 (d) Neither 1 nor 2
1. Kaziranga National A. Assam
Park 5. Consider the following statements:
2. Dharoi wetland B. Kerala 1. Ranipur Tiger Reserve and Pilibhit
3. Periyar Tiger C. Gujarat Tiger Reserve, which were seen in the
Reserve news, are located in Uttar Pradesh.
4. Kanger Valley D. 2. Haryana State Government’s Forest
National Park Chattisgarh Department and the US Agency for
Choose the correct option: International Development (USAID)
a) 1.A, 2.D, 3.B, 4.B announced the launch of the ‘Trees
b) 1.A, 2.B, 3.C, 4.D Outside Forests in India (TOFI)’
c) 1.A, 2.C, 3.B, 4.D program.
d) 1.D, 2.C, 3.B, 4.A
Which of the statements given above
is/are correct?
3. Consider the following statements.
(a) 1 only (b) 2 only
1. ‘Elephant Death Audit Framework’
is launched by Kerala to track the (c) Both 1 and 2 (d) Neither 1
reasons for the death of elephants in nor 2
the state.
Science & Technology and Environment by Ravi Garlapati Page 55
6. Consider the following statements: Which of the statements given above
1. Ministry of Environment issued the is/are correct?
draft Construction of Electric Lines in (a) 1 only (b) 2 only
Great Indian Bustard Area (c) Both 1 and 2 (d) Neither 1
Regulations, 2023 to protect the birds nor
from accidental electrocutions.
2. The ‘Great Indian Bustard’ (GIB) is
the State bird of Rajasthan. 9. Which of the following statements
Which of the statements given above regarding Wetlands are incorrect?
is/are correct? 1. Pantanal is the world’s largest
(a) 1 only (b) 2 only wetland located in South Africa.
(c) Both 1 and 2 (d) Neither 1 2. In India, the Wetlands
nor 2 (Conservation and Management)
Rules exclude river channels and
paddy fields from the definition of
7. Consider the following statements wetlands.
regarding the species discovered 3. Wetlands in India are declared by
recently in India. the Ministry of Jal Shakti.
1. A new species of jumping spider (a) 1 and 2 only
named Pseudomogrus sudhii was (b) 2 only
discovered in Rajasthan. (c) 1 and 3 only
2. Glischropus meghalayanus, is a Bat (d) 2 and 3 only
species discovered in Meghalaya.
Which of the statements given above 10. Consider the following statements
is/are correct? about Background Radiation:
(a) 1 only (b) 2 only 1. Background Radiation is the
(c) Both 1 and 2 (d) Neither 1 measure of radiation emitted from
nor 2 manmade sources like nucler power
plants.

8. Consider the following statements 2. Parts of Kerala are experiencing


about Environmental Impact nearly 3 times more background
Assessment (EIA) in India: radiation than assumed.
1. The Environment Impact Which of the statements given above
Assessment Notification, 2006, is is/are correct?
issued under the Environment a) 1 only b) 2 only
Protection Act 1986. c) Both 1 and 2 d) Neither 1 nor 2
2. Highway Projects of defense
significance that are within 100 11. Consider the following statements
km of the Line of Control are about the Waste to Wealth Mission:
exempted from EIA. 1. It is led by the Ministry of Housing
and Urban Affairs.
Science & Technology and Environment by Ravi Garlapati Page 56
2. The mission has launched the 14. Which of the following methods can
Swachhta Saarthi Fellowship. be used to treat radioactively
Which of the statements given above contaminated water?
is/are correct? 1. Aeration
a) 1 only b) 2 only 2. Reverse osmosis
c) Both 1 and 2 d) Neither 1 nor 2 3. Ion exchange
4. Granule Carbon adsorption
12. Which of the following statements Select the appropriate answer from
about Black Carbon is/are incorrect? the options provided below.
1. Black Carbon has a cooling effect (a) 1 and 2 only
on the atmosphere when it stays (b) 3 and 4 only
above the clouds. (c) 1, 2 and 3 only
2. Black Carbon increases the (d) 1, 2, 3 and 4
reflectivity (albedo) of ice when it
accumulates on it. 15. The following statements pertain to
3. Black Carbon remains in the the Plastic Waste Management Rules
atmosphere for only a few weeks. and related amendments:
Which of the above statements is/are 1. In India, Extended Producer
correct? Responsibility (EPR) is legally
(a) 2 only (b) 1 and 3 only binding.
(c) 1 and 2 only (d) 2 and 3 only 2. Recycled plastic products are not
permitted for storing, carrying,
13. Which of the following statements dispensing, or packaging ready-to-
about Energy Saving Certificates eat or drink foodstuff.
(ESCerts) is/are true? 3. To manufacture carry bags at the
1. ESCerts are market-based state level, registration from the
regulatory tools designed to state pollution control board is
reduce energy consumption in required.
energy-intensive industries. Which of the above statements is/are
2. The Perform, Achieve, and Trade correct?
(PAT) scheme is responsible for (a) 1 Only (b) 1 and 2 Only
issuing ESCerts. (c) 2 and 3 Only (d) 1, 2, and 3
3. The Central Electricity Regulatory
Commission regulates ESCert 16. Consider the statements below with
trading. reference to the Hazardous and Other
Which of the above statements is/are Wastes (Management &
correct? Transboundary Movement) Rules,
(a) 1 Only (b) 1 and 2 Only 2016:
(c) 2 and 3 Only (d) 1, 2, and 3 1. The Ministry of Environment,
Forest and Climate Change is
responsible for the transboundary

Science & Technology and Environment by Ravi Garlapati Page 57


movement of hazardous and other
wastes. 19. Which of the following options
2. The rules prohibit the import of provides the most accurate
hazardous and other wastes for explanation for the term "Dead zone"?
any purpose. a) It is a region in the mountains
Select the correct answer using the where vegetation and wildlife cannot
code given below. survive.
(a) 1 only (b) 2 only b) It is a frozen area in Antarctica that
(c) Both 1 and 2 (d) Neither 1 nor 2 remains permanently below zero
degrees Celsius.
17. The following statements refer to the c) It is a location in the ocean where
Bharat Stage (BS) VI emission there is low oxygen concentration,
standards. Which statements are also known as hypoxia.
correct? d) It is a space beyond the
1. The BS VI standards apply to light geostationary orbit where unused
and heavy-duty vehicles, but not to satellites can be stored.
three-wheeled vehicles.
2. Petrol vehicles will need to reduce 20. Consider the following statements
their nitrogen oxide emissions by with reference to BS VI emissions
25% according to the BS-VI norms:
emission norms. 1. In BS VI vehicles the maximum
3. The BS-VI norms require all permissible Nitrous oxide emission
vehicles to have onboard levels for Petrol vehicles is more
diagnostics. compared to the diesel vehicles.
4. The Ministry of Road Transport 2. BS VI fuel has reduced sulphur
and Highways determines the content of 100 ppm.
standards and timeline for 3. Pollution emissions standards in
implementation. India are fixed and regulated by the
Choose the correct answer using the central pollution control board.
codes given below: 4. Unlike BS IV, BS VI norms deals with
(a) 1 and 2 (b) 2, 3, and 4 selective catalytic reduction and diesel
(c) 1, 2, and 3 (d) 1, 2, 3 and 4 particulate filter.
Which of the statements given above
18. The function of a catalytic converter is is/are correct?
to: (a) 1 and 2 only
(a) Convert biomass into ethanol (b) 2, 3 and 4
(b) Convert sewage sludge into biogas (c) 3 and 4
(c) Convert methane hydrates into (d) 1, 3 and 4
natural gas
(d) Convert toxic gases emitted from
an engine into steam

Science & Technology and Environment by Ravi Garlapati Page 58


Exercise – I Key

1. d 2. c 3. b 4. a 5. c 6. b 7. c 8. c 9. c 10. b
11. b 12. b 13. d 14. d 15. d 16. a 17. c 18. d 19. c 20. c

Explanations

1. The statements are interchanged with respect to each other and are both wrong.
Madhav National Park, which is one of the oldest national parks in Madhya Pradesh,
serves as a crucial link between the Panna Tiger Reserve in Madhya Pradesh and the
Ranthambore Tiger Reserve in Rajasthan. The Madhya Pradesh Forest Department
has constructed three enclosures to accommodate five tigers from other reserves in
an effort to restore the tiger population. Recently, a male and female tiger were
released into the enclosures by government officials.

Madhya Pradesh Wildlife Board has approved the creation of a new tiger reserve
called Durgavati Tiger Reserve, which will be located within the Panna Tiger Reserve
(PTR). Due to the linking of the Ken-Betwa Rivers, approximately one-fourth of the
new reserve's 2,339 square kilometres will be submerged. A green corridor will be
developed to facilitate the movement of tigers from PTR to Durgavati.

2. Kaziranga National Park, a protected area in Assam, is known for its population of
one-horned rhinoceroses. The government has requested a factual report on the
rhino population following concerns about the accuracy of the March 2022 survey
results, which counted 2,613 rhinos.
The Dharoi wetland in Gujarat was the site of the second annual Bird Survey Dharoi
2023, which recorded 193 bird species out of the 616 found in the state. The wetland,
spread over 107 square kilometres, was created in 1978 by damming the Sabarmati
and Harnoi rivers for irrigation purposes.
Periyar Tiger Reserve in Kerala was designated as a national park in 1982 and was
the focus of a recent 4-day bird survey by the state forest department. The survey
identified 231 bird species, including 14 endangered and 20 endemic to the Western
Ghats, and documented 11 bird species not recorded in previous surveys.
The Kanger Valley National Park in Chhattisgarh is home to the state bird, the hill
myna, as well as over 200 other bird species identified in the state's first inter-state
bird survey conducted by bird watchers and forest department officials.

3. Statement 1 is incorrect. Tamil Nadu has launched the 'Tamil Nadu Elephant Death
Audit Framework' to enhance accountability by documenting the reasons for
elephant deaths. Only 13 of the 131 recorded elephant deaths in Tamil Nadu's forest
divisions from January 2021 to March 2022 were human-induced, as per media
reports.

Science & Technology and Environment by Ravi Garlapati Page 59


Statement 2 is correct. A new genus of wasp has been named Soliga ecarinata by
entomologists in recognition of the Soliga community's conservation of forests and
biodiversity in the Biligiri Rangan Hills of Karnataka. The Soliga lifestyle was also
acknowledged in the naming of the new genus.

4. Statement 1 is correct. The 'In Our LiFEtime' campaign was launched by India's
Ministry of Environment, Forest and Climate Change and the United Nations
Development Programme (UNDP) during a COP 27 side event in Sharm-el-Sheikh,
Egypt.
Statement 2 is incorrect. The campaign is intended to motivate young people to
promote sustainable living practices. The LiFE concept was first introduced by Prime
Minister Narendra Modi during COP 26 in Glasgow in November 2021.
The International Solar Alliance (ISA) is a global alliance of over 120 countries
initiated by India with the aim to promote the adoption of solar energy to achieve
sustainable development goals.

5. The Pilibhit Tiger Reserve (PTR) in Uttar Pradesh currently has around 27 tigers
residing in the sugarcane fields, but they will soon be relocated to the newly-
established Ranipur Tiger Reserve (RTR). It is to note that the Ranipur Tiger Reserve
(RTR), which is the fourth reserve of its kind in Uttar Pradesh, currently does not
have any tigers living there. Hence statement 1 is correct.
The launch of the ‘Trees Outside Forests in India (TOFI)’ program was announced by
the Forest Department of the Haryana State Government and the US Agency for
International Development (USAID). The program will provide funding of up to USD
25 million over a period of five years to seven states, which include Assam, Andhra
Pradesh, Haryana, Odisha, Rajasthan, Tamil Nadu, and Uttar Pradesh. The Center for
International Forestry Research and World Agroforestry (ICRAF) will be providing
assistance to the program. Hence statement 2 is correct.

6. Statement 1 is incorrect. The Draft Construction of Electric Lines in Great Indian


Bustard Area Regulations, 2023 were issued by Central Electricity Authority (not
Ministry of Environment) to make all electric lines of 33 kV and below passing via the
Great Indian Bustard Area underground, while overhead lines with bird flight diverts
will be used for those above 33 kV.
Statement 2 is correct. The Great Indian Bustard (GIB), which is protected under the
Wildlife Protection Act, is considered to be India's most critically endangered bird,
with about 150 of them living in Rajasthan, accounting for 95% of the world
population. GIB is the state bird of Rajasthan.
Recently, some GIBs were spotted in Pakistan, and environmentalists believe they
may have migrated there. This bird was recently added to the world list of protected
species of the 'Convention on the Conservation of Migratory Species of Wild Animals.'

Science & Technology and Environment by Ravi Garlapati Page 60


7. A new type of bat residing in bamboo forests in Meghalaya has been discovered by a
group of scientists. The newly discovered species has been named "Glischropus
meghalayanus" in honor of Meghalaya state which has completed 50 years of
statehood. Hence statement 1 is correct.
This discovery brings the total number of bat species found in India to 131 with
Meghalaya having the highest diversity of bats, comprising of 67 species.
A group of researchers recently found a new species of jumping spider in Rajasthan's
Desert National Park, which they named Pseudomogrus sudhii after arachnologist
Sudhikumar. This 4-millimetre-long spider is exclusively found in India and the
United Arab Emirates. Hence statement 2 is also correct.

8. Environmental Impact Assessment (EIA) identifies and predicts the impact of an


action on human health and the ecosystem. In India, EIA is notified under the
Environment Protection Act 1986. The public is involved in the process, and the
clearance process involves four stages: screening, scoping, public consultation, and
appraisal.
The recent amendment to the EIA Rules exempts certain highway projects of
strategic and defence importance, which are 100 km from the Line of Control, from
environmental clearance. Therefore, both statements are correct.

9. Statement 1 is incorrect as Pantanal, the world's largest tropical wetland, is situated


across Bolivia, Brazil, and Paraguay is located in South America and not South Africa.
Statement 2 is correct as the Wetlands (Conservation and Management) Rules
exclude river channels and paddy fields from the definition of wetlands in India.
Statement 3 is incorrect as wetlands in India are declared by the Union Environment
Ministry and not the Ministry of Jal Shakti.

10. Statement 1 is incorrect. Background radiation is the amount of radiation emitted by


natural sources such as sand, rocks, or mountains.
Statement 2 is correct. A recent study conducted by scientists at BARC revealed that
certain areas in Kerala are exposed to background radiation levels almost three times
higher than previously estimated. The study found that the radiation level in these
areas is around 70 milliGray per year, slightly higher than the radiation exposure for
nuclear plant workers.

11. The Waste to Wealth Mission is one of the nine scientific missions initiated by the
Prime Minister's Science, Technology, and Innovation Advisory Council (PMSTIAC)
and is managed by the office of the Principal Scientific Adviser (PSA) of the Indian
government. The goal of the mission is to find and implement technologies that can
convert waste into energy, recycle materials, and extract valuable resources.
Therefore, statement 1 is incorrect.

Science & Technology and Environment by Ravi Garlapati Page 61


The mission aims to integrate with Swachh Bharat and Smart Cities initiatives to
create circular economic models for financially viable waste management. The Indian
government has launched the "Swachhta Saarthi Fellowship 2022" under the Waste
to Wealth Mission to support young innovators who engage in community work for
waste management, awareness campaigns, surveys, etc. as Swachhta Saarthis. Thus,
statement 2 is correct.

12. Black carbon (BC) is a type of fine particulate matter made up of pure carbon in
various forms, resulting from incomplete combustion of fossil fuels, biofuels, and
biomass.
Although BC can stay in the atmosphere for only a few days to weeks, it contributes
to global warming in two ways: it absorbs sunlight and generates heat in the
atmosphere and settles on snow and ice, making it darker and hence absorbs more
sunlight, (thus reducing the albedo, which refers to the amount of energy reflected by
a surface) leading to more melting. This is a significant concern in regions like the
Arctic and Himalayas. Hence statements 2 is incorrect and 3 is correct.
However, when it hangs above low-lying clouds, it stabilizes the air layer above the
clouds, promoting their growth and cooling the planet. Hence statement 1 is correct.

13. The National Mission on Enhanced Energy Efficiency (NMEEE) has launched the
Perform, Achieve and Trade (PAT) Scheme, which is aimed at reducing specific
energy consumption (SEC) in large, energy-intensive industries. The Energy Saving
Certificate (ESCert) is a market-based regulatory mechanism under the PAT scheme
that helps to achieve this goal. Hence, statements 1 and 2 are correct.
Under this scheme, ESCerts are issued to those plants that have achieved their
targets, and plants that have failed to reach their targets can purchase ESCerts. The
designated consumers who are covered by this scheme include 13 sectors such as
Aluminium, DISCOMs, Hotels (under commercial buildings), Cement, fertilizers, Pulp
& Paper, Thermal Power Plant, Iron & Steel, etc. When a designated consumer
overachieves the notified SEC targets in the compliance year, the ESCerts are issued
by the Central Government for the difference between the notified target and the
achieved SEC, according to the PAT rules.
The Central Electricity Regulatory Commission (CERC) under the Bureau of Energy
Efficiency (BEE) regulates the trading of ESCerts. Therefore, statement 3 is also
correct.

14. The release of radioactive materials into the environment can contaminate air, water,
soil, plants, buildings, animals, and people, leading to hazards from ionizing
radiation. Remedial measures for treating radioactively contaminated water include
aeration, reverse osmosis, ion exchange, and granule carbon adsorption.
Aeration can remove high levels of radon (222Rn) from drinking water by promoting
volatilization and biological degradation. Reverse osmosis is used to remove various

Science & Technology and Environment by Ravi Garlapati Page 62


radioactive elements, including alpha and beta particles, uranium, radium, and
photon emitters. Ion exchange involves passing water through a resin containing
exchangeable ions, and it is used for the removal of radium, uranium, alpha and beta
particles, and photon emitters. Granule carbon adsorption uses Granular activated
carbon (GAC), a highly porous adsorption media, to remove radioactive
contamination from water.

15. All three statements in the question are correct. The Plastic Waste Management
Rules (2016, amended in 2018 and 2021) have been updated to include rural areas,
with responsibility for implementation given to Gram Panchayat. Guidelines for
Extended Producer Responsibility have been given legal force.
Individuals and businesses must segregate plastic waste at the source and pay user
fees.
Recycled plastic cannot be used for food packaging, and certain single-use plastics
will be prohibited from July 1, 2022. To prevent littering, plastic carry bags' thickness
has been increased from 50 to 75 microns, and then 120 microns from December 31,
2022. To manufacture carry bags at the state level, registration from the state
pollution control board is required.

16. Statement 1 is correct because the Ministry of Environment, Forest and Climate
Change is the nodal Ministry to deal with the transboundary movement of hazardous
and other wastes.
Statement 2 is incorrect because the import of hazardous and other wastes from any
country is permitted only for recycling, recovery, reuse, and utilization, including co-
processing. Disposal of hazardous and other wastes imported from any country is not
permitted in India.

17. The BS VI emission standards apply to all major on-road vehicle categories, including
light and heavy-duty vehicles, as well as two and three-wheeled vehicles, as of April
1, 2020. Therefore, statement 1 is incorrect.
According to the BS-VI emission norms, petrol vehicles will need to reduce their
nitrogen oxide emissions by 25%, while diesel engines will need to reduce their
HC+NOx, NOx, and particulate matter emissions by varying amounts. Hence,
statement 2 is correct.
The BS VI regulation also mandates onboard diagnostics (OBD) for all vehicles to
provide information about system efficiency. Therefore, statement 3 is correct.
The Central Pollution Control Board under the Ministry of Environment, Forests, and
Climate Change is responsible for setting the standards and timelines for their
implementation, not the Ministry of Road Transport and Highways. Therefore,
statement 4 is incorrect.

Science & Technology and Environment by Ravi Garlapati Page 63


18. A catalytic converter is a device located in the exhaust system of a vehicle near the
engine. Its purpose is to reduce the emission of gaseous pollutants, such as carbon
monoxide (CO), nitrogen oxides (NOx), and hydrocarbons (HCs). It utilizes a catalyst
chamber to convert the harmful compounds (toxic gases) emitted from the engine
into safe gases, such as steam. Therefore, option (d) is the correct answer.

19. The term "Dead zone" is commonly used to refer to a condition called hypoxia, which
is characterized by reduced levels of oxygen in water bodies. Hypoxia creates areas
in oceans and lakes where marine life cannot survive, and most organisms either die
or move away from these regions. While hypoxic zones can occur naturally, human
activities such as eutrophication can enhance or create dead zones. Thus, option (c) is
the best explanation for the term "Dead zone."

20. Petrol vehicles following BS-VI norms emit 25% less nitrogen oxides and diesel
vehicles emit around 70% less than the BS-IV norms. In general, petrol vehicles emit
less nitrous oxide compared to diesel vehicles, making Statement 1 incorrect.
BS-VI fuel contains less sulphur (10 ppm) compared to BS-IV fuel (50 ppm) to reduce
air pollution. Therefore, Statement 2 is incorrect.
The Central Pollution Control Board (CPCB), which is under the Ministry of
Environment, Forest and Climate Change, sets and regulates the standards and
timeline of the BS-VI norms, making Statement 3 correct.
BS-VI norms enable the use of advanced emission control technologies such as Diesel
Particulate Filters (DPF) and Selective Catalyst Reduction (SCR) systems to reduce
Particulate Matter (PM) and Nitrogen Oxides (NOx) emissions in vehicles. Therefore,
Statement 4 is correct.

Exercise – II a) Tsunami
Topic: Disaster Management b) Storm surge
c) Flood
1. In which city the National Institute of d) Wild fire
Disaster Management is situated?
a) Kolkata b) New Delhi 4. Which declaration among the following
c) Hyderabad d) Manipur deals with Disaster Management?
a) Yokohama Declaration
2. DDMA is headed by b) Rio Declaration
a) District Magistrate c) Paris Declaration
b) Chief secretary d) Tokyo Declaration
c) BDO
d) None of the above 5. Select the correct full form of ISDR?
a) International Sustainable
3. Which one of the following is a Development Report
geological disaster?

Science & Technology and Environment by Ravi Garlapati Page 64


b) International Strategy for Disaster d) None of the above
Reduction
c) International Significant Disaster 11. The Chernobyl disaster of 1986 in
Resources Ukraine was a case of
d) Intergovernmental Strategy for a) Epidemic disaster
developing Recreation b) Toxic gas disaster
c) Nuclear disaster
6. The total flood prone area in India is d) None of the above
a) 30% b) 20%
c) 12% d) 10% 12. What is it called when a large number
of people in a community get a disease at
7. The total cyclone prone area in India is the same time?
a) 15% b) 10% a) Influx b) Black death
c) 8% d) 20% c) Epidemic d) Pandemic

8. Percentage of drought prone area in the 13. In India, Cyclone is tracked through
India is which satellite?
a) 38% b) 68% a) INSAT b) IRS
c) 45% d) None of the above c) Ocean SAT d) None of the above

9. Consider the following statements: 14. Impact of an earthquake is measured


1. Atmospheric river is a narrow and by
elongated region in the a) Modified Mercalli scale
atmosphere that carries a b) Richter scale
substantial amount of water vapor c) Seismograph
outside the tropics. d) None of the above
2. It is associated with heavy rains
and flooding. 15. About 2/3rd of the cyclones that occur
3. It is recently observed in in the Indian coastline occur in the
Meghalaya state. a) Bay of Bengal
Which of the above statements is/are b) Coastal area of south India
correct? c) Coastal area of west India
a) 2 only b) 1 and 3 only d) None of the above
c) 1 and 2 only d) 2 and 3 only
16. International Tsunami information
centre is in
10. Which is the only active volcano in a) Honolulu b) Goa
India? c) Jakarta d) Puducherry
a) Volcano Popa
b) Volcano of Barren Island 17. The most calm part of the tropical
c) Volcano Etna cyclone is
a) Limb b) Eye
Science & Technology and Environment by Ravi Garlapati Page 65
c) Periphery d) None of the above c) Governor of States
d) Chief Minister of States
18. In India Tsunami Warning centre is
located at 25. Generally the number on Richter Scale
a) Kolkata ranges between
b) Hyderabad a) 0 and 6 b) 0 and 9
c) Ahmadabad c) 1 to 5 d) 1 to 12
d) None of the above
26. The level of risk of a disaster depends
19. The word Tsunami has been derived on
from A. French word B. Latin word C. a) Nature of the hazard
Japanese word D. Greek word b) Vulnerability of the elements which are
affected
20. Which is the most landslide prone c) Economic value of the elements which
area in India A. Ganges valley region are affected
B. South Indian plateau C. Himalaya and d) All of the above
north eastern hill D. None of the above
21. In northern hemisphere, the wind of 27. The Richter scale expresses an
the tropical cyclone blows in earthquakes
a) Anticlockwise direction a) Magnitude
b) Clockwise direction b) Location
c) Straight c) Duration
d) None of the above d) Depth

22. Tropical cyclones rarely develop 28. The point of the earth’s surface
within 50 kms of the equator because directly above the point where an
coriolis effect is earthquake occurs is called the:
a) weakest there a) Focus b) Epicenter
b) moderate there c) Fracture d) Fault
c) Strongest there
d) None of the above 29. The National Disaster Response Force
(NDRF) Battalion’s regional headquarters
23. Bhopal Gas Disaster is a kind of for Andhrapradesh, Telangana, and
a) Natural disaster Karnataka is located at?
b) Manmade disaster a) Vijayawada b) Bengaluru
c) Mixed disaster c) Hyderabad d) Visakhapatnam
d) None of the above

24. The National Disaster Management 30. Who among these govern/ head the
Authority (NDMA) is headed by National Crisis Management Committee?
a) Prime Minister of India a) Cabinet Secretary
b) President of India

Science & Technology and Environment by Ravi Garlapati Page 66


b) Ministry of Home Affairs a) Wild fires b) Floods
c) Prime Minister c) Earthquake d) Chemical spill
d) Ministry of Environment
33. What was the percentage increase in
31. The State Disaster Management heat-related deaths between 2000-2004
Authority is headed by and 2017-2021 according to a recent
a) Governor Lancet Report?
b) Chief Minister a) 25% b) 36%
c) Chief Secretary of the State c) 45% d) 68%
d) None of the above

32. Which disaster struck the South


European countries have recently?

Exercise – III Key

1. b 2. a 3. a 4. a 5. b 6. c 7. c 8. b 9. c 10. b
11. c 12. c 13. b 14. c 15. b 16. a 17. b 18. b 19. c 20. c
21. a 22. a 23. b 24. a 25. b 26. d 27. a 28. b 29. a 30. a
31. b 32. a 33. d

Explanations

1. New Delhi is home to the National Institute of Disaster Management (NIDM). It was
established in 2004 to offer training and development programs for dealing with
natural and artificial catastrophes. It was established it as a deemed university and
institute of teaching under the Disaster Management Act 2005.

2. Exp. District Disaster Management Authority (DDMA) under National Disaster


Management Act, 2005 is headed by a District Magistrate.

3. Tsunami is caused by earthquakes under ocean’s beds. Hence its geological disaster.

4. The Yokohama Declaration was adopted at the First World Conference on Natural
Disaster Reduction (organized by UN), held in Yokohama, Japan, in May, 1994.
The Yokohama Declaration was a landmark document in the history of disaster
reduction. The Declaration emphasized the need for a comprehensive and integrated
approach to disaster reduction that would involve all sectors of society, including
government, business, civil society, and individuals.

Science & Technology and Environment by Ravi Garlapati Page 67


5. ISDR stands for International Strategy for Disaster Reduction in its full form. ISDR is
a worldwide framework for disaster risk reduction developed by the United Nations
in 1999. The Yokohama Declaration of 1994 provided the basis for ISDR. The ISDR
aimed to promote a culture of disaster prevention and to reduce the impact of
disasters through better risk management, preparedness, and response.

6. India's flood prone area is up to 12% of the total land area, which is around 40
million hectares. India's major flood-prone states are as follows:
 Haryana, Punjab, Bihar
 Assam, Manipur, Tripura
 Orissa, Jharkhand,West Bengal

7. India’s overall cyclone-prone region accounts for about 8% of the country’s total land
area. The eastern coast (the Bay of Bengal coastal area) and the western coast
(Arabian Sea coastal area) of India are both impacted by cyclones. More cyclones are
generated on east coast than on west coast. A tropical cyclone’s two primary seasons
are May to June and mid-September to mid-December.

8. 68% of landmass in India is vulnerable to various levels of droughts.

9. Statement 1 and 2 are correct. An atmospheric river is a relatively long and narrow
region in the atmosphere that contains a large amount of water vapor and occurs
outside the tropics. Also known as moisture plumes, tropical plumes, tropical
connections, water vapor surges, or cloud bands, they can stretch over thousands of
kilometers and carry water vapor equivalent to the average flow of water at the
Mississippi River's mouth.
Statement 3 is incorrect. California is currently preparing for the arrival of
atmospheric rivers, which are predicted to result in significant precipitation, floods,
and heavy snow.

10. India's only active volcano situated in Andaman and Nicobar is Barren Island
Volcano. It comes under the arc of active volcanoes extended between Sumatra and
Burma. This volcano had its last volcanic eruption on December 8, 2020.

11. Exp. Chernobyl disaster is the worst nuclear disaster in the world that occurred in
1986 of the then Soviet Union (now Ukrain) in a nuclear power plant.

12. If the scale of the disease is global – then it is Pandemic.

13. INSAT series of satellites are used to track the cyclones in India.

Science & Technology and Environment by Ravi Garlapati Page 68


14. The Modified Mercalli scale measures the intensity of an earthquake based on its
effects on people, structures, and the environment, and is not a direct measure of the
earthquake's energy release.
Richter scale is used to measure the magnitude of an earthquake. The Richter scale is
a logarithmic scale that measures the amount of energy released by an earthquake at
its epicenter, based on the amplitude of the seismic waves recorded by seismographs.
The higher the number on the Richter scale, the more intense the earthquake.
Seismographs are instruments used to record and measure seismic waves generated
by earthquakes.

15. About 2/3rd of the cyclones that occur in the Indian coastline occur in the Bay of
Bengal. The Bay of Bengal is known for being a hotbed of tropical cyclone activity,
with an average of five to six cyclones forming in the region each year. This is due to a
combination of factors, including warm sea surface temperatures, high humidity, and
favorable atmospheric conditions. The coastal areas of Arabian sea also experience
cyclones, but the frequency and intensity of these cyclones are lower compared to
the Bay of Bengal.

16. The International Tsunami Information Centre (ITIC) is located in Honolulu, Hawaii.
It is a part of the Intergovernmental Oceanographic Commission (IOC) of UNESCO
and serves as a focal point for the exchange of tsunami-related information and data
between countries and organizations around the world.

17. The eye is the most calm part of a tropical cyclone, which is a region of relatively light
winds and clear skies at the center of the storm. The eye is surrounded by the
eyewall, which contains the most intense winds and rain of the cyclone.

18. The Indian Tsunami Early Warning Centre (ITEWC) is located in Hyderabad, India. It
was established after the 2004 Indian Ocean earthquake and tsunami to provide real-
time tsunami warnings and advisories to India's coastal states and neighboring
countries.
19. c) Japanese word. The word tsunami is derived from the Japanese words "tsu"
(meaning harbor) and "nami" (meaning wave), reflecting the fact that tsunamis are
often generated by underwater earthquakes or volcanic eruptions that cause massive
displacements of water.

20. The Himalayan and North Eastern regions of India are the most landslide-prone
areas in the country, due to their rugged terrain, high precipitation, and seismic
activity.

21. In the northern hemisphere, the wind of the tropical cyclone blows in an
anticlockwise direction around the center of the storm, due to the Coriolis effect.

Science & Technology and Environment by Ravi Garlapati Page 69


22. Tropical cyclones rarely develop within 50kms of the equator because the Coriolis
effect, which causes the cyclone to spin, is weakest at the equator and increases with
distance from it.

23. The Bhopal Gas Disaster was a man-made disaster that occurred on December 2-3,
1984, in Bhopal, India. It was caused by the release of toxic gas (Methyl Isocyanate)
from the Union Carbide India Limited (UCIL) pesticide plant, resulting in thousands
of deaths and injuries.

24. The National Disaster Management Authority (NDMA) is an agency of the


Government of India, responsible for coordinating the national response to natural
and man-made disasters. It is headed by the Prime Minister of India, who serves as
the Chairperson of the agency.

25. The Richter scale is a logarithmic scale that measures the magnitude or energy
released by an earthquake. The scale typically ranges from 0 to 9, with each whole
number increase representing a tenfold increase in the energy released.

26. The level of risk of a disaster depends on multiple factors, including the nature of the
hazard, the vulnerability of the elements that are affected, and the economic value of
those elements. All of these factors can influence the level of risk.

27. The Richter scale expresses an earthquake's magnitude, which is a measure of the
amount of energy released by an earthquake.

28. The point on the earth's surface directly above the point where an earthquake
occurs is called the epicenter. The focus is the actual point underground where the
earthquake originates.

29. NDRF was established in the year 2006 and comes under Home Ministry. It’s
headquarters is in New Delhi. It currently has 16 battalions that are spread
throughout the country.

30. The Cabinet Secretary heads National Crisis Management Committee. Cabinet
Secretary is the highest executive officer, and all the secretaries of the departments
are members of the committee.

31. State Disaster Management Authority (SDMA) under National Disaster Management
Act, 2005 is headed by a Chief Minister.

32. Several Southern European nations such as Greece, Spain and France have been
experiencing severe heatwaves and wildfires. The increasing temperatures
attributed to climate change have caused numerous fatalities. To prevent further

Science & Technology and Environment by Ravi Garlapati Page 70


danger, France has evacuated over 16,000 people from the south-west region, while
wildfires have continued to spread in Spain, Greece and Croatia.

33. According to a recent Lancet report, heat-related deaths increased by 68 per cent
between 2000-2004 and 2017-2021. The report also highlighted that vulnerable
populations were exposed to 3.7 billion more heatwave days last year than annually
in 1986-2005. The study focused on the health effects of climate change in the midst
of the Covid pandemic, global energy crisis, and the cost of living crisis due to the
Russia-Ukraine war.

Science & Technology and Environment by Ravi Garlapati Page 71


MCQs from
General Science

Science & Technology and Environment by Ravi Garlapati Page 72


Biology MCQs
Exercise - I 3. Protein digestion in the intestine
1. Regarding the immune system of the produces ammonia, which is
human body, consider the following converted into urea in the liver.
statements: Which of the statements given above
1. Leucocytes or white blood cells are is/are correct?
irregularly shaped, nucleated cells (a) 1 only (b) 2 and 3 only
that lack hemoglobin and are (c) 1 and 2 only (d) 1, 2 and 3
colorless.
2. White blood cells have a shorter 4. Erythroblastosis Foetalis is a
life span than red blood cells. condition where the second baby of a
3. Monocytes are primarily involved couple may die in the womb of the
in phagocytosis and degradation of mother due to Rhesus (Rh)
necrotic material in the human incompatibility. Which of the
body. following blood type combinations of
Which of the statements given above parents is most likely?
is/are not correct? (a) Father having Rh+ and Mother
(a) None (b) 1 and 2 only having Rh-
(c) 2 and 3 only (d) 1, 2 and 3 (b) Father having Rh+ and Mother
having Rh+
2. Numerous variants of Influenza A (c) Father having Rh- and Mother
virus have been identified thus far. having Rh-
What could be the potential causes for (d) Father having Rh- and Mother
the existence of such a large number having Rh+
of strains of this virus?
1. Influenza viruses have the ability 5. This organ has various supportive
to exchange genetic material via functions within the human body. It
reassortment. serves as an immune system filter for
2. It is a non-segmented DNA virus. blood, recycles old red blood cells, and
3. It is prone to frequent mutations. stores platelets and white blood cells.
Which of the above statements is/are In addition, it aids in the fight against
correct? certain types of bacteria responsible
(a) 1 only (b) 1 and 3 only for pneumonia and meningitis.
(c) 2 and 3 only (d) 1, 2 and 3 only Although it can differ in size and shape
among individuals, it is typically
3. Consider the following statements shaped like a fist.
about excretory organs: Which organ is being described in the
1. The skin is one of the excretory passage?
organs in the human body. (a) Liver (b) Heart
2. The adrenal glands are located on (c) Spleen (d) Pancreas
the top of the kidneys.
6. Which of the statements below
accurately describe the human brain?

Science & Technology and Environment by Ravi Garlapati Page 73


1. The human brain is the largest (a) 1, 2 and 3 only (b) 1 and 2 only
brain in relation to body size when (c) 2 and 3 only (d) 3 only
compared to all other vertebrates.
2. The occipital lobe, located at the 9. Which of these disease groups are all
back of the skull, is responsible for attributed to viruses?
visual processing. (a) Common cold, influenza, typhoid,
3. Sound and language are processed cholera
by the temporal lobe. (b) Influenza, AIDS, kala-azar, chicken
(a) 1 only (b) 1 and 2 only pox
(c) 2 and 3 only (d) 1, 2 and 3 only (c) Kala-azar, tuberculosis, anthrax,
small pox
7. Consider the following statements (d) Influenza, dengue, chikungunya,
relating to the human eye: polio
1. The sclera, which is the outermost
layer of the eye, provides its 10. Which of the following illnesses or
predominantly white color. infections are instances of zoonotic
2. During eye transplantation infections?
procedures, only the retina and iris 1. West Nile virus
are typically transplanted. 2. Brucellosis
3. The iris is a dark, muscular 3. Middle East Respiratory Syndrome
diaphragm that regulates the size 4. Nipah virus
of the pupil. 5. Blastomycosis
4. The cornea is primarily Select the correct answer using the
responsible for refracting or code given below:
bending light in the human eye. (a) 1, 2, 3 and 4 only
Which of the above statements are (b) 1, 3 and 4 only
correct? (c) 2, 4 and 5 only
(a) 1, 2 and 4 only (d) 1, 2, 3, 4 and 5
(b) 1, 3 and 4 only
(c) 1, 2 and 3 only 11. Which of the statements below is
(d) 1, 2, 3 and 4 inaccurate regarding disease-causing
microorganisms?
8. Consider the following statements: (a) Infection refers to the invasion and
1. Deoxyribonucleic acid (DNA) is proliferation of harmful
exclusively located within the microorganisms in an individual.
nucleus of a human cell. (b) Parasites include both protozoa
2. Defective genes on the X and worms.
chromosome within human cells (c) A virus is unable to reproduce
can lead to mitochondrial diseases. outside of a host cell.
3. In humans, mitochondrial DNA is (d) In order to survive for an extended
solely passed down from the period, a pathogen must quickly kill
mother. every host it infects.
Which of the above statements is/are
correct?
Science & Technology and Environment by Ravi Garlapati Page 74
12. Which of the following statements 15. Which of the following statements
accurately defines "Flavonoids"? regarding a bacteriophage is/are
(a) They are a group of pulsars that correct?
emit gamma-ray radiation. 1. It is a virus that can integrate its
(b) They are malicious software that own DNA into the DNA of another
aim to corrupt a host device's hard bacterium.
drive. 2. It hijacks the host bacterium's
(c) They are a set of plant metabolites cellular machinery to produce the
that provide health advantages to the bacteriophage’s own proteins.
human body. 3. Bacteriophages cannot infect and
(d) They are a kind of cyanobacteria replicate in human cells.
that can endure harsh environmental (a) 1 and 2 only (b) 2 only
conditions. (c) 2 and 3 only (d) 1, 2 and 3

13. Candida Auris, a multidrug-resistant 16. Regarding Plant and Animal cells,
pathogen that poses a severe which of the following statements
worldwide danger to human health, is/are correct?
was recently discovered for the first
1. Centrioles are typically present in
time in the environment of which
animal cells but generally absent
region of India?
in plant cells.
(a) Western Himalayas
2. Carbohydrates are stored in plants
(b) Hills of Southern India
in the form of starch and in
(c) Andaman and Nicobar Islands
animals in the form of glycogen.
(d) Tropical rain forests of Northeast
(a) 1 only (b) 2 only
India
(c) Both 1 and 2 (d) Neither 1
14. The state of Odisha has recently nor 2
reported its initial occurrence of
'Harlequin ichthyosis.' Evaluate the 17. Consider the following statements
following statements concerning the about Biomolecules :
illness: 1. Carbohydrates are linear chains
1. It is transferred from one connected by peptide bonds
individual to another by the same 2. Insulin in human body is a fibrous
mosquito that transmits dengue. protein
2. It predominantly affects the 3. Triglycerides is a type of fat found
growth of the skin in humans. in butter
Which of the statements given above Select the correct answer using the
is/are correct? code given below :
(a) 1 only (b) 2 only (a) 1 and 2 only (b) 2 and 3 only
(c) Both 1 and 2 (d) Neither 1 (c) 3 only (d) 1, 2 and 3
nor 2
18. Consider the following statements:

Science & Technology and Environment by Ravi Garlapati Page 75


1. Eukaryotic cell has a free-floating
genetic material with the absence of a 20. With reference to DNA, consider the
nucleus. following statements.
2. Prokaryotes can live with or 1. DNA, which stands for
without oxygen while eukaryotes live Deoxyribonucleic Acid, is a
only with oxygen. hereditary material found in all
Which of the statements given above living things.
is/are correct ? 2. While most DNA is located in the
(a) 1 only (b) 2 only cell nucleus, also known as nuclear
(c) Both 1 and 2 (d) Neither 1 DNA, a small amount of DNA can
nor 2 be found in mitochondria, also
known as mitochondrial DNA.
19. Which of the following structures are 3. Viruses can have either DNA or
present in animal cells? RNA as their genetic material, but
1. Cell Wall some viruses only have RNA.
2. Mitochondria Which of the above statements is/are
3. Ribosomes correct?
Which of the above statements is/are (a) 1 only (b) 1 and 2 only
correct? (c) 2 and 3 only (d) 1, 2 and 3 only
(a) 1 only (b) 1 and 2 only
(c) 2 and 3 only (d) 1, 2 and 3 only

Exercise – I Key

1. a 2. b 3. d 4. a 5. c 6. d 7. b 8. d 9. d 10. a
11. d 12. c 13. c 14. b 15. d 16. c 17. c 18. d 19. c 20. d

Explanations

1. White blood cells, also known as leukocytes, are colorless because they lack
hemoglobin, and they are irregularly shaped, nucleated cells. They are heterogeneous
in nature, and the normal concentration of white blood cells in the blood varies from
4,000 to 10,000 per microliter. Therefore, statement 1 is correct.
Red blood cells, on the other hand, have a biconcave disc structure. Leucocytes are
generally short-lived, that is for about 13 to 20 days. But the RBCs have an average
life span of 120 days after which they are destroyed in the spleen (the graveyard of
RBCs). So, Statement 2 is correct.
The two main categories of white blood cells are granulocytes and agranulocytes.
Neutrophils, eosinophils, and basophils are different types of granulocytes, while
lymphocytes and monocytes are agranulocytes. Monocytes are involved in
phagocytosis and degradation of necrotic material, while eosinophils are associated
with allergic and hypersensitivity reactions. Therefore, statement 3 is correct.

Science & Technology and Environment by Ravi Garlapati Page 76


Therefore, the correct answer is (b) 1 and 2 only.

2. The first and third statements are correct. The presence of numerous strains of
Influenza A virus is due to its constant mutation. One of the causes for the virus's
constant mutation is the process of reassortment, where influenza viruses swap gene
segments. This genetic exchange is possible because of the segmented nature of the
viral genome, and it happens when two different influenza viruses infect a single cell.
Reassortment is akin to viral sex. The second statement is incorrect. Influenza A virus
does not have an unsegmented DNA genome, as it is an RNA virus with a segmented
genome, comprising eight distinct strands. This makes its replication prone to errors
or mutations, resulting in what is known as "antigenic drift." This process leads to
minor but ongoing mutations in surface proteins, necessitating the frequent updating
of flu vaccines.

3. The skin serves as an excretory organ in the human body, as it helps to eliminate
some harmful elements and waste through sweat. Therefore, Statement 1 is correct.
The adrenal glands play a crucial role in the body's response to stress, producing
hormones that trigger physiological adaptations (body's fight-or-flight response) to
changes in the external environment. These glands are positioned on the top of each
kidney. Hence, Statement 2 is correct.
During protein digestion in the intestine, ammonia (NH3) is produced by bacteria.
This ammonia is then processed in the liver, where it is converted into urea, which is
a waste product that is eventually excreted in urine by the kidneys. Therefore,
Statement 3 is correct.

4. The Rh factor is a molecule present on the surface of red blood cells in some
individuals. Blood is considered Rh-positive if red blood cells have the Rh factor, and
Rh-negative if they do not.
An Rh-negative person, when exposed to Rh-positive blood, may develop specific
antibodies against the Rh antigens. Therefore, Rh grouping should be matched during
blood transfusions. A special case of Rh incompatibility occurs between the Rh-
negative blood of a pregnant mother and the Rh-positive blood of the fetus.
Rh antigens of the fetus do not get exposed to the Rh-negative blood of the mother
during the first pregnancy, as the two types of blood are well separated by the
placenta. However, during the delivery of the first child, there is a possibility of the
mother's blood being exposed to small amounts of the Rh-positive blood from the
fetus. In such cases, the mother may start producing antibodies against Rh antigens
in her blood.
In subsequent pregnancies, the Rh antibodies from the Rh-negative mother can cross
into the blood of the Rh-positive fetus and destroy the fetal red blood cells. This can
be fatal to the fetus or cause severe anemia and jaundice in the baby, a condition
known as erythroblastosis fetalis. This can be prevented by administering anti-Rh
antibodies to the mother immediately after the delivery of the first child.

Science & Technology and Environment by Ravi Garlapati Page 77


Erythroblastosis fetalis typically occurs when a woman with Rh-negative blood is
impregnated by a man with Rh-positive blood, and conceives a fetus with Rh-positive
blood. Therefore, option (a) is correct.

5. The lymphatic system's most massive organ is the spleen. It plays a crucial role in
maintaining the body's fluid balance, but one can survive without it. Positioned in the
left upper quadrant of the
abdomen, below the ribcage and
above the stomach, it acts as a
blood purifier. The spleen regulates
the number of red blood cells and
blood storage and assists in
combating infections. If the spleen
recognizes potentially harmful
bacteria, viruses, or other
microorganisms in the
bloodstream, it produces white
blood cells known as lymphocytes,
along with the lymph nodes. These
cells serve as defenders against intruders, generating antibodies that destroy foreign
microorganisms and prevent infections from spreading.

6. Statement 1 is correct. The human brain is larger in relation to body size than any
other mammal brains, and weighs about 1.5 kilograms, making up about 2 percent of
a human's body weight. The cerebrum makes up 85 percent of the brain's weight and
contains about 86 billion neurons and billions of nerve fibers. The cerebral cortex is
greatly enlarged in human brains and is considered the seat of complex thought. The
brain is divided into three sections: forebrain, midbrain, and hindbrain. The largest
part of the human brain is the cerebrum, which is divided into two hemispheres.
Statements 2 and 3 are correct. The occipital lobe processes visual information, the
temporal lobe processes sound and language, and the parietal lobe integrates input
from different senses and is important for spatial orientation and navigation.

7. Statement 1 is correct. The sclera, which is the white part of the eye surrounding the
cornea, covers more than 80% of the eyeball's surface area. In individuals with liver
failure, the sclera begins to turn yellow, a condition known as jaundice.
Statement 2 is incorrect. During an eye transplant, only the cornea is transplanted.
The retina is a thin layer of tissue that lines the back of the eye and converts light into
neural signals for visual recognition by the brain.
Statement 3 is correct. The iris, which is the colored part of the eye, helps regulate
the amount of light entering the eye. In bright light, the iris contracts to reduce the
amount of light that enters the eye, thereby protecting the retina.

Science & Technology and Environment by Ravi Garlapati Page 78


Statement 4 is correct. The cornea is the eye's outermost lens and serves as a
window that focuses and controls the entry of light into the eye. By focusing light
onto the retina, a clear image of the object is formed.

8. Statement 1 is incorrect. While most of the DNA is found in the cell nucleus, some
DNA is also present in the mitochondria, which is called mitochondrial DNA
(mtDNA).
Statement 2 is incorrect. Mitochondrial diseases are caused by faulty mitochondrial
DNA, not faulty genes on the X-chromosome. Certain disorders caused by mtDNA
dysfunction include diabetes, respiratory disorders, Huntington's disease,
Parkinson's disease, Alzheimer's disease, etc. Currently, there is no cure for
mitochondrial diseases.
Statement 3 is correct. Mitochondria are structures in each cell that convert
molecules into energy, and each contains a small amount of DNA. Since only egg cells
contribute mitochondria to the developing embryo, only females can pass on
mitochondrial mutations to their children.

9. Option d is correct. Viruses can cause various diseases, including the common cold,
influenza, dengue fever, AIDS, chickenpox, smallpox, Chikungunya, Ebola, polio, and
many others.

10. Zoonotic diseases are diseases transmitted from animals to humans. The following
statements are correct:
West Nile fever is a zoonosis caused by a virus transmitted by infected mosquitoes;
Brucellosis is an infectious disease caused by bacteria and people can contract it
through contact with infected animals or their products;
Middle East Respiratory Syndrome (MERS) is a virus transmitted between animals
and people;
Nipah is an example of a zoonotic disease caused by a virus transmitted from animals
to humans;
Blastomycosis, however, is not a zoonotic disease, as it is caused by a fungus found in
the environment, and not spread between people or animals.

11. Statement a is correct. An infection occurs when pathogenic microorganisms invade


and multiply in an individual or population, while a disease occurs when the
infection causes damage to the individual's vital functions or systems. Not all
infections result in disease.
Statement b is correct. The agents that cause disease can be classified into five
groups: viruses, bacteria, fungi, protozoa, and helminths (worms). Protozoa and
worms are commonly grouped together as parasites and are the subject of
parasitology.
Statement c is correct. Intracellular pathogens need to invade host cells to replicate.
These pathogens can be further divided into those that replicate freely in the cell,

Science & Technology and Environment by Ravi Garlapati Page 79


such as viruses and some bacteria, and those that replicate in cellular vesicles, such
as mycobacteria. Because viruses are obligate intracellular pathogens, they require
the machinery and metabolism of a host cell to replicate.
Statement d is incorrect. It is not advantageous for the long-term survival of a
pathogen to rapidly kill every host it infects or be wiped out by the immune response
before infecting another individual. Therefore, killing every host it infects is not
beneficial for the pathogen's long-term survival.

12. Flavonoids are plant metabolites that are found in various fruits and vegetables and
are believed to offer health benefits by means of cell signalling pathways and
antioxidant effects. They are important antioxidants and are associated with several
positive effects on health, including anti-viral, anti-cancer, anti-inflammatory, and
anti-allergic properties. Flavonoids are present in most fruits, vegetables, and herbs,
and can also be found in other food sources such as dry beans, grains, red wine, green
tea, and black tea.

13. Candida Auris, also known as C. auris, is a pathogen that is resistant to multiple drugs
and is considered a serious threat to global health. Recently, it has been identified in
the environment off the coast of the South Andaman district in the Andaman and
Nicobar Islands, marking the first time it has been found in the environment. Candida
Auris was initially discovered in a patient in Japan in 2009, and has since emerged in
over 40 countries on five continents over the past decade. While this fungus is mostly
found in tropical marshes and marine environments, it has also been detected
outside of hospital settings.

14. Statement 1 is incorrect because Harlequin ichthyosis is a genetic disorder caused by


mutations in the ABCA12 gene inherited from parents. The ABCA12 protein plays a
crucial role in transporting fats in cells that make up the outermost layer of skin.
Statement 2 is correct. Harlequin ichthyosis is a severe genetic disorder that affects
the skin, creating large diamond-shaped plates across the body that are separated by
deep cracks (fissures). The skin becomes dry and scaly, resembling fish skin, which is
why the term "ichthyosis" is derived from "ikthus," the Greek word for fish. Severe
mutations in the ABCA12 gene lead to the absence or partial production of the
ABCA12 protein, which results in the lack of lipid transport and affects skin
development to varying degrees, depending on the severity of the mutation.

15. A bacteriophage is a type of virus that infects bacteria, and the term "bacteriophage"
means "bacteria eater." Bacteriophages are composed of a small piece of DNA
surrounded by a protein coat and are not bacteria themselves. Therefore, Statement
1 is not correct.
After infecting a bacterium, the bacteriophage takes over the host bacterium's
cellular machinery to prevent it from producing bacterial components and instead
forces the cell to produce viral components. So, Statement 2 is correct.

Science & Technology and Environment by Ravi Garlapati Page 80


Bacteriophages are specific to bacterial cells and cannot infect and replicate in
human cells. Although bacteriophages are an important part of the human
microbiome, their ability to replicate in human cells is yet to be found, and research
is ongoing. So, Statement 3 is correct.

16. Animal cells possess centrioles, which are barrel-shaped organelles found near the
nuclear envelope in the cytoplasm. Centrioles play a key role in organizing
microtubules, which serve as the cell's skeletal system. In contrast, plant cells have
cell walls, plastids, and a large central vacuole that are absent in animal cells, but
they lack centrioles, which are absent in almost all plant cells. Therefore, statement 1
is correct.
Carbohydrates are a major source of
energy for both animals and plants. Plants
synthesize carbohydrates using light
energy from the Sun during the process of
photosynthesis. In contrast, animals
consume plants or other animals to obtain
carbohydrates. Plants store carbohydrates
as starch, while animals store them as
glycogen. Hence, statement 2 is correct.

17. Proteins are made up of amino acids, which are linked by peptide bonds.
Carbohydrates are macromolecules composed of carbon, hydrogen, and oxygen with
a 2:1 ratio of Hydrogen to Oxygen atoms, and they are linked by glycosidic bonds.
Therefore, Statement 1 is not correct.
Fibrous proteins, such as keratin and myosin, have a fiber-like structure due to
hydrogen and disulfide bonds and are generally insoluble in water. Globular proteins,
such as insulin, are more soluble and have a spherical shape. Therefore, Statement 2
is not correct.
Triglycerides, also known as neutral fats, are essential for energy storage and are
mainly found in vegetable oils and animal fats. Therefore, Statement 3 is correct.

18. The term "Eukaryotic" means: eu = true, karyon = nucleus, as they possess a well-
organized nucleus enclosed by a nuclear envelope. The DNA is enclosed in a nuclear
membrane, forming a nucleus. Therefore, Statement 1 is incorrect.
Prokaryotes and eukaryotes are capable of performing both aerobic (oxygen-
requiring) and anaerobic (non-oxygen-based) metabolism. As a result, both types of
organisms can live with or without oxygen. Hence, Statement 2 is also incorrect.

19. Plant cells possess both a cell wall and a cell membrane, with the cell wall
surrounding the cell membrane, resulting in a unique rectangular shape. Animal cells,
on the other hand, only possess a cell membrane and lack a cell wall. Mitochondria
and Ribosomes are present in both plant and animal cells. Mitochondria are

Science & Technology and Environment by Ravi Garlapati Page 81


organelles bound by a membrane that generate the majority of the chemical energy
required to fuel cellular reactions. This chemical energy is stored in adenosine
triphosphate (ATP), a small molecule. Ribosomes are composed of RNA and
associated proteins and are responsible for protein synthesis, which is necessary for
various cellular functions like chemical processes and repairing damage. Ribosomes
can be found floating in the cytoplasm or attached to the endoplasmic reticulum.

20. Statement 1 is correct in stating that DNA is the chemical compound that carries
genetic instructions in all living things. This molecule is composed of two strands that
twist around one another to form a structure known as a double helix. Each strand
contains alternating sugar (deoxyribose) and phosphate groups. DNA is made up of
23 pairs of chromosomes and provides information for constructing an entire
organism and its proteins. The DNA code is composed of four chemical bases:
adenine (A), guanine (G), cytosine (C), and thymine (T).
The second statement is also correct. Nuclear DNA can be inherited from both
parents, whereas mitochondrial DNA is only inherited from the mother.
Furthermore, nuclear DNA has a lower mutation rate than mitochondrial DNA.
Statement 3 is correct. Most viruses contain either RNA or DNA as their genetic
material. The nucleic acid can be single- or double-stranded. The entire infectious
viral particle, called a virion, is made up of the nucleic acid and an outer protein shell.
The simplest viruses only encode enough RNA or DNA to produce four proteins.

Exercise - II 2. Saturated fat is present in the


1. Which of the following statements majority of animal-based foods.
correctly defines the term C-peptide? 3. Trans fats, a type of saturated fat,
(a) A substance that forms in human are linked to a higher risk of heart
blood samples containing antibodies attacks.
against hepatitis B virus. Which of the above statements is/are
(b) A byproduct that is generated correct?
during the production and secretion of (a) 2 only (b) 1 and 3 only
the hormone insulin in the body. (c) 1 and 2 only (d)1, 2 and 3 only
(c) A monoclonal antibody utilized to
treat mild to moderate cases of 3. When discussing the origins of life on
COVID-19. Earth, the following statements should
(d) A substance employed in be taken into consideration:
modifying a gene product. 1. Prokaryotes, a type of single-celled
organism, were the first life forms
2. Consider the below statements about to emerge on Earth.
various types of fats: 2. The second atmosphere of Earth
1. Unsaturated fats, which are was formed due to volcanic
predominantly present in plants, eruptions that occurred in its early
are considered healthy for the stages.
body. 3. Blue-green algae played a role in
reducing the levels of carbon
Science & Technology and Environment by Ravi Garlapati Page 82
dioxide and increasing the amount (c) A totipotent cell has the capability
of oxygen in the atmosphere. of differentiating into any cell type.
Which of the above statements is/are (d) Pluripotent cells can differentiate
correct? into a placenta.
(a) 2 only (b) 1 and 3 only
(c) 1 and 2 only (d) 1, 2 and 3 only 7. Which of the following statements
about disease-causing
4. Consider the following statements microorganisms is incorrect?
about avian influenza: (a) Infection refers to the introduction
1. H1N1 is the predominant strain of and proliferation of harmful
avian influenza. microorganisms within an organism.
2. The transmission of avian (b) Protozoa and worms are both
influenza virus from one human to classified as parasites and can cause
another is very rare. disease in humans.
Which of the statements given above (c) Viruses require a host cell to
is/are correct? replicate and cannot do so on their
(a) 1 only (b) 2 only own.
(c) Both 1 and 2 (d) Neither 1 (d) A pathogen that intends to survive
nor 2 for a long period of time must
necessarily kill its host quickly.
5. Which of the following statements
about stem cells is incorrect? 8. Consider the following statements
(a) Embryonic stem cells are capable about the distinction between
of differentiating into any type of cell Phenotype and Genotype of an
in the body. organism:
(b) Stem cells in the bone marrow 1. Phenotype denotes the observable
have the potential to generate red traits of an organism, while
blood cells, white blood cells, and Genotype refers to its genetic
platelets. makeup.
(c) Somatic stem cells persist in our 2. While Phenotype can be inherited
bodies throughout our lives. to some extent, Genotype itself is
(d) Induced Pluripotent stem cells are not transmitted during the process
generated by genetically altering of reproduction.
embryonic stem cells to produce Which of the statements given above
adult cells. is/are correct?
(a) 1 only (b) 2 only
6. Which of the following statements (c) Both 1 and 2 (d) Neither 1 nor 2
about cell potency is correct?
(a) Pluripotency is only present in the 9. Which of the following statements
early cells of fertilized eggs. about Chlorophyll is incorrect?
(b) Totipotent cells are spores in (a) Chlorophyll is found in all types of
sexual reproduction and zygotes in algae.
asexual reproduction. (b) Chlorophyll can be present in
leaves of the plant that are not green.

Science & Technology and Environment by Ravi Garlapati Page 83


(c) The process of Carbon fertilization 2. Lymphocytes
can stimulate the production of 3. Erythrocytes
chlorophyll in plants. 4. Basophils
(d) Chlorophyll is located in the Which of the above statements are
cytoplasm of plant cells. correct?
(a) 1, 2 and 4 only (b) 1, 3 and 4 only
10. Consider the following statements (c) 1, 2 and 3 only (d) 1, 2, 3 and 4
describing different processes related
to food: 13. Consider the following statements
1. Ingestion refers to the breakdown about antimicrobial resistance:
of complex food molecules into 1. Poor water quality and sanitation
simpler components. can contribute to antimicrobial
2. Egestion is the process of taking resistance.
food into the body. 2. Antimicrobial resistance can
3. Digestion is the process of reduce our ability to treat common
breaking down food molecules infections.
into absorbable nutrients in the 3. Antimicrobial resistance can affect
digestive tract. people of all ages, although the
Which of the above statements is/are elderly may be more vulnerable.
correct? Which of the above statements is/are
(a) 3 only (b) 1 and 3 only correct?
(c) 1 and 2 only (d) 2 and 3 only (a) 1 only (b) 1 and 2 only
(c) 2 and 3 only (d) 1, 2 and 3 only
11. Consider the following statements
pertaining to African Swine Fever 14. Identify the correct statement
(ASF): regarding a helminth infection, which
1. ASF is a serious viral illness that was recently in the news:
impacts both domesticated and (a) The infection is caused by
wild pigs. sneezing.
2. ASF is a type of zoonotic disease, (b) There are currently no active cases
but it does not pose a lethal threat of the infection in India.
to humans. (c) Infected children may experience
3. There have not been any reported nutritional and physical impairment.
instances of ASF outbreaks in (d) There are no effective medicines to
India. control the disease at present.
Which of the above statements is/are
correct? 15. Consider the accuracy of the following
(a) 1 only (b) 1 and 3 only statements regarding human organs:
(c) 1 and 2 only (d)1, 2 and 3 only 1. The heart is the largest gland in
the human body.
12. Which of the following options 2. The gall bladder releases bile in
represent types of white blood cells the human body.
(WBCs)? 3. The pancreas is a dual-function
1. Neutrophils organ serving both exocrine and

Science & Technology and Environment by Ravi Garlapati Page 84


endocrine functions in the human 1. In contrast to DNA, RNA is capable
body. of undergoing mutations.
4. The liver is the only organ in the 2. In terms of chemistry, DNA is less
human body capable of self- reactive and more structurally
regeneration. stable compared to RNA.
Which of the above statements are Which of the statements given above
correct? is/are correct?
(a) 1, 2 and 4 only (b) 2, 3 and 4 only (a) 1 only (b) 2 only
(c) 1, 2 and 3 only (d) 1, 2, 3 and 4 (c) Both 1 and 2 (d) Neither 1
nor 2
16. Which of the following statements
regarding Vitamin A is/are correct? 19. The following statements are about
1. Vitamin A is a fat-soluble vitamin Kala-Azar. Choose the correct
primarily found in dairy foods. statement(s):
2. Night blindness, a condition 1. Kala-Azar is a neglected tropical
affecting the retina, is an early sign disease (NTD).
of Vitamin A deficiency. 2. The Western and Southern regions
3. Vitamin A deficiency can weaken of India are the areas where it is
the immune system and increase mostly prevalent.
vulnerability to diseases such as 3. Infected female sand-flies transmit
COVID-19. it to humans.
(a) 1 only (b) 1 and 2 only Which of the above statements is/are
(c) 2 and 3 only (d) 1, 2 and 3 only correct?
(a) 1 only (b) 1 and 2 only
17. Which of the following microbes are (c) 1 and 3 only (d) 1, 2 and 3 only
useful for human beings?
1. Lactobacillus 20. Which of the following diseases are
2. Aspergillus niger caused by viruses?
3. Streptococcus Thermophilus 1. Hepatitis C
Select the correct answer using the 2. Chickenpox
code given below. 3. Common cold
(a) 1 and 2 only (b) 2 and 3 only 4. Malaria
(c) 1 and 3 only (d) 1, 2 and 3 Choose the correct answer from options
below.
(a) 1, 2 and 4 only (b) 1, 3 and 4 only
18. Regarding human genetics, what are
(c) 1, 2 and 3 only (d) 1, 2, 3 and 4
the differences between RNA and
DNA?

Exercise – II Key

1. b 2. c 3. d 4. b 5. d 6. c 7. d 8. a 9. d 10. a
11. a 12. a 13. d 14. c 15. b 16. d 17. d 18. b 19. c 20. c

Science & Technology and Environment by Ravi Garlapati Page 85


Explanations

1. C-peptide is a substance that is produced when insulin is synthesized and released in


the body. The amount of C-peptide in blood is an indicator of the amount of insulin
being produced. In mammals, including humans, insulin is synthesized as a
prohormone, which contains an additional stretch called the C peptide. This C
peptide is absent in the mature insulin, as it is removed during maturation. Typically,
high levels of C-peptide in the blood indicate high levels of insulin production and
vice versa.

2. Statement 1 is correct. Beneficial unsaturated fats are liquid at room temperature


and are mainly found in plant-based foods, including vegetable oils, nuts, and seeds.
They can help improve blood cholesterol levels, reduce inflammation, stabilize heart
rhythms, and provide other benefits.
Statement 2 is correct. Saturated fat is solid at room temperature and is mostly found
in animal-based foods, such as milk, cheese, and meat. Poultry and fish contain less
saturated fat than red meat. Tropical oils like coconut oil, palm oil, and cocoa butter
are also high in saturated fat.
Statement 3 is incorrect. Trans-fats or trans-fatty acids (TFAs) are unsaturated fatty
acids that can come from natural or industrial sources. Consumption of TFAs is
linked to a higher risk of heart attacks and death from coronary heart disease. It can
increase the levels of bad (LDL) cholesterol and decrease the levels of good (HDL)
cholesterol. TFAs are also associated with an increased risk of developing type 2
diabetes, and they have negative effects on the brain and nervous system.

3. Statement 1 is correct. Prokaryotes were the earliest life forms on Earth, which used
carbon compounds to generate their own energy. Other organisms slowly evolved to
use the Sun's energy and other compounds to generate energy.
Statement 2 is correct. Early volcanic eruptions led to the formation of Earth's second
atmosphere which led to modern atmosphere. Volcanoes released gases like carbon
dioxide into the air. They also released water vapor that cooled to form the oceans.
Volcanoes are also responsible for creating land, which is essential for many life
forms.
Statement 3 is correct. Photosynthetic organisms, such as blue-green algae,
converted atmospheric carbon dioxide into marine sediments, reducing the carbon
dioxide content of the atmosphere and producing oxygen. The production of oxygen
resulted in the formation of the ozone UV shield as a by-product.

4. Statement 1 is incorrect. Avian influenza or bird flu is a viral infection that is


primarily found in birds, but it can also infect humans and other animals. The H5N1
strain is the most common cause of severe respiratory disease in birds, but other
strains such as H7 and H8 can also cause infection. Swine flu is caused by the H1N1
virus strain.

Science & Technology and Environment by Ravi Garlapati Page 86


Statement 2 is correct. Most cases of H5N1 infection in humans have been linked to
close contact with infected live or dead birds or environments contaminated with
H5N1. Although the virus is not highly infectious to humans, there have been rare
instances of human-to-human transmission.

5. Option a is correct. Stem cells are undifferentiated cells that can differentiate into
specialized cell types and form every organ and tissue in the body. There are
different types of stem cells, including embryonic stem cells, which are pluripotent
and can give rise to any cell type in the body, and tissue-specific or adult stem cells,
which generate cells specific to the tissue or organ in which they are found.
Option b is correct. Blood-forming stem cells in the bone marrow can produce red
blood cells, white blood cells, and platelets.
Option c is correct. Adult or somatic stem cells are tissue-specific and can generate
different cell types for the specific tissue or organ in which they live. They replace
lost cells due to normal repair, disease, or injury and are found in various tissues
throughout life, such as the umbilical cord, placenta, bone marrow, muscle, brain, fat
tissue, skin, and gut.
Option d is incorrect. Induced Pluripotent stem cells are produced in the lab by
converting adult cells (tissue-specific) into cells that behave like embryonic stem
cells.

6. Option a is incorrect. It is not pluripotency that is present in the early stages of the
fertized eggs but totipotency.
Option b is incorrect as zygotes are produced during sexual reproduction and spores
are the result of asexual reproduction.
Option c is correct. A single totipotent cell has the potential to develop into an entire
organism, including all the specialized cells and the placental support structure
needed for fetal development.
Option d is incorrect. Pluripotency refers to the ability of a cell to differentiate into
the three primary germ cell layers of the early embryo and therefore into all cells of
the adult body, but not into extra-embryonic tissues like the placenta.

Totipotent Pluripotent Multi-potent

Relative High Medium Low


Potency
Cell types Differentiate Differentiate into Differentiate into
capable of into any cells a limited range of
generating cell type from any of the cell
three types
germ layers
Terminology Toti = Whole Pluri = Many Multi = Several
Found Early cells of Inner mass cells of In many tissues
fertilised egg the blastocyst

Science & Technology and Environment by Ravi Garlapati Page 87


7. Option a is correct. Microorganisms of different types cause different diseases.
Infection refers to the entry and proliferation of disease-causing microorganisms in
an individual or population. Disease occurs when the infection causes harm to the
vital functions or systems of an individual. However, an infection may not always
lead to disease.
Option b is correct. The agents responsible for causing diseases can be classified into
five categories: viruses, bacteria, fungi, protozoa, and helminths (worms). Protozoa
and worms are usually studied together as parasites, and this field of study is called
parasitology.
Option c is correct. Intracellular pathogens are those that need to invade host cells to
reproduce. Such pathogens can be divided further into those that freely reproduce in
the cell, such as certain bacteria and viruses, and those, such as mycobacteria, that
replicate in cellular vesicles.
Option d is incorrect. Rapidly killing every host it infects is no better for the long-
term survival of a pathogen than being wiped out by the immune response before it
has had time to infect another individual.

8. Statement 1 is true as phenotype refers to the observable physical appearance of an


organism, while genotype refers to the genetic composition of an individual that is
responsible for these traits. Phenotype includes visible characteristics such as hair
color, eye color, skin color, height, and weight, whereas genotype contains the genes
responsible for these traits.
Statement 2 is incorrect as genotype is inherited from the parents to the offspring as
one of the two alleles during the reproduction process. The hereditary traits of an
organism may or may not be expressed in the next generation. The same genotype
produces the same phenotype in a particular environment, but phenotype is not
inherited, rather it is an expression of the genotype that appears outside the body as
physical appearance.

9. Option a is correct. Chlorophyll gives algae their green colour and is present in all
algae. However, some algae have other pigments that mask the green chlorophyll,
leading to different colour variations.
Option b is correct. Leaves that appear to be a colour other than green still contain
chlorophyll. The presence of other pigments masks the green colour, but
photosynthesis still takes place in these leaves.
Option c is correct. Plants respond to higher concentrations of CO2 by increasing
their rates of photosynthesis, leading to higher productivity. This has been used in
some greenhouse crops, such as tomatoes and bell peppers, where they are grown in
a carbon dioxide-enriched atmosphere to increase yields.
Option d is incorrect. Chloroplasts contain chlorophyll and carotenoid pigments, not
the cytoplasm.

Science & Technology and Environment by Ravi Garlapati Page 88


10. Statement 1 is incorrect. Complex components of food such as carbohydrates need to
be broken down into simpler substances before they can be used. This process is
called digestion not ingestion.
Statement 2 is incorrect. Taking food into the body through the mouth is called
ingestion. This process can also involve microorganisms absorbing food.
Statement 3 is correct. Breaking down of complex molecules into nutrients in called
digestion.
Egestion: The large intestine is shorter and wider than the small intestine, about 1.5
meters in length. Its function is to absorb water and salts from undigested food
material. The remaining waste forms semi-solid faeces which are egested through the
anus from time to time.

11. Statement 1 is correct. ASF is a viral disease that affects both wild and domestic pigs
and is characterized by acute hemorrhagic fever. It has a nearly 100% case fatality
rate and can be transmitted through direct contact with infected pigs or through
contaminated material and biological vectors like ticks.
Statement 2 is incorrect. African Swine Fever is not a disease that affects humans, but
it can be catastrophic for pigs. It is a non-zoonotic disease, and currently, there is no
effective treatment or vaccine available.
Statement 3 is incorrect. The current outbreak of African Swine Fever in India,
specifically in Assam, is not the first time the disease has been reported in the
country.

12. Option 1, 2, and 4 are correct. Leukocytes or White Blood Cells play a crucial role in
the immune system, as they help fight infections by attacking bacteria, viruses, and
germs that invade the body. They are colorless due to the lack of haemoglobin and
originate in the bone marrow, but circulate throughout the bloodstream. There are
five major types of white blood cells: neutrophils, lymphocytes, eosinophils,
monocytes, and basophils.
Option 3 is incorrect. Erythrocytes or Red Blood Cells (RBC) are the most abundant
cells in the blood and are formed in the red bone marrow in adults.

13. Statement 1 is correct: Insufficient clean water and sanitation, as well as poor
infection prevention and control, can facilitate the transmission of microbes, which
can lead to antimicrobial resistance.
Statement 2 is correct: The development and spread of drug-resistant pathogens
with new resistance mechanisms contribute to antimicrobial resistance, which poses
a threat to our ability to treat common infections.
Statement 3 is correct: Antibiotic resistance can affect people at any point in their
lives, as well as the healthcare, veterinary, and agricultural industries. Antimicrobial
resistance is therefore a critical global public health concern.

14. a. Incorrect. Helminth infections are caused by parasitic worms and can be
transmitted through soil. Hence called STH – Soil Transmitted Helminths.

Science & Technology and Environment by Ravi Garlapati Page 89


b. Incorrect. India has the highest number of STH-infected children in the world, with
64% of them under 14 years of age.
c. Correct. Helminth infections can cause a range of symptoms, including diarrhoea,
malnutrition, weakness, impaired growth, and physical development in infected
children.
d. Incorrect. There are effective medicines available to control the disease, such as
albendazole and mebendazole, which are recommended by the WHO for their
effectiveness, affordability, and ease of administration.

15. Statement 1 is incorrect. The liver is the largest gland in the body and weighs about
1.2 to 1.5 kg in an adult human, situated in the abdominal cavity, just below the
diaphragm, and has two lobes. The average weight of the heart is 250-300 gm and it
is not a gland (glands produce harmones).
Statement 2 is correct. The liver is responsible for the production of a digestive fluid
known as bile, which is released into the gallbladder, a small, pear-shaped organ
located just below the liver in the upper right side of the abdomen.
Statement 3 is correct. The pancreas is both an exocrine and endocrine organ. It has
the dual function of secreting hormones into the blood (endocrine) and secreting
enzymes through ducts (exocrine).
Statement 4 is correct. The liver is the only organ in the human body that can
regenerate. After the diseased portion of the liver is removed, fibrinogen and blood
platelets accumulate in the remaining liver, triggering the earliest stages of
regeneration.

16. Statement 1 is correct. Vitamin A is a group of fat-soluble retinoids, which include


retinol, retinal, and retinyl esters. Preformed Vitamin A can be found in high
concentrations in liver and fish oils, as well as in milk and eggs.
Statement 2 is correct. Vitamin A deficiency can result in night blindness. Vitamin A,
also known as retinol, aids in the conversion of nerve impulses to images in the
retina.
Statement 3 is also correct as Vitamin A is involved in various bodily functions such
as immune function, vision, reproduction, and cellular communication.
In addition, vitamins are classified as either fat-soluble or water-soluble. Vitamins A,
D, E, and K are fat-soluble, while Vitamin C and all B vitamins are water-soluble.

17. Microbes cause a large number of diseases in human beings. They also cause diseases
in animals and plants. But all microbes are not harmful; several microbes are useful
to man in diverse ways.
Micro-organisms such as Lactobacillus and others commonly called lactic acid
bacteria (LAB) grow in milk and convert it to curd. Hence option 1 is correct.
Aspergillus niger is a fungal microbe of great industrial importance. This mold is
used extensively in the production of citric acid and in the production of several
enzymes such as amylases, pectinases, and proteases. Hence option 2 is correct.

Science & Technology and Environment by Ravi Garlapati Page 90


Streptococcus thermophilus (S. thermophilus) is a type of probiotic ("good" bacteria)
found in the digestive tract. It produces lactic acid in the gut. "Good" bacteria such as
S. thermophilus can help break down food, absorb nutrients, and fight off "bad"
organisms that might cause diseases. Hence option 3 is correct.

18. Statement 1 is incorrect. Both DNA and RNA are capable of undergoing mutations. In
fact, RNA is often more prone to mutations due to its instability, which can lead to
faster evolution in certain organisms, such as viruses with RNA genomes.
Statement 2 is correct. RNA contains a 2'-OH group on each nucleotide, which makes
it more reactive and less stable than DNA. RNA is also known to be catalytic, which
makes it even more reactive. In contrast, DNA lacks this 2'-OH group and is
chemically more stable than RNA.

19. Statement 1 is true. Kala azar is a dangerous neglected tropical disease (NTD) that is
endemic in 76 countries. Kala-azar, also known as visceral leishmaniasis (VL), is a
tropical disease that is characterized by irregular fever, weight loss, anaemia, and
swelling of the spleen and liver. It is caused by a protozoan Leishmania parasite.
Statement 2 is incorrect because in India, Kala azar is endemic to Bihar, Jharkhand,
Uttar Pradesh, and West Bengal – that is predominantly the eastern part of the
country.
Statement 3 is correct, indicating that Kala azar is transmitted to humans by the bite
of infected female sand-flies. The World Health Organisation (WHO) estimates that
globally, about 7 to 10 lakh new cases of Kala azar occur annually.

20. Statements 1 and 2 are correct as Hepatitis C and Chickenpox are viral infections.
Statement 3 is also correct as the common cold is mostly caused by various types of
viruses, with rhinovirus being the most common one. However, statement 4 is
incorrect as Malaria is caused by protozoa and not viruses.

Science & Technology and Environment by Ravi Garlapati Page 91


Physics MCQs
Exercise - I Select the correct answer using the
1. Consider the following statements codes given below:
with reference to nuclear fission: a) 1 and 2 only b) 2 only
1. Nuclear fission produces more c) 1, 3 and 4 d) None
waste than nuclear fusion.
2. Atom bomb works on the principle 4. With reference to the Singularity and
of nuclear fission. Event Horizon of a Black hole,
3. The energy released in nuclear consider the following statements:
fission reactions is millions of 1. The Singularity is a location in the
times greater than the energy Centre of a black hole which contains
released in chemical reactions. a huge mass in an infinitely small
Which of the statements given above space.
is/are correct? 2. The Event Horizon is the boundary
a) 1 and 3 only b) 2 and 3 only defining the region of space around a
b) 2 only d) 1, 2 and 3 black hole from which nothing can
escape.
2. Which of the following statements 3. Within the Event Horizon, the
about the Coronal Hole is incorrect? escape velocity is less than the speed
a) These are regions on the Sun’s of light.
surface from where solar winds flow Which of the statements given above
out into space. are correct?
b) They can be the cause of a) 1 and 2 only b) 1 and 3 only
geomagnetic storms on Earth. c) 2 and 3 only d) 1, 2 and 3
c) They usually appear darker than
their surroundings due to their lower 5. With reference to Large Hadron
temperatures. Collider (LHC), consider the following
d) These are easily visible to human statements:
eyes, without using any visual aid. 1. It is built to study particles that are
the smallest known building blocks of
3. Which of the following is/are cited by all things.
the scientists as evidence/evidences 2. Liquid Argon is used in Large
for the continued expansion of Hadron Collider to keep its
universe? (UPSC PRELIMS 2012) components cool.
1. Detection of microwaves in space 3. ATLAS is the largest general
2. Observation of redshift purpose particle detector experiment
phenomenon in space at the LHC.
3. Movement of asteroids in space Which of the statements given above
4. Occurrence of supernova explosions is/are correct?
in space a) 1 only b) 3 only

Science & Technology and Environment by Ravi Garlapati Page 92


c) 1 and 3 only d) 1, 2 and 3 3. Volcanoes, earthquakes, and ocean
waves produce infrasonic sound
6. Which of the following statements waves.
about dark energy is/are incorrect? Which of the statements given above
1. According to Einstein's Theory of is/are correct?
Relativity, empty space can have a) 1 and 2 only b) 2 and 3 only
its own energy. c) 1 and 3 only d) 1, 2 and 3
2. The expansion of the universe will
be decelerated as more space 9. Which of the following statements
comes into existence and energy about Light Interferometer
gets diluted. Gravitational Observatory (LIGO)
3. The effect of gravity supports the is/are correct?
effect of dark energy in the 1. LIGO helped to prove the general
expansion of the universe. theory of relativity using lasers.
Choose the correct option from below. 2. LIGO operates in vacuum
a) 1 only b) 1 and 2 only chambers like the Large Hadron
c) 1 and 3 only d) 1, 2 and 3 Collider (LHC).
3. The collision of two black holes
7. Which of the following statements can produce gravitational waves.
about electromagnetic waves is 4. Only multiple objects can produce
correct? gravitational waves.
1. All electromagnetic waves travel at a) 1 only b)2 and 3 only
the same speed in a vacuum. c) 1, 2, and 3 only d) 1, 2, 3, and 4
2. Electromagnetic waves can
propagate through solid materials. 10. Which of the following statements are
3. Infrared radiation can be utilized true?
for relieving muscle pain and 1. Silicon and Germanium are
tension. commonly used in microprocessor
Choose the correct option from below. manufacturing.
a) 1 and 2 only b) 2 and 3 only 2. Superconductors achieve zero
(c) 1 and 3 only (d) 1, 2 and 3 resistivity due to the absence of
free electrons.
3. Superconductivity can be achieved
8. Consider the following statements:
at room temperature.
1. When a star in a distant galaxy
a) 1 only b) 2 and 3 only
rotates in a direction away from
Earth, redshift occurs in the c) 1 and 3 only d) 1, 2, and 3
electromagnetic radiation emitted
by the star. 11. Which of the following statements
2. Doppler effect is used by traffic regarding Sunspots is/are true?
police to measure the speed of 1. An increase in the number of
vehicles and check for over- Sunspots leads to a larger geo-tail
speeding. area than normal Sunspot activity.

Science & Technology and Environment by Ravi Garlapati Page 93


2. An increase in the number of
Sunspots results in a decrease in 15. Arrange the following particles in
the temperature of the Sun's increasing order of size.
corona. 1. Atoms
3. Sunspots represent areas with 2. Quarks
higher magnetic pressure. 3. Hadrons
Select the correct statement(s) from Select the correct answer using the
the options below: code given below.
a) 2 only b) 1 and 3 only a) 1-2-3 b) 2-3-1
c) 1 and 2 only d) 1, 2 and 3 c) 1-3-2 d) 3-2-1

12. Which type of radiation is used in 16. Consider the following statements
computed tomography? regarding the Singularity Theory
a) X-rays only about the black hole:
b) Gamma rays only 1. Inside a black hole, matter is
c) X-rays and beta rays compressed down to an infinitely tiny
d) X-rays and magnetic resonance point.
2. All conceptions of time and space
13. Which of the following statements are completely break down at the center
correct? of the black hole.
1. Pure water is a good conductor of 3. Sun is a massive star and can turn
electricity. into a black hole at end of the life
2. Water is amphoteric in nature. cycle.
3. Water contracts on heating Which of the statements given above
between 0°C and 4°C. are correct ?
a) 2 and 3 only b) 1 and 3 only a) 1 and 2 only b) 1 and 3 only
c) 2 only d) 1 only c) 2 and 3 only d) 1, 2 and 3

17. Consider the following statements:


14. What are the potential consequences 1. Boiling is the ‘surface phenomenon’
if Earth switches places with Mars? involving change of liquid state to
1. It may result in the extinction of vapour state.
some living organisms. 2. The boiling point of a liquid is the
2. The length of a day on Earth may temperature at which the vapor
increase. pressure of the liquid is less than
3. The orbital period of the planet atmospheric pressure.
around the sun may increase. 3. Evaporation is a ‘bulk phenomenon’
Choose the correct option based on which involves transition of the liquid
the given code: particles into gaseous particles.
a) 1 and 2 only b) 2 and 3 only Which of the statements given above
c) 1 and 3 only d) 1, 2, and 3 is/are correct?
a) 1 only b) 2 and 3 only
Science & Technology and Environment by Ravi Garlapati Page 94
c) 2 only d) None c) 1 and 3 only d) 1, 2 and 3

18. An artificial satellite orbiting around 20. Which of the following statements are
the Earth does not fall down. This is so correct with respect to Sound?
because the attraction of Earth 1. Sound travels faster in cold air than
a) does not exist at such distance warm air
b) is neutralized by the attraction of 2. Sound travels faster in solid
the moon medium than
c) provides the necessary speed for its the gaseous medium.
steady motion 3. Sound travels faster in dry air than
d) provides the necessary acceleration in humid air.
for its motion 4. Sound travels farther in cold
weather than in hot weather.
19. Consider the following statements: 5. Hitting the source with greater
1. Light is affected by gravity. force produces a louder sound.
2. The Universe is constantly Select the correct answer using the
expanding. code given below:
3. Matter warps its surrounding a) 1 and 2 only
space-time. b) 2,4 and 5 only
Which of the above is/are the c) 2,3,4 and 5 only
prediction(s) of Albert Einstein's d) 1,2,4 and 5 only
General Theory of Relativity, often
discussed in media?
a) 1 and 2 only b) 3 only
Exercise – I Key

1. d 2. d 3. a 4. a 5. c 6. a 7. d 8. d 9. c 10. d
11. d 12. a 13. a 14. c 15. b 16. a 17. d 18. d 19. d 20. b

Explanations

1. Nuclear fission is a reaction in which the nucleus of an atom splits into two or smaller
nuclei. The fission process often produces gamma photons and releases a very large
amount of energy.
Nuclear Fusion occurs when two atoms slam together to form a heavier atom, like
when two hydrogen atoms fuse to form one helium atom. While Fusion occurs in
stars, such as the sun, Nuclear Fission reaction does not occur normally in nature.
Nuclear fission produces more radioactive waste than nuclear fusion because very
few radioactive particles are produced in the case of nuclear fusion. Hence statement
1 is correct.
Atomic bomb works on the principle of nuclear fission. Hence statement 2 is correct.

Science & Technology and Environment by Ravi Garlapati Page 95


The energy released by fission is a million times greater than that released in
chemical reactions; but lower than the energy released by nuclear fusion. Hence
statement 3 is correct.

2. Option a is correct: Coronal holes are regions on the sun’s surface from where fast
solar wind gushes out into space.
Option b is correct: These fast solar wind streams sometimes interact with earth’s
magnetic field, creating what’s called a geomagnetic
storm. This can expose satellites to radiation and
interfere with communications signals.
Option c is correct: Because they contain little solar
material, they have lower temperatures and thus
appear much darker than their surroundings.
Option d is incorrect: Coronal holes appear as dark
areas in the solar corona in extreme ultraviolet
(EUV) and soft x-ray solar images. The phenomenon
is typically invisible to the human eyes.

3. Statement 1 is correct: The cosmic microwave


background (CMB) is thought to be leftover radiation from the Big Bang. The
detection of microwaves in space is an evidence for the continued expansion of
universe.
Statement 2 is correct: The redshift phenomenon refers to the apparent shift in the
wavelength of light emitted by objects in space, such as stars and galaxies, towards
the red end of the spectrum. It happens as a consequence of the Doppler effect. This
shift occurs because the objects are moving away from us, causing their light to be
stretched out, resulting in a longer wavelength.
Redshift is a fundamental concept in cosmology, and it provides important
information about the expansion of the universe. In fact, the discovery of the redshift
of light from distant galaxies by astronomers in the early 20th century led to the
realization that the universe is expanding. This discovery eventually led to the
development of the Big Bang theory, which explains the origin and evolution of the
universe.
Statement 3 and 4 are incorrect: Movement of asteroids in space and occurrence of
supernova explosions in space code are not evidences of continued expansion of
universe.

4. Statement 1 is correct: Singularity: It is the one-dimensional point or a small ring in


the Centre of a black hole which contains a huge mass in an infinitely small space,
where density and gravity become infinite and space-time curves infinitely. It is a
tiny volume with very high density.

Science & Technology and Environment by Ravi Garlapati Page 96


Statement 2 is correct: The event horizon: It is the "point of no return" around the
black hole. It is not a physical surface, but a boundary defining the region of space
around a black hole from which nothing (not even light) can escape.
Statement 3 is incorrect: Within the event horizon, the escape velocity (vescape)
exceeds the speed of the light (c) and an object is trapped forever if it falls within the
region defined by event horizon. Outside the event horizon, vescape < c and the
objects are able to escape.

5. Statement 1 is correct: The Large Hadron Collider is a giant, complex machine built to
study particles that are the smallest known building blocks of all things. It is a 27-km-
long track-loop buried deep underground and located at the Swiss-French border. It
is the world’s largest and most powerful particle accelerator.
Statement 2 is incorrect: The LHC uses liquid helium to keep its critical components
ultracold (colder than interstellar space). Since the LHC’s powerful electromagnets
carry almost as much current as a bolt of lightning, they must be kept chilled.
Statement 3 is correct: ATLAS is the largest general-purpose particle detector
experiment at the LHC. ATLAS is one of two general-purpose detectors at the Large
Hadron Collider (LHC). It investigates a wide range of physics, from the search for the
Higgs boson to extra dimensions and particles that could make up dark matter.
In 2012, scientists using LHC’s experiments had announced to the world the
discovery of the Higgs boson or the ‘God Particle’.

6. Statement 1 is correct. According to Einstein's theory of relativity, empty space can


have its own energy, which is a property of space itself.
Statement 2 is incorrect. Dark energy is a form of energy that exists in space and is
not diluted as space expands. Therefore, as more space comes into existence, more
energy-of-space appears, causing the universe to expand faster and faster.
Statement 3 is also incorrect. Although gravity is an attractive force that can slow
down the expansion of the universe, the effect of dark energy counteracts this force,
causing the universe to expand at an accelerating rate.

7. Statement 1 and 2 are correct as electromagnetic waves do not require any medium
for propagation and travel at the same velocity, which is the speed of light in a
vacuum.
Statement 3 is also true as infrared radiation has various uses, including being used
in physiotherapy to alleviate muscle pain and tension, taking photographs of distant
objects in fog or mist, and studying molecular structure through its absorption
spectrum.

8. Redshift and blueshift are used by astronomers to determine the distance of an


object from Earth. When an object moves away from us (Earth), the sound or light
waves emitted by the object are stretched out, resulting in a lower pitch and a shift

Science & Technology and Environment by Ravi Garlapati Page 97


towards the red end of the electromagnetic spectrum, which has longer wavelengths.
This is known as redshift. Therefore, Statement 1 is correct.
The Doppler effect is the change in frequency of a wave as observed by an observer
moving relative to the source of the wave. It is used in radar guns by traffic police to
measure the speed of moving vehicles. If a vehicle is moving towards the radar
device, the frequency of the reflected wave increases, and if it is moving away, the
frequency decreases, allowing the police to determine the speed of the vehicle.
Therefore, Statement 2 is correct.
Infrasonic waves are sound waves with frequencies below the range of audible
sound, typically below 20 Hz. Volcanoes, earthquakes, ocean waves, and meteorites
are some natural sources of infrasound. Therefore, Statement 3 is correct.

9. LIGO is a detector that uses lasers to detect gravitational waves. These waves were
first predicted by Albert Einstein in his general theory of relativity, which states that
massive accelerating objects such as black holes or neutron stars orbiting each other
would disrupt space-time, causing "waves" of distorted space to radiate from the
source. These ripples travel at the speed of light and carry valuable information
about the nature of gravity itself. Therefore, statement 1 is correct.
LIGO operates in a vacuum chamber, similar to the Large Hadron Collider (LHC) in
Switzerland, to minimize interference from air molecules and other particles.
Therefore, statement 2 is correct.
The collision of two black holes is a violent event that produces massive ripples
through the fabric of space-time, known as gravitational waves, which travel at the
speed of light. Therefore, statement 3 is correct.
Gravitational waves can be produced by any object with mass that accelerates,
including single objects like humans, cars, and airplanes. Therefore, statement 4 is
incorrect.

10. Semiconductors such as Silicon and Germanium are materials with conductivity
between conductors and non-conductors that are commonly used in microprocessor
manufacturing for various devices.
Superconductors are materials that exhibit superconductivity, a state in which there
is no electrical resistance. Unlike in conductors such as copper, where free electrons
move randomly and collide with other particles, offering resistance to the current
flow, in a superconductor, all the electrons align themselves in a particular direction
and move coherently without obstruction, resulting in zero resistance. This property
makes superconducting materials highly efficient for use in electrical appliances.
The critical temperature for superconductivity is usually below zero degrees Celsius,
making it difficult to achieve. However, recent research has shown that nanosized
films and pellets made of silver nanoparticles embedded in a gold matrix can exhibit
superconductivity at room temperature and pressure. Therefore, Statements 1, 2,
and 3 are correct.

Science & Technology and Environment by Ravi Garlapati Page 98


11. The Sun's magnetic fields interact with Earth's magnetic fields to create Geo tails
(refer to the image below; a type of blowing back of earth’s magnetic field). Sunspots
have a magnetic field about 2,500 times stronger
than Earth's, resulting in a more compressed
magnetic field of Earth side facing the sun and
increased area of the Geo tail on the other side of
the Earth. This confirms Statement 1. Sunspots
also have a high magnetic pressure and cause the
temperature of the Sun's corona to decrease due to
the concentrated magnetic field preventing the
flow of hot gas from the Sun's interior. Therefore,
Statements 2 and 3 are true.

12. Computed tomography (CT) imaging uses X-rays to create cross-sectional images of
the body. Therefore, option (a) is correct. Beta rays and magnetic resonance are not
used in CT imaging, so options (c) and (d) are incorrect. Gamma rays can be emitted
during radioactive decay, but they are not used in CT imaging. Therefore, option (b)
is also incorrect.

13. Distilled or pure water is free of salts and is not a good conductor of electricity. Water
needs to have dissolved ions to conduct electricity. Therefore, statement 1 is
incorrect.
Water is amphoteric in nature, meaning it can act as both an acid and a base. This is
because the water molecule contains hydrogen atoms and oxygen atoms that can
donate and accept protons. Therefore, statement 2 is correct.
Water has an anomalous behavior when it comes to its density. Unlike other liquids,
the volume of water decreases when heated from room temperature to 4°C, after
which it increases. Therefore, water contracts on heating between 0°C and 4°C.
Hence, statement 3 is correct.

14. If Earth and Mars swap positions, Mars would experience a significant increase in
temperature, which could melt the polar caps and release gases from the soil,
resulting in a warmer, more Earth-like climate. However, the opposite would happen
to Earth, where the intensity of sunlight would reduce, and the planet would freeze
over, causing most living organisms to go extinct. Therefore, statement 1 is correct.
Assuming that Earth's rotation time equals Mars's current rotation time (24.6 hours),
the length of a day on Earth would increase, not decrease, making statement 2
incorrect.
When Earth occupies Mars's position, the planet's orbital period around the sun
would increase since Mars's orbital period is around 687 days. Thus, statement 3 is
correct.

Science & Technology and Environment by Ravi Garlapati Page 99


15. A quark is a type of elementary particle and a fundamental constituent of matter.
Quarks combine to form composite particles called hadrons, the most stable of which
are protons and neutrons, the components of atomic nuclei.
An atom is a particle of matter that uniquely defines a chemical element. An atom
consists of a central nucleus that is surrounded by one or more negatively charged
electrons. The nucleus is positively charged and contains one or more relatively
heavy particles known as protons and neutrons.
Thus, atoms consist of hadrons and hadrons are made up of quarks. Hence option (b)
is the correct answer.

16. According to the Singularity theory, the center of a black hole is an ultimate no man's
land where matter is squeezed into an infinitely small point, and the concepts of
space and time cease to exist. Therefore, Statements 1 and 2 are correct.
Our Sun is an average-sized star that is not large enough to turn into a black hole at
the end of its life cycle. For the Sun to become a black hole, it would need to be about
20 times more massive. Stars that are born with this size or greater can end their
lives as black holes after exploding into a supernova and collapsing back into a black
hole. A white dwarf, not a black hole, is the end stage of the Sun's life cycle. Therefore,
Statement 3 is incorrect.

17. Statement 1 is incorrect: The temperature at which a liquid starts boiling at the
atmospheric pressure is known as its boiling point. Boiling is a bulk phenomenon
(Not surface phenomenon). Particles from the bulk (Not only from the surface) of the
liquid gain enough energy to change into the vapour state.
Statement 2 is incorrect: The boiling point of a liquid is the temperature at which the
vapor pressure of the liquid equals the atmospheric pressure. The vapor pressure is
the pressure exerted by the molecules of a substance in the gas phase, in equilibrium
with the molecules in the liquid phase. As the temperature of a liquid increases, the
kinetic energy of its molecules also increases, and more of the molecules gain enough
energy to escape the surface of the liquid and become a gas.
When the vapor pressure of a liquid equals the atmospheric pressure, the liquid
starts to boil. At this point, bubbles of vapor form within the liquid and rise to the
surface.
Statement 3 is incorrect: Evaporation is a form of vaporisation that happens on the
surface of liquids (not a bulk phenomenon) and it involves the transition of the liquid
particles into the gaseous phase. The liquid particles will generally escape and enter
the surrounding air as a gas when a molecule near the surface consumes enough
energy to overcome the vapour pressure.

18. An artificial satellite orbiting around the Earth does not fall down. This is because the
attraction of Earth i.e., gravity provides the necessary acceleration for its motion.

Science & Technology and Environment by Ravi Garlapati Page 100


Satellites don’t fall from the sky because they are orbiting the Earth. Even when
satellites are thousands of kilo meters away, Earth’s gravity still attracts them.
Gravity, combined with the satellite’s momentum from its launch into space - cause
the satellite go into orbit above Earth, instead of falling back down to the ground.

19. Statement 1 is correct. According to Albert Einstein’s General Theory of Relativity,


Light will be affected the same way as matter is affected by gravity.
Statement 2 is correct. Based on this theory, Hubble found that distant galaxies move
away faster than those that are relatively nearby and the universe is indeed
expanding.
Statement 3 is correct. According to this theory, gravity is caused by a warping of
space and time. Gravity is the curvature of the universe, caused by massive bodies,
which determines the path that objects travel.

20. Sound is a form of energy which produces a sensation of hearing in our ears. Sound is
produced by vibrating objects. The speed of sound depends on the properties of the
medium through which it travels.
Option 1 is incorrect: Sound moves faster in warm air than the cold air. The speed of
sound in a medium depends on the temperature of the medium. Higher the
temperature, faster the speed of sound.
Option 2 is correct: Sound travels faster in solids than gases. The speed of sound
depends on the density of the medium. Higher the density, faster the sound travels.
The velocity of sound decreases as it moves from solid to gaseous state.
Option 3 is incorrect: Sound travels faster in humid air than dry air as particles are
packed more densely in humid air than the dry air.
Option 4 is correct: Sound travels farther in cold weather than in hot weather. It is
because sound moves faster in warm air than colder air, which leads to wave bending
away from the warm air and back towards the ground. This stops the wave. That's
why sound is able to travel farther in chilly weather.
Option 5 is correct: Hitting the source with greater force produces a louder sound.
The loudness or softness of a sound is determined basically by its amplitude. The
amplitude of the sound wave depends upon the force with which an object is made to
vibrate. Higher the amplitude louder the voice.

Science & Technology and Environment by Ravi Garlapati Page 101


Chemistry MCQs

Exercise - I determine if chemical weapons


1. With reference to Click chemistry, were used.
consider the following statements: 4. OPCW reports to the United
1. It can produce exact copies of Nations General Assembly and
natural chemicals. the United Nations Security
2. It can be used to create a plastic Council through the office of
that can conduct electricity. the Secretary-General.
3. It can help in the treatment of Select the correct answer using the
cancer in the human body. codes given below:
Which of the statements given above a) 1, 2 and 3 only b) 1, 3 and 4 only
is/are correct? c) 2, 3 and 4 only d) 1, 2, 3 and 4
a) 1, 2 and 3 b) 1 and 2 only
c) 2 and 3 only d) 3 only 4. They are various structural forms of
the same element that can display
2. Which of the following substances contrasting physical properties and
turn blue litmus paper to red? chemical behaviors. These different
1. rain water forms are influenced by familiar forces
2. pure water such as pressure, light, and
temperature, which trigger their
3. milk of magnesia
transformations. The notion was
Select the correct answer using the
initially put forth by Jons Jakob
code given below:
Berzelius, a Swedish scientist, in 1841.
a) 1 only b) 1 and 2 only
What is being referred to in the above
c) 2 and 3 only d) 1, 2 and 3
passage?
a) Isotopes b) Allotropes
3. OPCW stands for Organisation for the
c) Isobars d) Isotones
Prohibition of Chemical Weapons.
Which of the following statements
5. Which of the following laws or acts are
is/are correct with reference to
relevant to safeguarding against
OPCW?
chemical disasters in India?
1. OPCW is an informal group
1. The Environment (Protection) Act
under the United Nations
of 1986
framework.
2. The Public Liability Insurance Act
2. OPCW provides assistance and
of 1991
protection to its Member
3. The National Environment
States against chemical threats
Appellate Authority Act of 1997
and promotes the peaceful use
a) 1 only b) 1 and 2 only
of chemistry.
c) 2 and 3 only d) 1, 2 and 3 only
3. OPCW can perform testing of
sites and victims of suspected
chemical weapons attacks to
Science & Technology and Environment by Ravi Garlapati Page 102
6. Which of the following statements 1. Phosphine is regarded as a
accurately describe Formalin? biomarker of life.
1. Formalin is a chemical that is 2. Phosphine cannot be produced
derived from formaldehyde. through industrial processes.
2. Formalin is a recognized a) 1 only b) 2 only
carcinogenic agent. c) Both 1 and 2 d) Neither 1 nor 2
3. Formalin is utilized to extend the
shelf life of perishable foods. 10. Consider the following statements
about styrene.
Which of the above statements is/are 1. Breathing in styrene can lead to
correct? health problems.
a) 1 only b) 1 and 2 only 2. Styrene is a key component used
c) 2 and 3 only d) 1, 2 and 3 only to manufacture polystyrene.
Choose the correct statement(s) from
7. How do aerosols differ from the options given below:
respiratory droplets in transmitting a) 1 only b) 2 only
viruses? c) Both 1 and 2 d) Neither 1 nor 2
1. Aerosols are smaller in size than
respiratory droplets.
11. Which of the following statements
2. Aerosols remain suspended in the
about Vanadium is/are correct?
air for a longer period than
1. Vanadium alloys are used in
respiratory droplets.
nuclear reactors.
Which of the statements given above
2. Vanadium is used to manufacture
is/are correct?
steel alloys.
a) 1 only b) 2 only
3. India is the world's largest
c) Both 1 and 2 d) Neither 1 nor 2 producer and exporter of
vanadium.
8. What identification marks are present
in Hallmarked Gold Jewellery? a) 1 and 3 only b) 1 and 2 only
1. BIS mark. c) 2 and 3 only d) 1, 2 and 3
2. Purity in carat.
3. Assaying Centre’s identification 12. Which of the statements below about
number. surfactants is true?
4. Jeweller’s identification mark. a) They are chemicals that are
Choose the correct option: surface active and can form foam.
a) 1, 2 and 3 only b) 2, 3 and 4 only b) They decrease the surface tension
c) 1, 3 and 4 only d) 1, 2, 3 and 4 between two liquids.
c) They are utilized to tackle the
9. Which of the following statements problem of foaming in sewage
about phosphine, a gas discovered in treatment.
Venus' atmosphere by recent d) They are soluble in water but
research, is/are accurate? insoluble in organic solvents.

Science & Technology and Environment by Ravi Garlapati Page 103


13. Consider the following statements 16. Regarding lithium, which of the
about Aqua regia: following statements is/are true?
1. Aqua regia is a mixture of Nitric 1. Lithium is the lightest metal on
acid and Sulphuric acid. earth.
2. Aqua regia is known as royal 2. Excess intake of lithium can cause
water. black foot disease.
3. Aqua regia can dissolve gold. 3. The lithium triangle region
Which of the statements given above consists of Chile, Argentina, and
is/are correct? Bolivia.
a) 1 and 2 only b) 2 only a) 1 only b) 2 and 3 only
c) 2 and 3 only d) 3 only c) 1 and 3 only d) 1, 2 and 3

14. Which of the following statements 17. With reference to Bisphenol A (BPA),
regarding quarks is/are correct? consider the following statements :
1. Quarks are elementary particles 1. It is a colourless crystalline solid
that come in six flavors: up, down, insoluble in organic solvents.
charm, strange, top, and bottom. 2. It is used to manufacture
2. The Large Hadron Collider beauty polycarbonate plastics and resins.
(LHCb) experiment investigates Which of the statements given above
the slight difference between is/are correct ?
matter and antimatter by studying a) 1 only b) 2 only
the "beauty quark" particle. c) Both 1 and 2 d) Neither 1 nor 2
3. Protons and electrons are
elementary particles made up of 18. Generally, curd is not kept in brass
three different quarks. and copper vessels. Which of the
a) 1 and 2 only b) 1 only following is the most probable reason
c) 2 and 3 only d) 1, 2 and 3 for not keeping curd in brass and
copper vessels?
15. Which of the following elements are a) Curd contains Lactic acid which can
radioactive? react with copper & brass to produce
1. Hassium harmful products.
2. Promethium b) Tartaric acid in curd reacts with
3. Technetium copper & brass to produce harmful
4. Zirconium gases.
5. Nobelium c) Curd primarily contains Acetic acid,
Choose the correct answer from the which becomes harmful when it reacts
options below: with brass and copper.
a) 1, 2 and 3 only d) Ascorbic acid in the curd reacts
b) 4 and 5 only with copper & brass to produce toxic
c) 1, 2, 3 and 5 only compounds.
d) 1, 2, 3, 4 and 5

Science & Technology and Environment by Ravi Garlapati Page 104


19. With reference to ‘Litmus Paper’, extinguish fire triggered by electrical
consider the following statements: equipment. Which of the following
1. It is a type of pH indicator that is best describes the reason for this?
used to test the acidity or basicity of a a) Nontoxicity of CO2 even in high
solution. concentration reduces the damage
2. It is made from natural dyes caused to humans trapped in fire.
extracted from lichens. b) CO2 fire extinguishers can
3. It can accurately provide the pH extinguish flammable liquid fires,
value of any solution. flammable solids, and flammable
Which of the statements given above gases.
is/arecorrect? c) CO2 will cut down the contact
a) 1 only b) 1 and 2 only between oxygen and burning object as
c) 2 and 3 only d) 1, 2 and 3 it is heavier than Oxygen.
d) Compared to water, more amount
20. Carbon dioxide (CO2) is considered a of CO2 can be stored in the given
better fire extinguisher than water to space.

Exercise – I Key

1. c 2. a 3. c 4. b 5. d 6. d 7. c 8. d 9. a 10. c
11. b 12. a 13. c 14. a 15. c 16. c 17. b 18. a 19. b 20. c

Explanations
1. The scientists, Carolyn R Bertozzi, Morten Meldal and K Barry Sharpless, have been
awarded the Nobel Prize for Chemistry, 2022 for their work in click chemistry.
Statement 1 is incorrect: Click chemistry cannot provide exact copies of natural
molecules. However, by using click chemistry it is possible to produce molecules that
fulfil the same functions.
Statement 2 and 3 are correct: Click chemistry reactions are now used to create
plastics that can conduct electricity by adding a chemical azide. Manufacturers can
also make them water proof by adding a chemical alkyne.
It can now be used to treat cancer. Bertozzi used click chemistry to make a product
that can be used to study glycans attached to the lymph nodes in the human body.
The product developed by her using click chemistry are now used by researchers
across the world to treat cancers.

2. Acidic solutions are those that have pH value less than 7 and basic solutions or bases
have pH more than 7. An acid/acidic substance turns Blue litmus paper to red and a
base/basic substance turns red litmus paper to blue. As rainwater is mildly acidic
(pH<7), it will turn the blue litmus paper to red. Pure water has pH=7 and hence it

Science & Technology and Environment by Ravi Garlapati Page 105


will have no effect on the colour of the litmus paper. Milk of magnesia is a basic
solution with pH>7 and hence it will also not react with the blue litmus paper.

3. Statement 1 is wrong. OPCW is an intergovernmental organization responsible for


implementing the Chemical Weapons Convention, which has 193 member countries,
and is not a UN organization. Statement 2 is correct, OPCW's objectives are to destroy
all chemical weapons, monitor chemical industry to prevent chemical weapons,
provide assistance and protection against chemical threats, and foster international
cooperation.
Statement 3 is correct, OPCW can perform inspections and testing of sites and victims
of suspected chemical weapons attacks. Statement 4 is correct. OPCW reports to the
UN through the office of the Secretary-General under the 2001 Relationship
Agreement. India's CAG GC Murmu was chosen as an external auditor to the OPCW
for a three-year term starting in 2021.

4. Allotropes are distinct forms of a chemical element that can exist in the same
physical state, resulting from varying arrangements of atoms and their bonding.
Allotropes can exhibit stark differences in their chemical and physical properties. For
instance, graphite and diamond are allotropes of carbon that both exist in the solid
state, with graphite being soft and diamond being extremely hard. Another class of
carbon allotropes includes fullerene.
Isotopes are atoms with the same number of protons (atomic mass) but differing
numbers of neutrons. Isobars are atoms of different chemical elements having equal
values for atomic mass. Isotopes have the same atomic number. Isobars have
different atomic numbers. Isotonesare atomic species that share the same number of
neutrons, and differ in the number of protons.

5. Statement 1 is correct as the Environment Protection Act of 1986 authorizes the


central government to take measures for preserving the environment, setting
standards and inspecting industrial units. Additionally, two rules - the Manufacture,
Storage and Import of Hazardous Chemicals Rules, 1989 (MSIHC) and the Chemical
Accidents (Emergency, Planning, Preparedness, and Response) Rules, 1996 (EPPR)
have been established under the Environment (Protection) Act, 1986 to ensure
chemical safety.
Statement 2 is also correct as the Public Liability Insurance Act, 1991 mandates that
hazardous units must have an insurance policy and deposit an equal amount in the
Environment Relief Fund to provide immediate relief to victims of chemical
accidents.
Statement 3 is correct as the National Environment Appellate Authority Act of 1997
creates the National Environment Appellate Authority, which can hear appeals
regarding the restriction of areas in which any industries, operations or processes
shall or shall not be carried out subject to certain safeguards under EPA, 1986.

Science & Technology and Environment by Ravi Garlapati Page 106


6. Statement 1 is correct. Formalin is a chemical that is obtained from formaldehyde,
which is a harsh-smelling gas and is present in a 37% aqueous (water) solution. It
has the chemical formula HCHO and is used as an antiseptic, disinfectant, and
primarily as a fixative for histology, which is the study of tissues under the
microscope.
Statement 2 is also true. If consumed in excessive amounts over a prolonged period,
it has been found to be carcinogenic.
Statement 3 is correct as well. It is used to extend the shelf life of food, particularly
fish, on fishing vessels in the ocean.

7. Aerosols are particles suspended in the air, including fine dust, mist, or smoke. In
terms of virus transmission, aerosols refer to micro droplets, which are much smaller
(5 microns or less) than respiratory droplets. Hence, statement 1 is correct. They are
expelled when people breathe, laugh or sing, while respiratory droplets are expelled
through forceful acts like sneezing or coughing. Compared to respiratory droplets,
aerosols take a longer time to fall to the floor and can remain suspended in the air for
longer periods. Hence, statement 2 is correct.

8. Hallmarking in India involves accurately determining and officially recording the


proportion of precious metal in articles made of precious metals. The official marks
used in India guarantee the purity or fineness of precious metal articles. Currently,
only two precious metals, gold and silver, are brought under the purview of
hallmarking in India. Gold Hallmarked Jewellery consists of four marks, which are:
the BIS mark, purity in carat and fineness for gold (such as 22K916, 18K750,
14K585), the assay centre's identification mark/number, and the jeweller's
identification mark/number.

9. Statement 1 is correct. Phosphine, a gas discovered in Venus' atmosphere, is


considered a biomarker of life as it is produced by anaerobic bacteria which can
inhabit oxygen-sparse environments. Phosphine is composed of one phosphorus
atom and three hydrogen atoms, similar to ammonia.
Statement 2 is incorrect. Phosphine can be produced through industrial processes, in
addition to being produced by bacteria that survive without oxygen. Phosphine is a
colorless, pungent-smelling gas, known to be generated by certain bacteria species.

10. The Visakhapatnam gas leak, which occurred at the LG Polymers chemical plant, was
caused by styrene gas. Statement 1 is correct, as inhaling styrene can cause various
health problems like eye irritation, respiratory issues, nausea, and even cancer in
rodents. Statement 2 is correct - styrene is a key raw material for polystyrene
synthesis and is also used in the production of fiberglass, rubber, and latex. It is
found in natural foods and cigarette smoke, as well as vehicle exhaust. Styrene is a

Science & Technology and Environment by Ravi Garlapati Page 107


flammable gas stored in factories as a liquid, which easily evaporates and requires
storage under 20°C.

11. Vanadium has various applications. Statement 1 is correct because vanadium alloys
are used in nuclear reactors due to their low neutron-absorbing properties, and
vanadium pentoxide is used as a catalyst for the production of sulfuric acid.
Statement 2 is correct because vanadium is mainly used to produce specialty steel
alloys. However, statement 3 is incorrect as China is the largest producer and
consumer of vanadium, followed by Russia and South Africa. While India is a
significant consumer of vanadium, it is not a primary producer of the metal.

12. Surfactants are chemicals that are surface-active and can form froth. They are often
found in synthetic detergents, and they are a problem in sewage treatment plants
and natural streams because of their ability to form froth. The conventional alkyl
benzene sulfonate surfactants are persistent, and although they are slowly broken
down by natural stream organisms, they remain a problem.
Surfactants are amphiphilic molecules, meaning that their heads are polar or
hydrophilic, while their tails are hydrophobic. They are soluble in both water and
organic solvents. The surfactant reduces the surface tension of water by adsorbing at
the liquid-gas interface. Therefore, the correct option is (a).

13. Aqua regia is a solution composed of a 3:1 mixture of hydrochloric acid (HCL) and
nitric acid (HNO3). Therefore, Statement 1 is not correct.
Aqua regia is commonly known as royal water in Latin. Hence, Statement 2 is correct.
Aqua regia is capable of dissolving noble metals such as gold, platinum, and
palladium from substrates, and is used in microfabrications and microelectronics
labs for this purpose. It is one of the few reagents that can dissolve gold and
platinum. However, it should be handled with extreme caution as it is highly
corrosive and may cause explosions or skin burns. Hence, Statement 3 is correct.

14. Quarks are the smallest known building blocks of the Universe and are elementary
particles. The standard model currently recognizes six flavors of quarks: up, down,
charm, strange, top, and bottom. Therefore, Statement 1 is correct.
The Large Hadron Collider beauty (LHCb) experiment focuses on investigating the
slight differences between matter and antimatter by studying a type of quark called
the "beauty quark" or "b quark,". Hence, Statement 2 is correct.
Leptons, such as electrons, muons, and taus, are elementary particles that carry one
unit of electric charge or are neutral. Protons and neutrons, on the other hand, are
composed of 3 quarks each (of two kinds of quarks, up and down quarks). Protons
consist of two up quarks and a down quark, while neutrons consist of two down
quarks and an up quark. Therefore, Statement 3 is not correct, as electrons are not
made up of quarks.

Science & Technology and Environment by Ravi Garlapati Page 108


15. Radioactivity occurs when unstable atomic nuclei disintegrate spontaneously to form
more stable atomic nuclei with the emission of radiation. Hassium, Promethium,
Technetium, and Nobelium are radioactive elements. It is to be noted that
Technetium is the least atomic numbered radioactive element with an atomic
number of 43.
Zirconium, on the other hand, is a non-radioactive, hard, silvery metal used in making
ceramics and as a semi-precious gemstone. Therefore, option (c) is the correct
answer.

16. Lithium is the lightest metal on earth with a density of 0.534 g/cm3, making it the
least dense metal. Therefore, Statement 1 is correct. However, excess intake of
lithium can cause lithium toxicity, but it is not linked to black foot disease. Therefore,
Statement 2 is incorrect.

The Lithium Triangle is a remote region that spans Chile, Argentina, and Bolivia,
containing over 63% of the world's lithium reserves, with Bolivia's Salar de Uyuni
having the richest lithium deposits. Hence, Statement 3 is correct.

17. Bisphenol A (BPA) is a chemical compound that is commonly utilized for producing
different types of plastics. It is a colorless solid that can dissolve in most organic
solvents but has limited solubility in water. Thus, Statement 1 is incorrect.
The major use of BPA is in the production of polycarbonate plastics, which are used
in a wide range of products such as shatterproof windows, eyewear, water bottles,
and epoxy resins that coat metal food cans, bottle tops, and water supply pipes.
Therefore, Statement 2 is correct..

18. Option a is correct: Curd is obtained from milk by coagulating milk through a process
called curdling.
Curd contains Lactic acid. When lactic acid in the curd reacts with the metals like
Brass and Copper, it causes corrosion and the creation of products that cannot be
digested by the human body.
Option b, c, and d are incorrect:
Tartaric Acid is an organic acid found in Tamarind and
grapes.
Ascorbic acid is an antioxidant agent that functions in
fighting bacterial infections, in detoxifying reactions, and in
the formation of collagen in fibrous tissues. It is found in
citrus fruits.
Acetic acid is also known as ethanoic acid. Acetic acid is a by-
product of fermentation and gives vinegar its characteristic
odour.

Science & Technology and Environment by Ravi Garlapati Page 109


19. Statement 1 is correct: Litmus paper is a simple and easy-to-use pH indicator that has
been used for centuries to test the acidity or basicity of a solution. When litmus paper
is dipped into an acidic solution, the dye in the paper changes color from blue to red.
Conversely, when litmus paper is dipped into a basic solution, the dye changes color
from red to blue. The paper can also be used to test the pH of a neutral solution,
which will not change the color of the paper.
Statement 2 is correct: Litmus paper is made from natural dyes extracted from
lichens, specifically Roccella tinctoria and Roccella pygmaea.
Statement 3 is incorrect: Litmus paper has a limitation of not showing the accurate
pH value of given solution. Litmus paper only shows whether a material is acidic or
basic, while pH strips specify the pH value (alkaline).

20. The lowest temperature at which a substance catches fire is called its ignition
temperature. To extinguish fire, the ignition temperature of the burning objects must
be brought down using certain materials. The most common materials used to
extinguish fire are water, heavy clothes, gases etc.,
Option a is incorrect: CO2 is a toxic gas and highly suffocating gas, whose
concentration of even 9% in the breathing air would make a person unconscious
within minutes
Option b is incorrect: Co2 fire extinguishers can extinguish flammable liquid fires but
are not suitable for use on flammable solids, and flammable gases.
Option c is correct: CO2 being heavier than oxygen, covers the fire like a blanket thus
the contact between the fuel and oxygen will be cut off and the fire is controlled.
Hence for fires involving electrical equipment and inflammable materials like petrol,
carbon dioxide (CO2) is the best extinguisher.
Option d is incorrect: Although it is true that in the given space large amounts of CO2
can be stored by compressing it, it is not the reason why CO2 was preferred over
water to extinguish fire caused by electrical equipment.

Science & Technology and Environment by Ravi Garlapati Page 110


Order on Amazon: https://amzn.eu/d/gNZielj
Purchase soft copy only for Rs. 60/- on Simplify Success App: https://tinyurl.com/2ffksz85

You might also like